182
Tax Office G-302 T, 10-12 1800’s — Customs and Duties 1894 — Pollack declared federal income tax unconstitutional 1909 – Corp I.C. allowed 1913 — 16th amendment Sources of Tax Law: Legis: IRC (Title 26 USC) plus legislative history (more important in tax than other areas of law) Admin: Treasury Regulations/Reg. Rule (Issued by IRS but not binding)/Reg. Proc/Notice (IRS thinking about clamping down on something or expanding enforcement) (Treasury Dept. + IRS) Judicial: Case opinions TP files returns — not audited (done) TP files return — audited — no deficiency asserted TP files return — audited — deficiency asserted — TP Agrees and Pays TP files return — audited — deficiency asserted — TP can either 1) not pay and go to Tax. Ct. for a “re- determination” or 2) pay the tax and sue for a refund in district court or claims court — 1) Tax Ct. appeals to fed. Court of appeals for the TP’s Circuit or 2) Appeal to Fed Ct. Appeals or CT of appeals for fed circuit I. Gross Income A. Benefits in Kind 1. In General Should professor’s salary be taxed? Is something gross income? Basic place to start is sec. 61 of IRC. Salary is included in this section. Regulations would give more specific insight (reg 1.61).

Tax - Harvard Law · Web viewAmount Today/Interest Rate/# yrs into the future/total amount in x yrs 2nd way to think of it: ... Alternative: don’t deduct the repayment in year 2,

Embed Size (px)

Citation preview

Page 1: Tax - Harvard Law · Web viewAmount Today/Interest Rate/# yrs into the future/total amount in x yrs 2nd way to think of it: ... Alternative: don’t deduct the repayment in year 2,

TaxOffice G-302T, 10-12

1800’s — Customs and Duties1894 — Pollack declared federal income tax unconstitutional1909 – Corp I.C. allowed1913 — 16th amendment

Sources of Tax Law:Legis: IRC (Title 26 USC) plus legislative history (more important in tax than other areas of law)

Admin: Treasury Regulations/Reg. Rule (Issued by IRS but not binding)/Reg. Proc/Notice (IRS thinking about clamping down on something or expanding enforcement) (Treasury Dept. + IRS)

Judicial: Case opinions

TP files returns — not audited (done)TP files return — audited — no deficiency assertedTP files return — audited — deficiency asserted — TP Agrees and PaysTP files return — audited — deficiency asserted — TP can either 1) not pay and go to Tax. Ct. for a “re-determination” or 2) pay the tax and sue for a refund in district court or claims court — 1) Tax Ct. appeals to fed. Court of appeals for the TP’s Circuit or 2) Appeal to Fed Ct. Appeals or CT of appeals for fed circuit

I.Gross IncomeA. Benefits in Kind

1. In GeneralShould professor’s salary be taxed?

– Is something gross income? Basic place to start is sec. 61 of IRC. Salary is included in this section. Regulations would give more specific insight (reg 1.61).

– Presumption that anything coming in is income unless there is a specific exemption, but there is no definition of income in the entire IRC

– Would a Cadillac be taxable? Not a problem that is not cash (in-kind), it is compensation.

– What if she gave her a Cadillac because she was embarrassed that Ring was driving an ugly car? Probably still compensation but there are some ways to say it is not income. Maybe not everything that comes from the employer is income. Compensation v. Job-Related

– All the things that might be income — all benefits? What if I value something at 30K but buy it for 20K, is this consumer surplus taxable? No, its just not practical. So its obviously not all benefits are taxed, so one of the factors influencing the definition of income is practicality/administrative implications.

– Factors:

Page 2: Tax - Harvard Law · Web viewAmount Today/Interest Rate/# yrs into the future/total amount in x yrs 2nd way to think of it: ... Alternative: don’t deduct the repayment in year 2,

o Practicality/Uniformity o Incentives of how people want to pay and get paid

– What if dean gave tickets? o Probably income, whether it is transferable or not will influence the value.

– What if Ring just loves teach does not want to get paid is it still income? Yes. – In-Kind raises tremendous questions about valuation…– Steamship tickets are taxable even though it is surprise prize, not actually listed in

sec. 61. Debate in Reginald Turner case is not over whether it is income, but how to value the prize. IRS thinks the value should be the market value of the tickets ($2200). Taxpayer thinks they should be valued at $500, unclear of the justification (not planning on it, not transferable, etc.). Must make sure that the market item and the actual item are actually substantially the same in order to justify.

– Court decides to split the difference. – TP’s did not plan on it, but they could have refused. The court says that if you

win a luxury for yourself, the value might not be at fair market value. A sense that there should be an individualized assessment in determining the value.

– Textbook example: Ring receives sample casebook, is that income? Section 61 implies that it probably should be. It has some value. The question is why it wouldn’t be considered value (Haverly), principal receives the book and then donates to the library and tries to get a tax deduction on his return. He recorded $0 income but then claimed $2000 deduction. Government wants that you can only count it as a deduction if he reports it as income. An equal option that neither reported as income nor is a deduction taken. In Haverly the court wind up makes the TP report income but the court could have also said no deduction. Double benefits are frowned upon (not income but deductible).

– Why is not counted as income if you don’t deduct it? Court says because the IRS may enforce income as it sees fit. The issue of administrative practicality comes up again.

– Air-Conditioning in Ring’s office income/compensation? Does seem like it superficially, it is a benefit and could be an incentive to take a job. Could make the practicality argument but can easily make the argument that it is required for the job (easier to make this argument for a desk). Basic tool of the trade v. decorating amenities will come up again when we talk about business benefits.

– Employer offers to pay taxes (Old Colony), the question is whether this is income? IRS says that it is income because functionally same as salary. Same if employer paid grocery bill.

– Tax Payer argues that seems like an every ending cycle for an agreement to pay someone’s taxes, because every payment of taxes would be considered income, which would in be taxed, so on and so on

– The Court’s answer that it is income and deals with Taxpayer’s fear by saying that they would deal with that scenario when it comes up, right now the IRS only wants to collect the first round of taxes.

– Employer is trying to guarantee take home pay. After Old Colony, how do you make this guarantee? The way is to guarantee a certain gross income using the following formula (called grossing up):

Page 3: Tax - Harvard Law · Web viewAmount Today/Interest Rate/# yrs into the future/total amount in x yrs 2nd way to think of it: ... Alternative: don’t deduct the repayment in year 2,

o You want a certain after tax amount (A) = Taxable Income (T) – Tax Paid (TP).

o A = T-tTo A = T (1-t)o T = A/(1-t)

– Hypoo EE has 100K salary + 200K other income. Employer agrees to payee’s

income tax on salary. Q. How to stack – which $100K is salary in the tax brackets?

31% 300K 28% 200K 15% 100K

o Could choose a bracket or use an avg. tax rate on the 300K. Employer would argue for 15%, employee for 31%.

– Main issues so far:o When does the form of payment actually matter?o When is value play heavily into what we do? When do we need include

something into income because how hard it is to value? o What other policy concerns? If you are choosing to exclude something

from income is it fair? Is it efficient? What broader choices are made?>

2. Fringe Benefits– Relationship between §61 and §119. §119 excludes meals and lodging from gross

income under certain circumstances. Does §61 normally cover meals and §119 is a specific exclusion or does §61 never cover these items and §119 is just to clarify. Makes a big difference as to whether failing 119 automatically makes the benefit taxable income.

– Benaglia, this is pre-section §119. The Basic issue is whether the meals and lodging given to a hotel manager and his wife taxable income. The TP’s argument that it is not is that it is for the “convenience of the employer.” Argues that in order to do this kind of job you need to be on the premises. Why do we care whether it is for the convenience of the employer? Because it may be less of a benefit if it a convenience of the employer? Why would we think this to be the case? One reason is that affect’s the employee’s flexibility, it is not full benefit just like Reginald Turner. You might no take the job if you need to report it as income, to someone in Benagla’s social class would not spend the fair market value for the benefit just like Turner.

– A real sense that it seems harsh that the whole room board to be considered the employee’s income. But why not charge at least some of it as income? Why does convenience of employer eliminate it all? Seems like practicality is a factor yet again.

– In Benaglia, IRS makes the argument that he is managing two hotels, and he can and does only live at one so this defeats that it is necessary to perform the job.

Page 4: Tax - Harvard Law · Web viewAmount Today/Interest Rate/# yrs into the future/total amount in x yrs 2nd way to think of it: ... Alternative: don’t deduct the repayment in year 2,

Also refer to the language of the letter which refers to a negotiation over this “benefit.” Also the employee went on extended trips, which indicates it is not a necessity.

– In the end the Court says that if you can characterize it as primarily for the convenience of the employer it will not be accounted as taxable income.

– There is an alternative to this all or nothing “convenience of the employer” Benaglia method

o Tax on replacement cost o Threshold, above which is taxable income o Address the mismatch (no taxable income but also no deduction for the

employer)o include just the cost to the employer as taxable income to Benagliao Tie threshold to salary

Topics:EE Fringe Benefits– Meals/Lodging §119

o Meals, Kowalski– Lodging

o §119 generallyo §119 (d)o §107

– Other Statutory Fringeso exampleso cafeteria plns §125o other fringes

– §119 passed in 1954– What if professors get free food at Hark? 119 requires on premise and for the

employer benefit. Premises? Yes. Employer Benefit? Probably not, not necessary for professors to be on-call for their entire shift. Better argument for security/health personal. Argument the other way that professor-student interaction is for employee benefit.

– Reg 1.119 says that it must have a substantial non-compensatory component (strengthening the employer benefit piece)

– Under 119, does not cover lunch at ABP, because must be on the premises, this requirement enforces that it is for employer benefit and it makes 119 less expansive

– Is free food and housing always a benefit? Military food? – Kowalski

o Issue: Are his meals taxable income?o TP arguments

Covered under 119: It is for the benefit for the employer

Page 5: Tax - Harvard Law · Web viewAmount Today/Interest Rate/# yrs into the future/total amount in x yrs 2nd way to think of it: ... Alternative: don’t deduct the repayment in year 2,

Eating anywhere in the state is on the premises of his employer (the state itself)

Covered under common law doctrine of convenience of employer

o Ct. Majority 119 only covers in-kind meals according to the legislative history,

and this is cash allowance rejects the idea that the whole state is the employer premises No substantial non-compensatory purposes — negotiated with

union, increases with your rank, advertised as a benefit, etc. Says the 119 leaves no room for the common law doctrine of

convenience of employer, congress passed 119 to clean up the previous mess

o Dissent Says that there is no explicit distinction between in-cash and in-

kind distinction in sec. 119 But giving cash obviously has more potential to be for

compensatory measures Says that there is a good reason they switched to from the system

which provided in the barracks to the current system (security concerns)

Congresso Provides relief to Kowalski, and other troopers who did not report up to

decision but did not provide refunds to those who did report it as income prior to the decision

One rationale is that the administrative costs of collecting the back taxes not worth the extra revenue

But it undermines policy going forward Incentive not to report cases on the fence

– How could you get around Kowalski? o Old Scheme of Barrackso Chain Restaurant Deal/Meal Tickets (still have premises problem)

– There are others to cover meals besides 119, which we will look at later…”meal money,” travel, entertaining clients, etc…if fail 119, try to recharacterize to fit under another section

Housing: Under 119 (a), if Law School builds Kagan a house on Holmes, does she have

to count it as income?o §119 says that the employee must be required to accept, on the

premises, and for the benefit of the employer. There is no piece of required to accept for food. Adding the requirement into the contract is no big deal if both parties are amenable. Seems like can argue that she satisfies the other two requirements.

Page 6: Tax - Harvard Law · Web viewAmount Today/Interest Rate/# yrs into the future/total amount in x yrs 2nd way to think of it: ... Alternative: don’t deduct the repayment in year 2,

o Still ok if she brings her family to live there, under 119 (a) spouses and dependents are covered.

o Is Brattle Street ok? Owned by the university but not on the campus. Could argue that it is still on the premises because it is still part of the university.

– 119 (d) idea: if you rent from university for which you work, we won’t do a very specific FMV rental analysis to see if you are a getting a special deal and should be taxed —> just see if they meet the §119 (d) test, worried that FMV – charge = benefit

– Method under 119(d) o (a) 5% of appraised value or (b) average rentals from non ee’s and

non-students received by University for comparable housingo Subtract the lesser amount rent paid by EE of (a) or (b) o If $0 —> no tax o If —> tax on differenceo *Note that Fair Market Value (FMV) does not appear in this formula

– Hypoo House FMV $200K, 5% =10Ko Average Rentals = $11K (unit might charge less than FMV to be nice) o FMV Rental = $12Ko TP pays $10,500o Method

Compare 10K and 11K, take the lesser which 10Ksubtract 10,500, which is –500 and $0 taxable income.

– Why this special benefit to universities and university employees?– Sec. 107 if a minister gets free housing or money to pay for rent, not taxable,

money must actually be used for housing– Why have 107 in the code? Common practice. There is a constitutional

objection in Warren, but the original issue was whether only the FMV could not be counted as income.

– Why these particular benefits? What is the effect of having these exclusions? – Section 125, cafeteria plan

o Employer gives a selection of benefitso Used to be that the employer would give you the option of some cash,

life insurance, child care, etc. which would otherwise would be tax-free, under the old rule you would be taxed if you chose cash

o Under the old rules if you chose you other in-kind benefits covered under IRC as tax free benefits, even though they were offered cash, you would still be taxed precisely because you were offered the option of taking cash, this meant that if employers wanted to offer these benefits they could not offer the cash-option

o This set up a scenario where different groups of employees were competing

o Under section 125, the option of cash does not taint the otherwise tax-free status of the benefit, still taxed on cash, of course

– Other fringe benefits

Page 7: Tax - Harvard Law · Web viewAmount Today/Interest Rate/# yrs into the future/total amount in x yrs 2nd way to think of it: ... Alternative: don’t deduct the repayment in year 2,

o Prior to 132 (1984), lots of ruling and decisions in which there is non-statutory exclusions of income, Congress wants to clean it up (same motive as 119), going to replace all the non-statutory exclusions out there.

o If it’s not in 132, the legislative history tells us it is not excludedo I work for AA and I get free personal flights off-duty, is this taxable in

1984? Suppose the seat would have been empty and get no food, then no tax because covered under 132 (b) No-additional cost service. Can make some arguments that there is extra fuel, labor, etc. And it is really possible that there was no possible sale at all?

o Why is no-additional cost even relevant test for excluding it from income? One reason we generally don’t like to waste things. Another reason is that employer cannot make a simultaneous deduction to this empty seat since it is no additional cost. No double benefit going on.

o What are different ways to look at employer giving employee free seat?

132 (b) treats it as there is no tax consequence The other way to look at is to give $100 income to EE and

$100 cost to employer, which is deductible. Next EE spends $100 on seat, taxable income to employer. The net is = 0, so this matches 132 (b) for the employer but the employee bears a tax burden. 132 (b) removes the employee tax burden.

o For this to qualify as a benefit under 132 (b) benefit can’t be discriminatory (i.e. only offered to highly

compensated employees) must be an employee

132 (h) includes spouses, dependents and retired employees

132 (h) (3) For airlines, parents also can claim income exemption, why?! Maybe it is because airline tickets are not as easily transferable, goods v. services. Also, Congress seemed sensitive for business practice, didn’t want to carve it out if historically employees were getting it for free

Benefit has to be in the line of business in which you work, if AA starts making Soup, and you work for the soup part of the company can’t get airline seats, tax-free, maybe don’t want to give an big advantage to large companies

o 132 (c) qualified employee discount, discount up to a certain amount

will be tax-free, again, why? One reason is that it is not completely compensatory -- employers want employees to endorsing and more familiar with the product.

o 132 (d) employers give you something really work-related, even more

so than product from employee discount (i.e. desk, paper, etc.).

Page 8: Tax - Harvard Law · Web viewAmount Today/Interest Rate/# yrs into the future/total amount in x yrs 2nd way to think of it: ... Alternative: don’t deduct the repayment in year 2,

If you broke it out to $100 of income for paper and $100 deduction, you would get the same result but this makes it much easier.

o 132 (e) de minimums Trivial items (both goods/services) received by employee

which the administrative costs of tracking these as income would be far in excess of the tax (but cannot be so frequent that they are no longer trivial)

Could supper money in Kowalski be covered under this provision even though it was not covered under 119? Regs say that the meal money must be occasional, for overtime and money must be spent during the time period.

o Parking? What if only the president gets a parking spot? Everyone else

gets a charge, isn’t this discriminatory? 132 says that parking can be discriminatory.

o Gym Membership? Has to be non-discriminatory (in sec. 274), on the premises and

operated by the employer Can try to work with the term “operated” and premises by in

the provision to try to get around actually opening a gym in your building (i.e. a private room in Bally’s)

Congress wants to make it hard to get this benefito 117 (d) tuition assistance

Must be non-discriminatory o Hypo

Employee travels for work and employer lets employee keep FF miles and use them for personal use

It obviously could be considered income under §61 since it is a benefit you are getting as an employee from the employer

There is no special code making them exempt from income, could argue that they are no-additional cost fringe benefit, but this is not in the line of business.

Could also argue that it is de minimis benefit, can imagine many administrative obstacles to counting and valuating the frequent flyer miles (i.e. when they are received v. when they are used, different prices, bookkeeping, singles, revocability)

The current state is that the IRS will not enforce their reporting What if employer offers to pay $ to buy back frequent flyer

miles? Rulings indicated this is income. Would you ever have to report frequent flyer miles

accumulated on your own? Obviously not, because you are paying for it.

Page 9: Tax - Harvard Law · Web viewAmount Today/Interest Rate/# yrs into the future/total amount in x yrs 2nd way to think of it: ... Alternative: don’t deduct the repayment in year 2,

3. Imputed Income– Imputed income is when you use your own resources and time to create

something you then yourself use. This takes place outside the market.– Hypo

o A and B both buy houses for $100,000o A rents out his house for $10,000, which he uses to rent a house

from C for $10,000o B buys a house for $100,000 and lives in o As an economic matter A nets 0, as a tax matter he is taxed on the

rental income and his rental expenditure is not allowed to be deducted

o For B, there is nothing to report on his return o At the end of the year, A and B both own the similar houses and

live in a similar house, but A is being taxed on $10,000 of income o The idea of imputed income reflects that we can think of B renting

from himself, so he should be treated the same as A. o B’s income is outside the market and we just don’t tax this

transaction. o We could also make rent deductible as alternate to imputed income

to even things out. – Some other examples:

o Car, washing machine, computer, furniture, etc…– Why?

o Cultural inclination of owner-occupied home o Political resistance to being taxed on theoretical income

– A (lawyer) makes $10,000 and pays $5,000 B (gardener/housekeeper), IRS is taxing $15,000. They get married and continue doing what they are doing but now the IRS is only taxing $10,000 even though the same amount of work is taking place. One thing to think about is that there is no check to see whether the work was really done since it operates outside the market (what if she stopped doing her work). Extraordinary protection given to transactions within the home/family.

– Assume 30% tax rate, A + B one of them stays at home and is considering leaving for a market job:Pre-Tax After-Tax

Home 8,000 8,000Market 10,000 7,000

So at the end of the day we walk away with more value if you stay at home and don’t enter the market.

– A (lawyer) B (Doctor) who exchange an annual check-up and a provides an estate planning review o Services for services (bartering) covered under §61 and are taxable

Page 10: Tax - Harvard Law · Web viewAmount Today/Interest Rate/# yrs into the future/total amount in x yrs 2nd way to think of it: ... Alternative: don’t deduct the repayment in year 2,

o What about if it is between neighbors? Could argue that this is more like a non-market transaction than market bartering. Have a better argument if neighbors are doing less commercial things for each other (helping each other with yard work, etc.)

– A is a reporter, earns $800/day, A is thinking about painting her house (worth $500) – how to decide if paint yourself or hire out?o What is the tax rate? If it 50% then her take home is $400, so she should

paint herself even though the market values her work as a reporter. This is one effect of imputed income because the $500 is not taxed

– Bottom Line:o If you see something that looks like barter that is taxable, but if

what you are doing is for yourself or from you family that imputed income not taxable.

o Should we be taxing leisure? If you could work on Sat and get $800 and instead you lounge in the Sun aren’t you in effect getting a $800 benefit? Should we think about leisure as something we want to tax? What about taxing you on your ability? You could have taken an $800 job but you take a $100 job because it is more personally satisfying, you are obviously receiving at $700 in benefit?

o WHY NOT? The answer may vary at different stages!– Family Farm?

o You eat a significant portion of the food you growo Seems like a better case that this person should be taxed because

everyone else who works to put food on the table have to pay taxes

B. Recovery of Capital1. Introduction to Basis

– Hypo: Buy land for $1000, sell for $1500 o Only bear tax on $500o $1000 represents a portion of your money, which presumably you

have already been taxed on, only if you earn more than that do you get anything back

– §1001, §1011, §1012 – these §’s outline the gain formulao $1500 (amount realized) -$1000 (basis) = $500 (gain/loss)

– What would happen if we taxed you on the $1500?– Hypo: War price control and official price of car is $10K, you pay $15K

and then you sell it for $15Ko TP would like to report $0 income (basis of $15K)o IRS would say that TP should be report $5K (basis of 10K,

because that is all you were allowed to legally invest in the car).o Brings up the idea whether basis should always be what you pay or

could it be some other standard value? Should consider criminal regime in tax regime?

Page 11: Tax - Harvard Law · Web viewAmount Today/Interest Rate/# yrs into the future/total amount in x yrs 2nd way to think of it: ... Alternative: don’t deduct the repayment in year 2,

o How about the constitution? Clearly, yes, the 16th amendment limits what Congress can do. Constitution limits the definition of income separate from IRS and Congressional limitations.

o Aside: the system DOES NOT adjust for inflation.

2. Damage Payment– Car crash and ins. co. pays you back for 11K? Analogize to a forced sale, 11K is

amount realized now we need to figure out the basis, which is probably higher, something like 19K. So unlikely to have any gain.

– But what about if you bought if for $4K and it appreciated? Then your gain is 7K. – What about employment salary of 100K? What is the basis? Don’t think of it in these

terms because it is human capital, which we practically treat the basis as $0. – Raytheon settles with RCA in anti-trust for $440 K, $60K is some licenses which is

taxable, but the status of the $380K is up in the area.o The issues whether the settlement is for lost profits or for

lost/destroyed of goodwill Lost/Destroyed will is like a forced sale

Court rules it is for good will, because basically their business is gone

o Amount realized (380K) – basis (cost incurred to create the goodwill, in this case needs to be determined)

o Court says it will assumed basis is $0 because burden is on TP to prove greater than $0, TP has more access to information than IRS

o What sort of expenditures would you make to create goodwill?

Marketing Lost profits you are still in business

This is taxed like any other profitso What is the big picture rule of Ratheon?

Tax treatment of damage payments depend on character of the damages, treat it like it what it replaces:

If it is replacing a sale, treat it like a sale, if it is replacing lost profits treat like lost profits

This is opposed to characterizing it based on lawsuit – Dean Kagan sues the Record for copyright infringement and gets $. This is the

issue in Raytheon. How would you describe what the money is for? It is a like a forced license of her copyright (forced royalities), not a forced sale because she retains the copyright. So this is ALL taxable income, royalties are income under §61.

– Grenshaw Glass:o Basic issue is whether punitive damages are taxable income. Why

can’t we just use the Raytheon rule is treat the damages like what it

Page 12: Tax - Harvard Law · Web viewAmount Today/Interest Rate/# yrs into the future/total amount in x yrs 2nd way to think of it: ... Alternative: don’t deduct the repayment in year 2,

replaces? Because it is not clear that it replaces anything, it is a bonus/penalty.

o Court says that Congress did not indent to exclude punitive damages, §61 intended to cover as much as it could within the bounds of the constitution (Eisenhower case argued for a more restrictive interpretation of §61)

– Hypo:o Inajanna Land

1920 TP buys fish prop. for $100, someone pollutes and pays TP $20 in 1990, TP sells property in 1995 for $120

Cash Flow Total Income If want to tax $20 in ‘90

If want to exclude $20 in 1990

Alternative: Partly taxed in 1990

1920 cost $100

1990 receive $20

1995 sale $120

AR = $140 - B = $100T = $40

1990: Report income of $20

1995: Report AR of $120, B of $100, report income of $20

1990: Receive $20, but not taxed (must be assuming that the basis $20)

1995: AR is still $120, basis is $80 because $20 of your investment was returned, income of $40

1990 AR of $20, Basis of something greater than $0 and less than $20, asses how much you paid for the stick in the bundle that you were compensated for ($3)

In 1995, AR is 120 and B is $97

– What is the difference in the regimes? Time value of money, can make more money if the bill comes later rather than sooner…

– Court’s use the 1st regime in dividends and try to use the 3rd when they can.– Hypo: Bush, Sr. buys lands in Maine in 2 parcels: 27 acres including coast for

$1.4M, 3 months later only 3 non-coastal acres for $100K, later Bush sells 24 acres of non coastal prop for $1.24MMo Tax Return Bush wants (option #2)

AR $1.24MM and B of $1.24MMo IRS will argue for option #3

AR of $1.24MM Each non coastal acre is $33K, $33 x 24 = $800K, which is the

basis Taxable income of $1.24MM - $.8MM

Page 13: Tax - Harvard Law · Web viewAmount Today/Interest Rate/# yrs into the future/total amount in x yrs 2nd way to think of it: ... Alternative: don’t deduct the repayment in year 2,

– Clarko TP pays too much (19K) because tax lawyer screws upo Issues is whether this is like Old Colony situation of someone paying for a

third-party’s taxes o CT says this is not a payment of taxes, no additional compensation, it is

meant to fix a harm, what is replacing? It seems to replacing something you should have enjoyed tax-free.

o Here they are replacing the “right to the ‘best return’” and this should be enjoyed tax-free, and here this tax-free benefit is solely being restored

A (no Loss) B (Loss, No Reimb) C (Loss, Reimb)Out of Pocket 0 -20K (-)20 + 20 = 0Tax Consequences 0 0K 0 (clark)

o B could argue that he should get a deduction for loss of $20K because he is a worse position than A and C

o B could also argue that effectively we are giving C a deduction because letting them offset with the lawsuit damages, B says let me do the same thing

o But what is the policy for not letting B deduct? Because in C, there is some check on the legitimacy of the claim since someone verified the loss by paying it for you. B requires more investigation by IRS and we don’t want to open the floodgates.

o 1950 Victims got payments from German gov’t and also POW got payments, how would we think about this? What is it reimbursement for? Suffering/Imprisonment, which are personal freedoms, you should be able to enjoy personal freedoms tax-free for payments for this should not be taxed.

o What if lawyer says that he gives up personal freedom for his salary, shouldn’t be taxed? This is voluntary, however so not the same obviously.

o Dean Kagan is hit by a car, injured and recovers $50K from the bus company. Is this taxable? A split between the logic of how to answer this question v. what the statute has to say about this subject (Section 104).

o Read through C.2.

Hypo, continued:– 50K unspecified damages to dean from bus driver

o Raytheon dictates that you must specify the amounts were for, can’t get out of taxes by accepting unspecified settlement

o Pre-104, lost income would be taxed, punitive damages would be taxed, pain and suffering would probably be taxed as income (but Dean could argue that it is just re-compensation not a gain, Raytheon line of reasoning that replacing something that is tax free)

o Post-104, starting point is no-tax, then there are exclusions (10K hospital bills, 30 K lost wages, 10K pain and suffering)

Unlike pre-104, lost wages are NOT taxed

Page 14: Tax - Harvard Law · Web viewAmount Today/Interest Rate/# yrs into the future/total amount in x yrs 2nd way to think of it: ... Alternative: don’t deduct the repayment in year 2,

policy sentiment: o never made whole by compensation settlements so

they should be generous o saves the administration costs/arguments of

breaking down lump-sums) o There is no basis, here is a limit of basis-analysis

punitive damages are taxed – Same as above but no one to sue

Real World (A has no one to sue, B does) Tax WorldA B A B

1000 (i) 1000 (i) 1000 +1000(-)500 (l) (-)500 (l)0 (c) + 500 (c)500 (n) 1000 (n) +500 +1000

(i) = income; (l) = loss; (c) = compensation; (n) net income

A is worse off in the tax world, where B’s taxable income corresponds to the real world. This because you can’t deduct for self-compensation. This is similar to Clark hypo, unfair when people can’t identify the person to sue but need this in order to guard against fraud. What 3rd party is going to pay you 500 unless he loss really took place.

– §213 says deduction for medical expenses, can Dean both deduct medical expenses and not count it as income under 104? No, both sections state that you can’t get a double benefit (same idea as Haverly/textbook case)

– Suppose you receive payments under an accident policy, not a health insurance policy (10K for a lost arm)

o Accident and health plans are separatedo Excludable if self-insured accident insurance

– 1% chance of injury in a no tax world you spend $1 premium to get $100 insurance– Tax World(s) of 50 % tax rate

o Premium are deductible but receipt is included in income Premium

BT - $2 AT - $1 (you save a dollar through the deduction)

Receipt Before Tax – need a $200 policy After Tax - $100 of coverage

Looks like taxes don’t change behavioro Premium not deductible, receipt not included

Premium BT $1 AT $1

Receipt

Page 15: Tax - Harvard Law · Web viewAmount Today/Interest Rate/# yrs into the future/total amount in x yrs 2nd way to think of it: ... Alternative: don’t deduct the repayment in year 2,

BT $100 AT $100

Again looks the same a no-tax world, in theory – What if employer provides the insurance policy? We are in §105 (a) if employer pays

for it. (§104 if you bought your own insurance). o §105 (b) says no tax on medical payments, this exception almost

completely undoes (a) brining us almost back to §104o §105 (c) even more exclusions under accident-type payments

But if what you are really being paid is lost wages, it is taxable, i.e. it is being calculated by how much time you missed work (so you would rather sue the bus driver than get paid by an employer plan)

Medical Plan Premium Paid Receipt Net EffectEE provided May be

deductible under §213, limited

§104 (a) (3) excludes

Double Benefit

E’or provided Not-taxable under §106

Excluded under 105 (b)

Double Benefit

This is not similar to the tax-worlds above because we are allowing exclusion and deduction relating to the same thing, a double benefit.

Accident Plan Premium Paid Receipt Net EffectEE provided No Deduction Excluded under

104 (a) (3) Single Benefit

E’or provided Not taxable under §106

Excluded if fits under §105 (c), can’t look like sick pay/lost wages

Double Benefit

Observations:

1) draw a distinction between the 2 types of plans 2) Favors medical, a little more cautious in our generosity of accident plans 3) Favor employer provided benefits over employee-provided benefits

What about personal injury other than physical/accident/illness? Threlkhold/Burke/Scheier

Threlkhold: if you someone for defamation and you recover is that taxable? Originally, court tried to draw line between business and personal reputation? If for business may be

Page 16: Tax - Harvard Law · Web viewAmount Today/Interest Rate/# yrs into the future/total amount in x yrs 2nd way to think of it: ... Alternative: don’t deduct the repayment in year 2,

for lost wages. Casebook points out that before 1996 defamation was covered under §104 as excludable (probably incorrectly)

Burke: Sex-discrimination claim seeking back-wages, question is whether the back wages were excludable under §104? Sup. Ct. says it is taxable because it is not-tort like, they are like lost wages. After this Congress fixes the discrimination law to make it look more tort-like to make it exempt under 104.

Schleirer: Age-discrimination, Court says again that it is back pay so not exempt under §104. Congress could have changed this law to make it look more like a tort as well, but they didn’t.

Instead, in 1996 Congress narrow’s §104 by adding a physical injury/illness requirement. Prior versions just said “personal injury.”

– Hypo: Hit by cycle, get $100 for hospital bills, $200 for lost wages and $100 and pain and suffering

o All of it is excludable, once you get hit, all of it can be excluded Don’t have to argue whether pain and suffering is physical

– Get $5000 for hospital bills and $5000 for punitiveo Only $5000 for hospital bills excludable, punitive is off the table

– How about psychic reading injury?o Taxable under §104 because not physicalo Emotional distress is not excludable, unless to the extent you incur actual

medical expenses (therapy bills) – Policy concern/thoughts

o Priority to medical attentiono Related to the incident o What about an ulcer after IIED?

Clearly excludable – If there was the physical accident and you are suing for IIED it is excluded, this is

important, even if you specify emotional distress separately, if it derives from a physical accident it is excluded

Physical Injury/Illness Emotional Injury Defamation/DiscriminationAll excluded except punitive

Not excludable except for medical costs (last sentence of §104)

No exclusions

– What incentives does this create?

o §104 specifies medical bills for IIED but not Defamation/Discrimination, so if you are suing for an ulcer caused under Defamation want to add an IIED claim as well

Page 17: Tax - Harvard Law · Web viewAmount Today/Interest Rate/# yrs into the future/total amount in x yrs 2nd way to think of it: ... Alternative: don’t deduct the repayment in year 2,

– Hypo: Sue for defamation & win: $10K loss of business, $10K embarrassment/mental suffering

o $10K for loss of business will definitely be taxed under the background rule that lost profits are always taxable unless excluded under §104

o $10K for embarrassment/mental suffering would not be excluded because not a physical injury under for §104

TP could argue that this payment is more like making you whole rather than lost-profits, something you would normally enjoy tax-free because §104 does not explicitly say this is taxable, but there is a strong implication that it is

3. Tax-Benefit Doctrine– Sanford & Brooks

o Status of recovery under K dispute in 1920? Is it related to losses or a gain?

o TP was doing contract work for gov’t, from 1913-1916 they did work and had expenses of $176K, they then sued the gov’t and recovered 176,000K on the contract.

o TP argues that the $176K is not taxable because it is just recovery of losses from 1913-1916, they want the court to use a transactional approach rather than a year-to-year analysis for gains/losses, this transaction would net to zero.

But from 1913-1916, TP was deducting the 176K as losses But they argue that reality was that they had losses larger than

176K so having the additional deductions did them no good in the years they would taken (no benefit derived), therefore there is not double deducting

o So between 1913-1916 paid no taxeso In 1920, they recover 176K, is that amount income?o TP says no because:

under a transactional view, focus on the fact that the deductions unused

Recovery of capital outlay, 176K was just a recovery of investment Damage Payment, loop this back to #1 or #2 Carry over excessive net-losses from previous years, even things

out over years of operation, attack the annual analysis of taxationo IRS says yes because

Taxes are calculated on annual basis not transactional, and in 1920 this was income, this is constitutional

– Policy justifications: Administrable (accounting and collection) and foreseeable

– Under TP’s argument there is tremendous amount of uncertainty (i.e. when will the “transaction” be completed? Probably only at death of business or person)

– But why a year, isn’t this arbitrary? Court still agrees with IRS, says annual calculation is constitutional and appropriate.

Page 18: Tax - Harvard Law · Web viewAmount Today/Interest Rate/# yrs into the future/total amount in x yrs 2nd way to think of it: ... Alternative: don’t deduct the repayment in year 2,

– Dobsono Buys 300 shares in 1929, sells 100 shares at a loss in 1930, sells another

100 shares in 1931 at a loss, in 1939 he wins settlement ~45K for fraud against the companies, split in part over 1930/1931 sales

o He took deductions for the losses in 30/31 but he had huge net losses so no real gain (like above), can’t go back and change his 30/31 returns because those years are closed

o Question is whether settlement $ income?o TP says no

He still recovered less than his original investment even with the settlement

o Here Court agrees says not taxable because he never recovered his basis, no real income

o In order to see this Court has to look back to 29/30o Tax Court says not taxableo Ct. Appeals says it is taxable and scolds Tax Court for opening returns

from previous years, which is forbidden and for accounting on equityo Sup. Ct. agrees with tax court, saying that:

Need more deference to the Tax Court findings Says that the Tax Court did not open closed years, all they did was

in trying to characterize a current issue they went to look at facts, to find the basis for recovery – they are drawing a line between opening up old tax returns and simply fact finding, where you are allowed to look at previous years

– A distinction drawn from Sanford is that this is an investment rather versus a deduction, but not really clear how meaningful this distinction is (both of them laid out cash in earlier years and recovered some in a later year)

– Court says that somehow tax system protects investments more than deductions

o Think about this distinction throughout the course Weak distinction:

Investment – you acquire something Deduction – you spend

Is there a more constitutional protection of investments/basis?

– Even though constitutional not to allow TP to use unused losses but Congress passed 3 Mechanisms of today to help taxpayers in similar situations (Don’t need to know the specifics of all 3, just that they are related but a little different)

o §111 Generic tax benefit provision (everything not covered §186) Only use this after a generic NOL is expired under §172 the income has to be related to a previous deduction (transactional

link)

o §172

Page 19: Tax - Harvard Law · Web viewAmount Today/Interest Rate/# yrs into the future/total amount in x yrs 2nd way to think of it: ... Alternative: don’t deduct the repayment in year 2,

Allows TP’s to use unused losses, NOL (Net operating loss) can go back 2 years and brought forward up to 20 years

Income does not have to be related (no transactional link required), just need unused overall losses

o §186 Clarification/expansion of §111 In anti-trust/contract suit in a later year, the amount you received

you can reduce by any unused loses relating to the claim (this would be useful to Sanford)

– Hypo under §172 o 1913,14,15,16 – 10%o 1917, 1918 – 10% -- WWIIo 1919, 1920 – 70%o Sanford can carry extra losses of 1913-1916 of 173K, Sanford would have

been much better off to carry over to 1919,1920 because that 173K would have been taxed 70%, 1917-1918 only saving 10%, but need to use this the first year of net income (?)

o Can’t control/choose which years you can use the NOL based on tax rate or whatever, but we have loosened the annual snapshot a bit

o §172 Does not address/help with difference in tax rates across years– Hypo #2 under §172

o 1988 -- $10K loss fully deductible (you had positive income and used the full deduction, no unused losses), tax rate of 10%

o 1990 $10K income from that prior activity, tax rate of 70%o TP is upset because his deduction only saved 10%, whereas TP income

was taxed at 70%o Tax savings less than tax due (1K of savings v. 7K in tax) o This is not a case of NOL carryover, but the argument is that it is

unconstitutional because of the lack of transactional approach (rejected by Sanford)

o Obviously this could go both ways, in instances where the taxes go way down the TP tax savings is greater than taxes due

o Still seems unfair in cases of no net losses when rates change between years of capital recovery, but nothing you can do about this effect

– Why this called Tax Benefit?o Yes, you took a deduction but it do not actually save you taxes, you did

not get to use a tax benefit – Reverse question:

o What if you fully deducted something and then it turns out it you weren’t entitled to?

Deduct business expense for office supplies but then you use it personally, should you now report income?

4. Annuities – Pay a certain amount of money and at some time they start paying you an amount for

the rest of your life (or sometimes over a term)– Hypo: You pay $800 premium corp pays you $100/year for 15 years

Page 20: Tax - Harvard Law · Web viewAmount Today/Interest Rate/# yrs into the future/total amount in x yrs 2nd way to think of it: ... Alternative: don’t deduct the repayment in year 2,

o What is the problem? How are we going to tax the individual payments? When are we

going to treat as income and when as recovery of basis? Three options

Not taxed until recover all of your basis (TP wants this, time value of money)

Basis last and income first (tax the first 7 years) Something in between

– Pre ‘34, recover your basis firsto This might be bad for retirement purposes (you are taxed more in later

years) o Government doesn’t like it because you are deferring

– ’34 - ‘54 o 3% rule, taxable amount each year equals 3% of the premium (Egtvedt

case) Egtvedt complains because

His premium is $100K Annual payment is $4,884 3,000K is taxable (1,884 is recovery of basis, not taxable) This rule does not allow him to recover his basis anytime

within his lifetime But court rules that 3% rule is ok saying that 3% is the avg. return

on annuity premium Does constitution require that you can’t tax until TP has recovered

basis? absolutely not. In line with Sanford. o The bigger your premium the bigger your taxable portion, seems a little

counterintuitive (bigger investment more should be tax-free)– Post-‘84

o §72 Outlines this ratio:

premium/expected return = amount excluded/annual receipt o Solving for amount excluded

Hypo, $50 premium and expecting $100 total, $10/year for 10 years

$50/$100 = x/$10 x = $5 will be tax-free

Here the bigger your premium, the more that is excluded from every annual payment (exact opposite intuition of 3% rule)

– Term annuity set # of years, very easy to calculate expected return– Annuity Hypo:

o Pay $1735 for K which will pay $1000/yr for 2 yrso $1735/$2000 = x/$1000

amount excluded = $867.50 – Now same Hypo, but instead of calling it an Annuity called it a loan/self-amortizing

mortgage (this looks exactly like a self-amortizing mortgage)o How would we tax it now? (assume interest rate is 10%)

Page 21: Tax - Harvard Law · Web viewAmount Today/Interest Rate/# yrs into the future/total amount in x yrs 2nd way to think of it: ... Alternative: don’t deduct the repayment in year 2,

Year 1 Year 2Principal (non-taxable return of basis)

$826.50 $908.50

Interest (taxable income) $173.50 $91.50Total $1000 $1000

In contrast, under Annuity the picture looks the same for both years 1 and 2: $867 (tax free return of basis)$132 (taxable income)

– If you re-characterize as interest more taxable up front, in annuity you are taxed equally

– TP prefers to characterize as annuity rather than loan because of TIME VALUE OF MONEY, the benefits of deferred taxes

– Why this benefit of annuities?o We want to create incentives for people to buy annuities (we like people

saving for retiremento Steady, predictable picture of post-tax income for the TP

– Life annuities:o Payments for “the rest of your life”o Can’t definitively determine expected return (don’t know how long people

are going to live) Use actuarial table used by the insurance co. to determine your

premium The code has their own actuary table

– Hypo, 75 yrs old, premium is 50K, pay $5k/year for rest of your lifeo Life expectancy is 12.5 o Expected return = $5k x 12.5 = 62,500o 50K/62,500 = x/5,000 o x= 4,000 is amount excluded from each paymento Under the old 3% rule

$1500 taxable, $3500 excluded §72 is working better for this TP

o Pre-86 this excludable amount is extended over all the payments no matter how long you lived, TP is happy about this, a windfall for the long-lived (if you don’t make it to 12.5, you get an anti-windfall)

New provision (72 (b)(2), (3))says that after you live past your expected lifespan, nothing is excluded, you have recovered your basis. Alternately, if you don’t make it 12.5, in your last return, if you have not fully recovered your basis, you can take a deduction on whatever is left of the basis, a way to compensate you

After-tax income goes down for TP after 12.5 years The new rule is more accurate on a taxpayer by taxpayer basis (just

like the shift away from the 3% rule)– Hypo: Premium 30K, starts at age 65 pays 5000 per year

Page 22: Tax - Harvard Law · Web viewAmount Today/Interest Rate/# yrs into the future/total amount in x yrs 2nd way to think of it: ... Alternative: don’t deduct the repayment in year 2,

o 30K/(5K * 20) = x/5,000o x = 1,500 excludable

– What if employer pays (some) premium on the annuity for you?o Section (f), general rule if you are not taxed on it originally, you are taxed

on it when it is paid out (no of it is your investment/basis)– 2003 buy annuity $100,000 starts in 2023, but in 2004 the K is worth $110,000, the

company had invested it. You withdraw 10K, and wonder whether it should be taxable. You can argue that it is not taxable because the annuity does not start until 2023 and you are just withdrawing 10K from your 100K basis (return of part of my premium), so not taxable.

o Bad for gov’t, because people evaded taxes by sheltering their investments in annuities

o Congress responds with 72 (e), to prevent people from manipulating annuities to invest and get money, looks like situation above is usually taxable

5. Life Insurance– §7702, Life insurance defined, 2 basic kinds

o Term – die between certain years you collect, if you don’t you get nothing and lose your premiums

o Whole Life – keep paying premiums until you die, and then a payout– Term policy is essentially a bet, whole life is a combo of a bet + a savings account – Term: if die in 1 yr $

o How do life insurance figure out how much to charge you v. pay outo See how likely it is for someone of your age/health will die in a yearo Every year you get older your premium gets higher, this is the reason for

whole life – Whole life

o Level premium for your whole life (although starts out higher) Early on Life insurance is collecting more money then needed for

payouts from that age group, they invest this extra amount, so in later years, when the annual payments are not high enough for the pay outs for your age group they draw upon the investment pool they created. Most of the money paid out at the end is based on investment money rather than a bet with your peers that you will die first.

– Tax rules: premiums are not deductible but payments excluded (§101) o Why?

Death is the ultimate Personal Injuryo Why not?

Investment aspect, should be taxed just life interest especially in terms of

Analogous to lost wages (you are trying to replicate your salary for your family in case you die)

o Other possibilities Deduct premium, but taxing the receipts

Page 23: Tax - Harvard Law · Web viewAmount Today/Interest Rate/# yrs into the future/total amount in x yrs 2nd way to think of it: ... Alternative: don’t deduct the repayment in year 2,

No deduction and tax the receipt Impute a return for the savings pieces (income v. return of basis) Wait till payout and then tax the gain

– 2 (A) don’t need to know, not going to cover transfers of life insurance K’s between corporations

– Hypoo TP get policyo $5K yr premiumo Dies, spouse benefit

Tax effect= no tax on payout– TP transfer policy in return to business partner for 15K, pays premium for another

year, then TP dies. o Tax consequences

TP is taxed on 15K of selling the policy (payout has to be reason of death)

Typically, transferee can only exclude up to 15K (consideration for the transfer) + premiums he paid over the year

But in this case it is tax-free because of the exception under 2 (B) for business partners and other legitimate business person

– A sells to co-shareholders in a small business for 15K. Buyer continues to pay premium for 3 years then A dies.

o Doesn’t fit in the exception, because there is no exception for co-shareholders in (2) (B) as opposed to corporations

o Can excluded 15K + premiums paid – 101 (g)

o Exemptions for selling life insurance to certain type of licensed for those you really need the cash (terminally ill patients, chronically ill)

– Your employer pays for your life insurance contracto §79 says that the premiums they pay will be income except for policies

up to $50,000, why? To encourage life insurance avoid social welfare costs but want to avoid abuse.

– Hypo, Dad has life insurance K which will pay 100K on death and daughter has securities worth $45K, and they exchange assets:

o Tax consequences Looks like a transfer for consideration Dad pays tax on 45K Daughter pays tax on payout except 45K + premiums she paid,

no exception in 101 (a) (2) (B) Daughter’s lawyer could argue that it was an exchange of mutual

gifts, there was no quid-pro-quo, if mutual gift not a transfer for consideration (no taxes on gifts)

C. Realization1. The Requirement of Realization

– Macombero Macomber had shares of stock, and corp. paid out a stock dividend (extra

shares) and IRS wanted to tax at the market value, should it be taxable?

Page 24: Tax - Harvard Law · Web viewAmount Today/Interest Rate/# yrs into the future/total amount in x yrs 2nd way to think of it: ... Alternative: don’t deduct the repayment in year 2,

What is a stock dividend? Increase all shares of stockholders, pro rata

o Cash dividend would clearly be taxableo Situation Simplified:

Yr 1, set up a company and each pay $1 for each share, 50 shares, 5 owners ($1/share) (each shareholder has $10 total)

Yr 2, the company makes $1000, and still 5 shareholders at 10 shares each, there is now $1000 in the company ($20/share) (each shareholder has $200 total)

In year 3 there is a payout of 50% stock dividend, now each owner has 15 shares ($13.3/share) (each shareholder still has $200 total)

o Why not bound by Eisner, which held that stock dividend not held as income? Gov’t says

Stock dividend “based on” pre-1913 earnings Congress has now passed a statute that could be re-interpreted to

include stock-dividends Prior Case did not involve the constitutionality of taxing stock

dividendso Why does the Court not just cite Eisner?

Clarify realization Explore realization Some ambiguity because of the new statute

o Gov’t argues The corporation earned profits and thus the shareholder be taxed

on any payouts of the underlying profits Also argue that clearly would be taxed if you received a cash

dividend and then turned it out around to buy more stocks o Court says

They must define “income,” “gain derived from capital, labor or both.”

The focus on the word derived there has to be some severability to count as income

All Macomber has is still stock, she has not derived anything separate

Gov’t says that a gain is enough, shouldn’t read severability into the Constitution

o Macomber actually gets rich in Yr 2, the stock dividend did not make her wealthier she just has more pieces of paper.

o Bottom line: NOT TAXABLE Court also points out administrative difficulties in determining

when she exactly became rich She has a liquidity problem, she will have to sell some of her

shares to pay her taxes Corporation and stockholder are not the same legal person,

shouldn’t mush them together, goes against what the code has been setting up

Page 25: Tax - Harvard Law · Web viewAmount Today/Interest Rate/# yrs into the future/total amount in x yrs 2nd way to think of it: ... Alternative: don’t deduct the repayment in year 2,

o Dissent Says there are 2 ways to do stock dividend

Classic (like above) 2 step

o pay out cash dividendo shareholder can buy more stock at below market

price Makes no sense that one way would be taxable and the other

equivalent way is noto 4 options when to get taxed, question of the case is where to get taxed

Cash received – sale or cash dividend (taxable) Choice of cash or stock (taxable) Stock dividend (Court draws line here, TP’s share of the pie has

not become bigger) Corp earns $ (yr 2) (Dissent would draw line here or even beyond,

pie has gotten bigger for shareholder)o Mrs. Macomber basis

Stays $10 between yr. 1 and yr. 2, she has invested no moreo Case is constantly cited but almost never followed (!)

– Mere appreciation of your asset is not enough, not even a stock dividend is enough– Implications

o A works 40 hrs a week and gets paid $800/weeko B is unemployed and his stocks go up $800 that weeko A is taxed but B is not!

If you are earning money through investments you are able to defer taxes a great deal

– One thing to bear in mind: What is the constitutional requirement of realization?

– Realization Requirement – idea of severability, not just that your thing has grown, you have severed the income from something. Not only that you become wealthier.

– Bruun case 1915 lease begins 1929 lessee put new building up 1933 lessee default, property returned to TP 1979 – when lease was to end

o Tax problem, did TP realize income when he received the land back with the new building on it?

o IRS wants TP to pay the difference in value of the new building and old building as soon as he gets the land back

o Sup. Ct. says agrees that it income when TP receives the land o TP argues under Macomber, that there has been no severability, there

should be no tax until TP sells the land o Court draws the distinction, that under Macomber you just got new pieces

of paper, getting land with a new building on it is different

Page 26: Tax - Harvard Law · Web viewAmount Today/Interest Rate/# yrs into the future/total amount in x yrs 2nd way to think of it: ... Alternative: don’t deduct the repayment in year 2,

o Court also says the Macomber is just about stock dividends, but this is not really the case, the case was about all income

o One argument against this is that property overall has gone down because of the depression, but the Court focuses the fact that the new building is a new asset

o Liquidity issue, what is the new tax going to paid with? – Congress steps in with §109 which says no tax on return from lessee, must wait for

selling or something more – Congress can always tax less than they are constitutionally allowed to, but why is

Congress doing it in this case? – §109 and §1019, says, however, that when you get property tax-free from the lessee

you get no new basis, you don’t get anything more just because you’ve gotten your property back

o Of course if you build your own building you get additional basis – §109 also accounts for the situation where you exchange improvements in return for a

decrease in rent, to the extent that it is going on, it is not option but that is required for rent, that part is taxed

o Obviously huge and blurry factual line here, though landlord’s probably want in the contract

– Why is it rule of realization a good rule?o Fluctuation in value of assetso Administrabilty o Liquidity issue, is there really cash on hand to pay for the new tax?

(sometimes we care about it more than others, we’re no always consistent) 2. Value of Deferral

– Present Value: 2 different ways of saying same thing:o Today I have $98

If interest raters are 2.04% then in 1 yr, I’ll have $100 ($98 + (.0204 x $98) = $100

Thus $98 is the present value today of $100 in 1 yr at 2.04%o Pieces:

Amount Today/Interest Rate/# yrs into the future/total amount in x yrs

– 2nd way to think of it:o I know I will need $100 in 1 yr to pay a bill, how much to have to set aside

in a savings account today if rates are 2.04%? 98!– One way to think about deferral is as a loan

o $100 in wages, tax rate of 50% and special rule allowing a tax deferral for 1 yr

o This is like a $50 loan for 1 year for 0% to do whatever it is you want, it’s really there money

– Handout problems– Traditional Picture, 10% interest rate and 50% tax rate:

Yr. 1 Yr. 2Pre-tax receipts 100 55

Page 27: Tax - Harvard Law · Web viewAmount Today/Interest Rate/# yrs into the future/total amount in x yrs 2nd way to think of it: ... Alternative: don’t deduct the repayment in year 2,

Tax 50 2.50After-tax amount 50 52.50

– Exemption for Savings AccountYr. 1 Yr. 2

Pre-tax receipts 100 55Tax 50 0After-tax amount 50 55

– Deferral on the income tax for yr 1Yr. 1 Yr. 2

Pre-tax receipts 100 110Tax 0 55After-tax amount 0 55

– This looks like the 2nd situation, the two regimes result in the same bottom lime result for TP, deferring income is the same thing is taxing you when you earned it and did not tax the interest income

– We highlight this is to show that deferrals are very valuable, you can have your savings account tax free (just like the interest-free loan)

– This does not only work in a 2 year system – In traditional situation, the interest rate BT is 10% and AT is 5%– For situation II and III, AT and BT are exactly the same at 10%, just another way at

looking at it– Hypo invest at 10%. 100K tax rate 50% goal is to live off of interest. How much is

that look at the 2 regimes:o Tax the bonus immediately, exempt investment

Yr 1, 100K 50K Yr 2, 55K 5K of income to live on

o Defer tax on bonus, tax investment Yr 1, 100K Yr 2, 110K Take out 10K from the account and wind up with 5K to live on

o Looking at the same picture for both regimes – Our system is looking more to events rather than actual increases in value to tax– As TP you don’t want these events take place or it make it seem like the even didn’t

happen – But what are the negatives of having a realization requirement? How does it make

things harder? 3. Discount Obligations

a) OID – Original Issue Discount – In yr 10 it will pay x, in yr. 1 you pay something less than x because you could have

earned interest somewhere else in the intervening yr’s, the question of how much less is a market question

– $100 -- $133, $33 represents interest income to the TP, the question is how to go about taxing the $33

Page 28: Tax - Harvard Law · Web viewAmount Today/Interest Rate/# yrs into the future/total amount in x yrs 2nd way to think of it: ... Alternative: don’t deduct the repayment in year 2,

– The primary question is when will you tax the income, TP wants to defer it as far as possible (10th yr)

– Before 1969, the regime we imposed on OID bonds is that tax until the bond matured (TP’s dream come true)

– 1969-1984Yr. 1 Yr. 3Buy for $100 Get $133 from IBM

69-84, tax is ratable, TP gets taxed $11 per year, this is to counteract deferralOne essential difference between stocks and bonds is that with a bond you know how you will get, stocks can fluctuate, thus we are less prone to give bonds deferrals, it is more like annual interest payments

– Post 1984o Tax based on compound interest/economic accrual; 10% in this caseo Yr. 1 Give $100, they owe you $10 o Yr. 2 $110, they owe you $11o Yr. 3 $121, they owe you $12o Differs from 69-84 rule which just says you earn $11/yr, which is not

economically accurate o You are paying tax even though you receive no cash from IBM (also true

in 69-84) o There is a liquidity problem but for some reason here we are not overly

concernedo IBM allowed to deduct the amount of interest without paying out cash

– Why do we care about increased economic precision?o Seems like 69-84 regime gov’t better off because get to tax TP more BUT

have to consider that IBM is simultaneously taking a deduction, so it seems like it sort of nets anyways

o But actually the only people who would buy OID bonds were untaxable nonprofits so gov’t was getting no extra income but was getting the overstated deduction from IBM, hence the decision to move away from ratable

b) Market Discount Bonds– Very similar to OID bond but tax system treats them differently, need to keep in mind why…– You’re a buying a bond at a price that is going to be discounted because of a lower fluctuation in prevailing interest rates or company status– $1000 bond which pays 10% annually = $100/year which is the only income piece in this scenario– Say now that the market interest rates rises to 12% and you are bond of course is still paying 10%

o Now no one will pay $1000 for your bond because they 12% for a different bond

Page 29: Tax - Harvard Law · Web viewAmount Today/Interest Rate/# yrs into the future/total amount in x yrs 2nd way to think of it: ... Alternative: don’t deduct the repayment in year 2,

o For instance, they will pay $870, for the $1000, in effect this exactly like OID bond, he gets $1000 at the end $10% + $130 discount = 12%

o He gets $100/yr + $130 (discount) in income, which functions just like interest income

– How is this taxed? §1276-§1278o TP is getting $100/yr which is obviously taxed as income o Also if TP holds the income till the end, he will be taxed on $130 when he

gets the $1000 which is the discount piece, functions just like interest but he doesn’t get taxed till the end! So market discount bonds are getting favorable treatment over OID bonds.

Pre-’84 taxed at capital gains rates Post ’84 taxed at ordinary income rates

o If TP does not keep the bond till the end Rule

Basic: the $130 is basically interest and is being earned over time, if he sells in the middle he is not earning interest at that time so we don’t want to tax him on the full $130 if he doesn’t hold on to the bond till then end

o We tax TP on his accrued market discount up to the amount of gain (how much of the discount interest he has gained)

Definition:o x/total market discount = # days he held/total # of

days from the day he purchased till the end of the life of the bond (note: not the same as the total life of the bond)

After ‘84 Rule:1. Hold MDB till end, then full MD taxed as income2. Sell MDB Before end then AMD is taxed as OI to the extent there is gain on the saleQ. How figure out AMD?

a) ratable method AMD/TMD = # days held/total # of days from date of purchase to end of a bondb) Yield to Maturity/accrual method (treat as compounded interest rather than simple interest) * TP prefers because it will produce smaller AMD earlier

Hypo: IBM issues bond for $1000, 10% annual interest payments, $1000 returned at end of 11 years, Joe buys bond for $1000, at end of yr. 1 sells for 870 (annual + 130 =12%) to Harry. Harry holds till end of yr. 6 then sells to Su for $900.

Joe: – Taxed $100 of interest income

Page 30: Tax - Harvard Law · Web viewAmount Today/Interest Rate/# yrs into the future/total amount in x yrs 2nd way to think of it: ... Alternative: don’t deduct the repayment in year 2,

– Income realization of investment = (Basis = $1,000; AR= $870) = $130 loss on sale of the bond when he sells to Harry

Harry: – $100/yr. of interest income (note: do not include $130 of market discount on annual

basis even though it is exactly like interest income distinguished from OID bonds),– AMD/130 = 5/10 (yr’s harry could have held it), AMD = $65, this amount that is

taxed when he sells to Sue– Income realization of invest = Basis = $870, AR = $900 = $30 gain – BUT Under the rule he only reports $30 of taxable income (!?) (in addition to the

$100)

What happens on sale of bond? – Trigger tax on the interest– Trigger of the recognition the gain/loss on the asset– The rule combines them, says calculate the AMD and then figure out how much the

gain is and only tax the AMD to the extent there is gain

Alternate vision for Harry:1) Calculate AMD and tax you on it -- $65 OID (but he has not actually received it

yet)– we don’t want to double tax him, so we need to find a basis– $65 + $870 = new basis of $935, he sells it for $900, so he had a loss for

$35 – Net picture is $30 of gain (just like above)

Same facts as above but Harry sells for $800– Code:

o Tax him $100 end of very yearo AMD is still $65 o AR (800) – B (870) = -70o So, can’t tax any of the $65 AMD, Harry will simply take a $70

loss on his returnAlternate Vision

– Calculate & tax AMD = $65– Add to basis: $935– Tax the sale – AR (800) – B ($935) = - $135– So on the return $65 of OI and $135, which is a loss of $70, same as

above!

Same facts as above but Harry sells for $950– Code:

o Tax him $100 end of year for interest

Page 31: Tax - Harvard Law · Web viewAmount Today/Interest Rate/# yrs into the future/total amount in x yrs 2nd way to think of it: ... Alternative: don’t deduct the repayment in year 2,

o AMD is still $65o AR (950) – B (870) = $80o He is taxed on $65 as OI and $15 as Capital Gaino He is reporting $80, but breaks out the $65 as OID under §1276,

$15 is probably categorized as capital gain– Alternative Vision

o Calculate + tax $65o Add to basis; now $935o Tax sale $950 - $935 = $15o So again reporting $80 total, same as above!

– Why treat market discount different than OID? OID we tax your interest every year, with market discount you don’t.

o Aside: you have the option of being taxed on the interest annually (in case you want to make certain deductions related to the bond)

– IBM has no clue that their bond was sold at a discount, so the TP would have to calculate AMD on their own (IBM won’t send them a statement with AMD taken into account), don’t want to force TP to make this complicated calculation?! (Really?)

4. Intro to Capital Gains– If you own property and sell it for a gain or less it is taxed in special way (usually

lower) if you meet the requirement– Out of realization, the tax is looking for events to tax you, how do you create

incentives for the event? We lower the capital gains tax rate.

Pg 6. of the handout

Yr. 1 Yr. 2 Profit Rate of profitPre –Tax 100 110 10 10%Tax 0 5After Tax 100 105 5 5%

Tax rate 50% Investment Rate 10%

Deferral: Acceleration of Deduction for Basis, can deduct basis before you sell the asset

Yr. 1 Yr. 2 Profit Rate of profitPre –Tax 100 110 10 10%Tax (savings) (50) 55 (have

already deducted your basis)

After Tax 50 55 5 10%

Page 32: Tax - Harvard Law · Web viewAmount Today/Interest Rate/# yrs into the future/total amount in x yrs 2nd way to think of it: ... Alternative: don’t deduct the repayment in year 2,

Capital Gains Rate w/deferral:

Yr. 1 Yr. 2 Profit Rate of profitPre –Tax 100 110 10 10Tax 0 2After Tax 100 8 8 8%

20% Capital Gains tax

Capital Gains Rate w/Deferral

Yr. 1 Yr. 2 Profit Rate of profitPre –Tax 100 110 10 10%Tax (50) 22After Tax 50 88 38 76%

The effect of combining, you have a higher AT profit then BT because of the rate differential!

D. The Factor of Motivation1. Windfalls

– Find $1MM on the street, required to pay tax o Seems like you should because you have cash on hand

– Find Oil under your house?

o Seems sensible not to tax it because it doesn’t seem like it has been realized

– Buy a piano and find $ in pianoo Much more like #1, found money, is taxable (Cesarini)

– Antique desk turns out to be very valuable? o No realization, nothing new sounds a lot like Macomber.

– Antique Desk, Abe Lincoln’s pen turns up inside. o Tough call, close to Cesarini but no cash in hand just something valuable

in hand – All of these answers are along a spectrum (cash on one end, discovering your painting

is Monet on the other end) 2. Gifts

– $100,000 from your aunt as a gift. Is it taxable? There is no income tax under §102. (but note there is a gift tax (which is on the giver), which we will not cover)

– Why not the $1MM because there was no donative intent – Glenshaw Glass – all accretions to wealth are supposed to be income, which seems

to imply that there should be a tax , seems to be driven by the personal nature of gifts

– Compare:A gets 10K gift – no tax

Page 33: Tax - Harvard Law · Web viewAmount Today/Interest Rate/# yrs into the future/total amount in x yrs 2nd way to think of it: ... Alternative: don’t deduct the repayment in year 2,

B gets 10K for factory work tax wages – Why?

o Money to A has already been taxed (though we do this in other places)o Proof exchange/adminstrability difficulto Protection of family transfers

We could map out 3 different ways to tax gifts1) Tax both donor & donee – no deduction/inclusion in income2) Tax donee only – donor takes deduction/donee includes income3) Tax donor only – no deduction/not included in donee’s income (this is today’s

system)

– Rates? – Rather tax the donor than the donee if you want more money

– Administration?– Easier to find out info on donor than donee, because the $ is tracked when

it comes in initially

– For gift tax, need to report gifts of 10K and greater

– Give my brother a gift of interest income on a savings account, what is the tax consequence?

– It is included income under 102 (b)(2)

Irwin v. Gavit– Π Received gift of interest income in trust for 15 yrs – Issue: excluded gift or taxed gift– What does he try to distinguish his gift from?

o Traditional contingent interest: A to B for 15 yrs remainder to C – Π is saying that he gets cash, he does have not interest in the trust, he will receive a

specific amount of cash gift over time, not income – Court decides it is a gift of income and will be taxed

Ex. 1A dies, leaves trust w/corpus $100K to TP Tax Results: no tax on 100K but subsequent 10K/year taxed

Ex 2.A dies, leaves trust w/corpus of $100K income to B for 10 years remainder to C Tax Results: B taxed on 10K/yr, C 100K free, tax after

If we change the rule for Ex. 2 so that B is not taxed, we are giving too much of gift exclusion because the size of his gift is 100K – we can let both B and C take full deductions, people would just always split up their gifts like Ex. 2

Page 34: Tax - Harvard Law · Web viewAmount Today/Interest Rate/# yrs into the future/total amount in x yrs 2nd way to think of it: ... Alternative: don’t deduct the repayment in year 2,

However is there a more fair way to do it without giving an exclusion of 100K? Maybe we can keep the exclusion at the right number and make sure it is more equitable

Hyp#1 – I give my brother $10,000 to b paid in 5 annual installments, seems like this would just be gift of cash

Hypo #2 – I give my brother the income from savings acct $2k/yr for five years will be taxed, it is income, the $2k/yr has not been taxed yet

Hypo: I buy property for 100K, which generates 10K per year income I wish to give my brother the income for 5 yrs, then the corpus to my niece

2 ways:

1) I keep prop and make a gift of $10K (no tax) to bro each year. In five years I give my niece the property no tax on 100K

2) I divide the property today give my brother a 5 year interest (NPV = 40K (at 10% interest), my niece the remainder in 5 years (must be 60K), so allow exclusions for BOTH brother and niece along the following lines:

o Brother receives the right to receive 10K income over 5yr. period (40K) o Niece Right to receive 100 in 5 yrs (60K) o Brother happy/niece is unhappyo IRS unhappy because need to calculate interest rate, how long the interest

is for, in order to determine PV. Seems messier for them.

Aside: Compare to bond: if interest rates 10%– $10/yr for 5 yrs– 10K at the end of five years– What would you pay today? $100K (so this package of rights is worth $100K)– Brother and niece got the same package, so we know that on day 1 their package must

be worth 100K, and from there we can split it– Of course you can’t do this, specifically under §273

What are the possible args for unequal taxation between brother and niece?– If donor knows rules – he can adjust the gift so that at the end of the day brother and

niece walk away with the amounts you want them to have

Commissioner v. Early:– Husband dies W gets his stock when he dies which is under §1014 FMV (1014)

~ $2.28M– W gives gift to TP, carryover basis of $2.28M (1015a) (cash does not have basis)– TP is then sued by 48 heirs, to settle the lawsuit he gives the ownership of the stock

for a life interest in the state– TP wanted to count the value of his stock (plus lawyer’s fees) as his basis, so he

claimed that he “bought” a life interest with his stock, under §273 he could not have basis for a gift

Page 35: Tax - Harvard Law · Web viewAmount Today/Interest Rate/# yrs into the future/total amount in x yrs 2nd way to think of it: ... Alternative: don’t deduct the repayment in year 2,

– Court says that since the original status was as a gift recipient modifying the form through settlement, but it is still a gift

– All about how to interpret the facts – Clarification:

o A has an asset and sell 15 yr interest to B B gets to recover his investment against the income, since you know it will be fixed period of 15yrs and divide the investment and see what the ratable return is every year (this is how a purchase is treated v. gift)

– Taft v. Bowerso Situation

– 1916 purchase $1000– 1923 gift $2000 (FMV)– 1923 sale $5000

o Gift recipient then sells, argument is over what his basis is o To wants $2000 to be his basis

– Says that it is unconstitutional because they are taxing him on income he did not earn

o IRS says– That we must $1000K, so there is not some gain which escapes

taxation – Otherwise families would transfer stocks among themselves and

escape taxes completely– Gift recipients take gifts with knowledge

o Court rules that basis will be carried over for gifts, basis is $1000 and TP loses

§1015 Tax at GiftDonor No deduction, not taxed Tax at gift (income of

$1000)Donee Carry over basis FMV basis ($2000)

Hypo:1916 buy 10001923 gift 8001923 sale for…– 5K

o AR 5000Ko Basis 1000o Tax on $4000

– 700o AR 700o Basis 800o Loss of $100

– $925

Page 36: Tax - Harvard Law · Web viewAmount Today/Interest Rate/# yrs into the future/total amount in x yrs 2nd way to think of it: ... Alternative: don’t deduct the repayment in year 2,

o AR 925o Basis ($1000 or $800)o IRC treats this as neither a gain or a loss

– This all under §1015

– Obviously IRS and Congress do not want you to be able to pass along losses but they do want you to transfer gains.

– Why is this?

– If you have depreciated property it is not smart move to give it as a gift, because the loss deduction is lost. Better off selling the asset, taking the deduction, and giving him/her the money.

– What if instead of gift it was at death?

o Under 1014 you get FMV of asset at time of death, “step-up” basiso Want to hold on to appreciated assets, the gain will disappear on your

deatho You want to sell all of your depreciated assets, because the loss will

disappearo Real beneficiaries are the heirs of people who has a big pool of assets and

does not need to sell of their assets to live (the rich are favored again!)o Has impact on liquidity: people will hang on to their appreciated assets,

even if it is not the best investment move.o Every once in a while Congress will contemplate 1014, because they could

increase tax revenueo In 1976, they tried but it was repealed o What are some of the reasons for 1014?

Administrabilty of finding original basis rather than FMV – §1022

o After 2009, the basis is lower of the FMV and the decedent’s basis o What are we trying to achieve? o Trying to capture gaino But why not say flat carry-over basis? Because don’t want to be able to

pass on your loss.

– Hypothetical: A owns Greenacre (basis of $50, FMV of $400); B owns Boeing (basis $125, FMV $400). A exchanges Greenacre for Boeing.

o Is this a realization event?o Yes, it is as if A had exchanged Greenacre for cash (barter is generally

taxable); see § 1001.o A’s basis in plane is now 400, as is B’s in Greenacre

A BAR $400 AR $400

Page 37: Tax - Harvard Law · Web viewAmount Today/Interest Rate/# yrs into the future/total amount in x yrs 2nd way to think of it: ... Alternative: don’t deduct the repayment in year 2,

B 50 B 125Gain $350 Gain $275

New Basis $400 New Basis $400Farid-Es-Sultaneh

– Agreement for stock for dower rights– Issue is whether it was an exchange or gift?– And they are fighting over the basis.– If it is an exchange her new basis should be FMV at time of exchange– If it is a gift then it is carry over basis

Government’s issues: No adequate consideration Timing - agreement signed afterwards. Language - agreement says “gift.” “Exchange?” Gift tax definition - if you rely on the same definition (from the gift tax context),

this case would be a gift.

Sketchy ways to transfer the loss1) Hide2) Purchase3) Change the order of the transaction (give her cash gift and then she can purchase the

property back from owner)

Panel Prep:

a) Commissioner v. DubersteinFacts: Duberstein and Berman did a lot of business together. Duberstein gave Berman some client tips and one turned out to be so helpful, he decided to give him a Cadillac as a present, even though Duberstein resisted and Berman did not owe him anything. Berman deducted the present on its corporate income tax. Duberstein did not include value of Cadillac in gross income, deeming it a gift, Tax Commissioner asserted a deficiency.

Procedural History: Tax Court sided with the Commissioner saying it was remuneration for services. Court of Appeals reversed.

Facts: Stanton was comptroller of the Church and President of Trinity Operating company, subsidiary of Church to manage real estate holdings. Upon his resignation, the Board of trinity passed that “in appreciation of his services” he be given 20K provided he does not claim additional pension and retirement benefits (just cautious, he didn’t really have any other rights). Earlier, the board had passed similar resolution (without the word gratuity) for the company’s treasurer who was actually fired. The series of events led to

Page 38: Tax - Harvard Law · Web viewAmount Today/Interest Rate/# yrs into the future/total amount in x yrs 2nd way to think of it: ... Alternative: don’t deduct the repayment in year 2,

Stanton’s resignation. Secretary who quit at the same time, got the same deal. Tax Commissioner asserted a deficiency, taxpayer paid.

Procedural History: District Court overturned and said it was a gift, Court of Appeals reversed.

Government Args:1. New test for a gift should be: “Gifts are defined as transfers of property made for

personal as distinguished from business reasons.” Court REJECTS and says they will instead look to prior cases:

– Statute does not use term gift in the common-law sense, but more colloquial sense, even though a transfer without consideration in return is a common law gift, it might not be under this statute. (Old Colony)

– If there is a moral/legal duty or incentive of anticipated economic benefit, not a gift (Bogardus)

– In return for services already rendered also, not a gift (Robertson) – Must be detached and disinterested generosity (LoBue) out of affection,

respect, admiration, charity or like impulses.– Most crucial consideration is the transferor’s “intention,” though cannot be

donative intent. Also must be an objective determination and better to call it “motive.”

– Motive must be a factual determination, dominant reason why the transferor made the transfer.

2. Government relies on a pair of principles for its test (1. normally not a gift when given from employer to employee, 2. definitely not a gift when deducted as business expense by the corporation)

– Court says that whether it is a gift should be a matter of fact rather than law, weighing all the relevant instant factors

– Gov’t method, courts would be looking to complex state laws on employer/employee definitions and federal deduction law

– Of course, these facts can be persuasive but should not be matters of law3. Primary weight in this area must be given to the conclusions of the trier of fact on

a case-by-case basis. – If Congress wants specific factors that should be determinative they can

pass clarification – Means Appellate review of the determinations must be quite restricted,

including findings of the Tax Court4. Applying to Duberstein, upholds decision of Tax Court that it was not a gift.

Court of Appeals could not say that the lower courts factual determination was “clearly erroneous.” Applying to Stanton, plurality thinks that District Court did not make enough of factual inquiry into whether it was a gift.

– Duberstein is reversed and Stanton is vacated and remanded for further proceedings in the District Court.

Harlan– Concurs with Duberstein, would affirm Court of Appeals in Stanton

Page 39: Tax - Harvard Law · Web viewAmount Today/Interest Rate/# yrs into the future/total amount in x yrs 2nd way to think of it: ... Alternative: don’t deduct the repayment in year 2,

Whittaker – Concurs, but says whether a particular transfer is or gift is determination of mixed

law and fact

Douglas– Dissents, says both are gifts under Bogardus test

Black– Councurs in Duberstein– Dissents in Stanton, lower court’s finding that it was a gift was not clearly erroneous

Frankfurter– Concurs in Dubertsein– Dissents in Stanton – Says that while general formula is futile, it is possible to isolate factors which

militate against a gift in particular situations– In these two particular cases (employer to employee, payment in business relation)

presumption should be that they are of a business nature, and burden should be on beneficiary to prove otherwise.

– Too much unpredictability by the leeway majority gives to triers of fact. – Would uphold Ct. of Appeals in Stanton, because taxpayer failed to sustain the

burden of proof that it was not a gift, which is what he would apply. No findings needed by the trial judge.

b) Stanton v. U.S (district court case on remand)– Refers to testimony and resolution that the Board wanted it to be a gift– Quote from Ct. of Appeals in Bogardus that you can give gifts for past services if you

don’t legally owe the performer of the services anything, which is the case here– It was the result of the good will, esteem and kindliness of the minds of the Vestry of

Trinity Church

c) U.S. KaiserFacts: A union gave room rent and food vouchers to a striker (not a union member at the beginning of the strike). Jury verdict said this was a gift rather than striker’s pay.

BrennanAffirms lower court decision. Says there was a basis for jury to conclude that it was a gift looking at the Union’s constitution (to show that was not the result of legal or moral obligation) and that it was simply a charitable donation rather than for an anticipated economic benefit. Cannot say, as a matter of law, that because there was a sympathetic interest that prevents it from being a gift, this is just the case in charity as well. If jury used Court’s instructions, it is empowered to render the verdict that it did.

Frankfurter:

Page 40: Tax - Harvard Law · Web viewAmount Today/Interest Rate/# yrs into the future/total amount in x yrs 2nd way to think of it: ... Alternative: don’t deduct the repayment in year 2,

– Can’t conclude as a matter of law that all striker’s benefits are gifts, thought sure does look like one

– In this case jury got legitimate, fair instructions on definition of gift – Union had taken on the role of a charitable organization in these vents– Even sometimes an employer to employee may be gift (Turkey for Christmas) – Sufficient evidence to support that it was acting like a charitable organization – A different jury may have found differently, but this is inevitable result of Duberstein

Revenue Ruling– All cases like Kaiser will be counted as gifts– The fact that benefits are paid only to union members will not, in and of itself, be

considered determinative of whether benefits will be regarded as gifts or as gross income

Where are we today in this area? IRC § 1041 governs transfers of property between spouses or incident to divorce. General rule is that transfers between spouses are not taxable events; carry-over basis

applies. This treatment differs from the treatment of gifts because here there is no

“bizarre” loss rule. You can give your spouse your loss. Look at the regulations in this section for an example of the Service’s use of question

and answer format in regulations.

Gifts: Business Gifts See text pages 198-213, 218-223.

Commissioner v. Duberstein (1960) (page 170) (includes Stanton opinion) Case where TP received a Cadillac in a business relationship context. Government is arguing for a business/personal distinction - basically no such thing as

a business gift. Court rejects this bright-line rule.

Court’s approach to defining a gift is “facts and circumstances” looking to the “objective” intent of the donor. Gift does not mean a lack of consideration. Ring’s “tip as a gift” hypothetical

proves this because a tip is not a gift, but you don’t have to pay a tip to get food and service.

Court highlights “detached and disinterested generosity.”When a gambler wins, he may give a tip to the dealer. Is this a gift or income? Because there’s no obligation to make the payment, this is more like a tip and thus a gift. The 9th Circuit held this.

If one’s employer gives a fine vase upon one’s marriage, the government would tax this but a court probably wouldn’t tax this and would reward life’s good moments. The court seems willing to find gifts in commercial contexts.

The Code:

Page 41: Tax - Harvard Law · Web viewAmount Today/Interest Rate/# yrs into the future/total amount in x yrs 2nd way to think of it: ... Alternative: don’t deduct the repayment in year 2,

IRC § 274(b): allows employer to deduct a gift only up to $25. IRC § 102(c) [added in 1986]: basically says no employee gifts (sounds the almost

bright line the government put forth in Duberstein)

Historical Transition: We see the case law (e.g. Duberstein), then § 274(b), then § 102(c).

It seems that the Code is shifting towards the government’s bright line rule it argued in Duberstein.

There may still be ways to get small gifts from the employer under the de minimis fringe idea. You can see in the regulations that this may be allowed despite § 102’s “no employer gifts.”

A $200 wedding gift under 102 is income taxable, under Duberstein it is probably a gift

– 1.102 – 1(f)(2) If someone has a family business, are all transfers within the family a business exchange? One would have to demonstrate the transfer was a natural object of the employer’s bounty – that this was a person one would give a gift to naturally.

Suppose the employer makes a transfer to the spouse of a deceased employee.Estate of Sydney Carter v. Commissioner (2d Cir. 1971) (page 191) Employee died and the employer wanted to give the employee’s wife the profit bonus

the employee would have gotten had he survived. There was no obligation to make the payment.

Traces the case law in this area. It looks like the tax court has tended to say that these transfers are not gifts, while the district courts have tended to view them as gifts.

In this case, the Tax Court had held “no gift.” The Second Circuit overruled this determination and held that the transfer was a gift. How does the Second Circuit’s reversal fit with cases like Duberstein that give

great deference to the trier of fact? Second Circuit seems to suggest that the Tax Court is not following prior

cases in this area. Later, the government has to deal with the fact that the company took a deduction in

this case (while the employee spouse received it as a gift). See note on page 223.Applying the later statutory provisions to the case law: IRC § 102(c)

Would this have changed the analysis in Duberstein? No - Duberstein was not an employer/employee relationship.

Would this have changed the analysis in Carter? Possibly - although the spouse is not an “employee,” the payment to the spouse was based on how long the employer had been at the company, their compensation level, etc.

IRC § 274(b) Would this have changed the analysis in Carter? Company would not have been

allowed to deduct this payment because it exceeded $25.

United States v. Kaiser (1960) (page 182)

Page 42: Tax - Harvard Law · Web viewAmount Today/Interest Rate/# yrs into the future/total amount in x yrs 2nd way to think of it: ... Alternative: don’t deduct the repayment in year 2,

Jury held that strike benefits (room rent, food vouchers, etc) were not taxable. Important to this Court to defer to fact finder Important facts for this Court:

Kaiser was not a member of the union This makes it seem like less of an exchange and more like a gift.

Kaiser was certainly in need Lingering question is how close you have to be to the facts of Kaiser to not have it

taxed What if it was AA strike where member received 60K in strike benefits?

The large amount seems more than just minimally sustaining someone, a charity organization does not hand out 60K, this would seem like court more likely to consider this income

3. Prizes and Awards – Starbucks gives $1MM as its one millionth customer

o Under Pauline their no capital and no labor investment, so it would qualify as a gift

But what is the difference between the prize and finding money on the street which would count as income, the difference seems to be the presence of the 3rd party and a 3rd party motive

o Under the code today, §74, prizes and awards are presumed to be income unless meeting requirements of 74 (b) which are not met here in this case or in Pauline

– §74 (b)o Earned the award by virtue of Religious, charitable, artistic, science

efforts, etc.o No action required by youo Have to give it away to government or charity (comes in 1986)

You wouldn’t always get the full $1MM for a charitable deduction, so this really guarantees you would get the full benefit, it is always better to exclude it because you don’t caught in the trap of rules which limit the amount of deductions

– What if you win an award for best essay in tax award?o Seems like it won’t be excluded even though it is education, because you

actually submitted the essayo You have a better argument, if your professor submitted it for you

– What if you win research stipend to travel in Asia?o Seems like meets education requirement, but how would you give this

away to charity?

a) Hornung– Winner of best game in NFL championship and gets a Cadillac, says it should be a

excludable as a gift under 102 or a excludable prize under 74 (before the requirement that you give it away)

– Court is not persuaded that it is artistic, educational or scientific so it is not a prize

Page 43: Tax - Harvard Law · Web viewAmount Today/Interest Rate/# yrs into the future/total amount in x yrs 2nd way to think of it: ... Alternative: don’t deduct the repayment in year 2,

o Thus it is income as a prize under 74 (a)– Court also says it is not a gift, because there is no detached and disinterested

generosity from the Sports magazine b) Willis

– Baseball player who gets hickock belt for being the best player of the season– Argues that he is distinguishable from Hornung, saying that he played over the whole

season and that it is trophy which is different than receiving a car – he would not go out and buy a car

– The court is not as flippant but no more persuaded that there is the scientific/artistic activity going on that is required to meet the exception

– There also seems to be a liquidity problem, he might have to sell it to pay the taxes but notable that Willis had actually used some of the diamonds from the belt for a ring for his wife

– Does the IRS go after Olympic Medals? They could be under §74 analysis, but that the public sentiment would be much against it, so they don’t enforce it

Awards from your employer:

– Could you just say it was a gift under 102?o No, because there are no gifts in the employer-employee setting

– Turn §74 (c) which creates an exception for EE achievement awards can be excluded than income, we have to turn to 274(j) to find out what is a proper EE achievement award

o Under 274 (j) the requirements are No more than $400 Length of service/safety Meaningful presentation Cannot be disguised compensation Has to be tangible personal property, cannot be cash

– Keep in mind that 74 (c) is about employee treatment (excludability), 274 is about treatment of employer (deductibility)

– A $400 gold watch taxable for length of service?o §74

Seems to meet #1, #2, and #5 Need more facts to determine #3 and #4

o §274 (j) employer should be able to deduct the entire thing

– How about $400 cash for length of serviceo §74 (c)

Completely fails because it is casho Employer can claim a standard deduction for business compensation

– How about $400 gold watch for best attendance?o Seems kind of hard to fit into length of service/safety, seems stupid for

employer to phrase it this way – What if the gold watch is sent to you via interoffice mail?

o Seems not be a meaningful presentation

Page 44: Tax - Harvard Law · Web viewAmount Today/Interest Rate/# yrs into the future/total amount in x yrs 2nd way to think of it: ... Alternative: don’t deduct the repayment in year 2,

– What about $500 watch meeting other requirements?o Under 74 (c)

Can exclude up to $400 $100 taxed as income

o Under 274 (j) Employer can only deduct $400 The other $100 is not deductible not even as a business expense

deduction o So you go over both sides loses

4. Scholarships $ Awards §117 Qualified Scholarships can be excluded from income

o In order to be excludable scholarship must only cover fees, books, tuition (room and board are not considered to be qualified expenses)

o Also have to be a candidate for a degree o What is the policy behind this rule?

We have strong social policy of viewing education as valuable

How you get the money: Scholarship Work Borrow

State University (state subsidized)

Private School (alumni donation

subsidies)How money used to pay tuition is taxed: no tax taxed taxed no tax no tax

What explains this? o Seems easy to explain for no tax but what is the justification for being

taxed on working and borrowing to pay for education? The education credits over the last couple of years have focused on

giving deductions for those who work/borrow to pay for school Hypo

o Want to invest in yourself: Invest in education the return is high wages, do not get any

deduction for the work/investment you put into your education against these high wages

Invest in Real Estate, here we do deduct your basis from amount realized

Point is treat these investments differently A messy line is when we talk about Graduate TA, how we characterizing the

benefit of qualified tuition reduction (benefit salary) §127

o covers employer education benefito If it meets the requirement it is a tax free benefito Question is why do we need §127 when we have §117? In §117, the

employer needs to be an educational institution, §127 is the broader provision of educational benefit given by all other employers

5. Social Welfare Payments Why are welfare payments excluded?

Page 45: Tax - Harvard Law · Web viewAmount Today/Interest Rate/# yrs into the future/total amount in x yrs 2nd way to think of it: ... Alternative: don’t deduct the repayment in year 2,

o Wouldn’t they not have to pay taxes if their income was low enough anyways?

o Double taxation? Maybe/o Policy concept that it is for a basic support, it is not a quid-pro-quo.

Under §85, unemployment wages are taxableo Closer to replacing wages

Under §86, social security is partially taxed, the scheme looks like this;

Taxpayer Income: <$25,000 $25,000 - $35,000 >$35,000Tax Treatment for

SS:No include 50% of social

security incomeinclude 85% of social

security income

Why not tax it all?o Idea that both employers and employees put their own $$ into social

security system, which they don’t get a deduction for, so this looks a little like a return of investment

the graduated part of this is pure social policy Income

o gifto prizeo scholarships

If you fail to successfully argue that it is a prize can you argue it is a gift? Seems like congress intended to make distinctive concepts, gifts is not a background category, if it fails prize/scholarship it is income, cannot not fall back into large a gift category

The whole idea of motivation is much more carefully defined than it was, prizes/scholarships are considered to be in distinct category from gifts

E. Competing Claims and Offsetting Liabilities1. Cancellation of Indebtness

Borrow $20k from Uncle – no tax Borrow $20K from president of corporation – no tax assuming it is legitimate Buy stock for $100, rises to $1,000,000 you borrow against the stock and spend it

on whatever you want and then you dieo In the year of borrowing, there is no tax burden on youo When you die

Tax-wise, the basis of the stock is step-up basis under §1014o So consequence you have been able to live off the $1,000,000 without

paying taxes, your heirs can sell the stock to pay off the debt, but they are not taxed on that either

o So under this situation no one is taxed and you get to live off $1,000,000 of your appreciation

Law School lends ILS coffee shop $50K, they go out of business and Harvard cancels the debt, what is the tax effect?

o The baseline rule is that 50K is taxable now that it has been canceled, why is that?

Page 46: Tax - Harvard Law · Web viewAmount Today/Interest Rate/# yrs into the future/total amount in x yrs 2nd way to think of it: ... Alternative: don’t deduct the repayment in year 2,

o § 61 (a) (12) tell us that a discharge of indebtness is specifically considered income

In Kirby Lumber, the company issued bonds for $12MM and then repurchased them $11MM, had $12MM come into the company as not taxable event, and then they repurchased it for $11MM, a net gain of $1MM

o The court’s reasoning seems to be that the company netted $1MM *just deferred because of the loan)

o The idea that the company has freed up $1MM in assets, they have extra cash on hand from the cancellation of debt, this argument stops being a prominent one

Parents forgive $25K law school loan, what is the tax consequence?o IRS could of course argue you have 25K in C.O.D. incomeo TP argues it is a gift

Challenge dental bill, get him to reduce it from $100 to $40o IRS could say it is $60 C.O.D. incomeo TP says it is a bargaining situation

Pay $100 dental bill by giving a painting you bought for $40, now FMV $100o No element of debt cancellation but just not involving cash, the tax on the

patient would be a capital gain on the painting ($100AR - $40B =$60 gain)

you go to the dentist and he bills you for $100. But the dentist is an art lover, so you give him a painting that you bought for $40, but that has a FMV of $50. The dentist nonetheless likes the painting and so he accepts it in payment of your bill

How are you taxed? If painting is worth less than $100, part would be C.O.D. and part would be

untaxed appreciation. In this case, $50 would be C.O.D. and $10 untaxed appreciation.

You will probably argue with the Service over whether or not this was a negotiation of the bill. Trying to determine the true characterization of the situation

you have a $100 bill from your dentist, but he is busy and forgets about the bill. After a while, the statute of limitations passes and the dentist can’t recover his $100 bill. The government typically wins these cases and the bill is treated as C.O.D.

ILS Coffee House from above hypothetical (with canceled loan from HLS) goes insolvent. According to IRC § 108(a)(1), you are not taxed on C.O.D. if you are insolvent

(no inclusion in gross income now). But there is nothing in the Code that lets you exclude salary if you are insolvent.

Why do we exclude C.O.D. but not salary (if you get a job when you are insolvent you are taxed on the wages)? “Reality” component - there is liquidity in salary that is not present in

C.O.D.

Page 47: Tax - Harvard Law · Web viewAmount Today/Interest Rate/# yrs into the future/total amount in x yrs 2nd way to think of it: ... Alternative: don’t deduct the repayment in year 2,

IRC § 108(b) reduces your “tax attributes” by the amount you exclude from income under § 108(a). This is an offset in a sense (you don’t have report and pay tax on it some point later by making you give up all the tax goodies, deductions/losses/basis up to your COD Income). So the next year if you sell building with a basis of 10K it will be bumped up to $60K in the current example of COD of 50K. By taking away your tax attributes, you will probably have to pay more

taxes in the future. You don’t get to choose which attributes are reduced. What other setting we have reduced basis rather than taxing income?

Inajaland, the 3 different ways to the injury to the land, etc. 108 (f) covers LIPP

Harvard forgiving your debts is not considered income under this section

Hypothetical: Ring promises to pay student $1,000,000 under seal. Student realizes Ring doesn’t have any money and so cancels the debt. Is this COD income for Ring? What is bothering the student here is that Ring has not gotten anything in return. If

Ring had been given a house, etc. in return for the money, this is an easy case of COD income. (Time 1 get $ and promises, Time 2 the debt is canceled)

Idea with COD was that you already got the benefit in cash, but we didn’t tax you then. In this case, there is nothing that we are not taxing up front. The way the Code is written though this is technically COD, but you never really

got any income that was not taxed. Ring thinks that the Code does not do a great job on the debt side (this hypothetical is

an illustration), but does a very good job on the gain side.

David Zarin (Tax Court 1989) (page 296) Compulsive gambler with gambling debts of $3.4 million that were canceled for

$500,000. TP’s arguments that the $2.9 million was not COD income:

1) Argues it was a purchase price adjustment under § 108(e)(5). If it falls under this section, it would not be taxable (similar to dentist hypothetical last week).

2) Because this gambling debt was not enforceable under NJ law at the time, TP argues that this cannot be cancellation of debt.

3) No anti-basis - the TP didn’t get anything from this debt (sort of like Ring’s peppercorn hypothetical).

4) Wants to offset the gain with gambling losses. Tax Court’s bottom line answer to this is that the gain from settlement is taxable. Tax

Court’s responses to TP’s arguments:1) Rejects the § 108(e)(5) purchase price adjustment argument on the basis that this

negotiation exception only applies to debt to acquire property. In this case, the Tax Court views the chips as money, not as negotiable property.

2) Tax Court believes that parties viewed it as a debt (as opposed to say a gift). Evidenced by TP intention to repay the debt. Legal enforceability is not a requirement for taxation.

Page 48: Tax - Harvard Law · Web viewAmount Today/Interest Rate/# yrs into the future/total amount in x yrs 2nd way to think of it: ... Alternative: don’t deduct the repayment in year 2,

3) TP did get something for this - the opportunity to gamble.4) Tax Court raises two problems with offsetting this:

Matching - gambling losses are generally only deductible to the extent of gambling gains. This COD income is not really gambling gain (it is a gain on the casino’s debt cancellation).

Accounting period - the losses were in prior years, while the gain was in this year.

Zarin v. Commissioner (3d Cir. 1990) (page 304) Circuit court does not view this as a real debt:

IRC § 61(a)(12) says that you are taxed on the income from cancellation of a debt (COD).

Appeals Court looks to § 108(d)(1)(A) for a definition of “debt.” Holds that in this case the TP is not “liable” for this debt because it is unenforceable. Therefore, there is no debt and can therefore be no cancellation of debt income.

Third Circuit analyzes the gambling debt problem as disputed debt or contested liability and finds that the settlement amount ($500,000) should be treated as the amount of debt cognizable for tax purposes. Zarin was arguing about how much he owed for this entertainment.

Therefore, the court reverses the Tax Court and holds no COD tax liability.

TP Argument in Zarin

T. Ct. Response (concludes that it is taxable)

Ct. of Appeals (reverse says it is not taxable)

Debt is unenforceable, no debt to be canceled

Parties treated it as a debt, legal liability not necessary

Agrees, looks at definition of debt under section 108 and says this is not a debt

It is price dispute

Should be able to deduct gambling losses 165 (d)

1) Occurred in Separate years2) Matching, not winnings from gambling, this is COD Income, under 165 gambling losses can only be offset by gambling gains

Money debt reduction 108 (e) (5) so a negotiation, not C.O.D.

Fail the proper requirements under 108 (e) (5)

No income, got nothing upfront

Did get something: the right to gamble

Would the situation have changed if the debt was enforceable under NJ law?

Sam borrows 3MM from bank to gamble

Page 49: Tax - Harvard Law · Web viewAmount Today/Interest Rate/# yrs into the future/total amount in x yrs 2nd way to think of it: ... Alternative: don’t deduct the repayment in year 2,

If bank lets you pay $500K for your debt you would certainly get 2.5 MM COD income, the debt is certainly enforceable

Zarin borrows from Casino One difference with Sam’s scenario Casino is not actually out of money like the

bank is, what the Casino is giving away is not cash but the ability to gamble (did they really lose $2.5MM or gain $500K + the aura he created?) But at some point it becomes unworkable.

Hired Gambler, ee of casino who gambles with casino money to create aura

2. Claim of Right Initial distinction:

1) Get $100 as a loan; there is no tax because you are expected to pay it back.2) When you get $100 from your client as a retainer, you would be taxed on that

because you are not expected to return it. We also see this distinction between a security deposit (will be returned) and

prepaid rent (will not to be returned).

a) North American Oil Consolidated v. Burnet (1932) (page 307) Facts:

1916: receivership created, property earns money but goes to neither party. 1917: money goes to the corporation because of district court decision 1922: final determination by Appeals Ct., yes the money should go to corporation.

The tax question: in which year should this money be claimed as income? The corporation did not want the income taxed in 1917 because there were high

tax rates due to the war. Argues: 1916 - that is when the money was actually earned. 1922 - that is when the company knew they could keep the money finally.

The Court holds that 1917 is the appropriate year for this income. In 1916, the company had not actually received the money. As far as 1922 goes, you could argue that nothing is really ever truly final. Would

not be good for the tax system if you did not have to pay taxes until the decision was final.

In 1917 you have a strong claim of right (you can act like it is yours), always a possibility that can someone says your claim is not legitimate but it is reasonable to treat it is as taxable at this point

b) Lewis 1944 TP receives a bonus 1946 determined that he must pay it back Question is whether you can you go back and take the bonus off on the 1944

return or you simply can take deduction 1946 The incentive is the fact that in 1944 the tax rate was very high because of WWII

Page 50: Tax - Harvard Law · Web viewAmount Today/Interest Rate/# yrs into the future/total amount in x yrs 2nd way to think of it: ... Alternative: don’t deduct the repayment in year 2,

Court says that she does have to report in 1944 and take a deduction in 1946, says that there is always chance you will have to refund it, but in 1944 you acted like it was yours under strong claim of right

Example of what might happen:

Year 1 70% tax rate $10,000 income $7,000 paid in taxesYear 2 30% tax rate $10,000 deduction $3,000 reduction in taxes

This seems really unfair. In year 1, you paid $7,000 in taxes, and then, through no fault of your own, when the bonus has to be repaid in year 2, you only get a $3,000 reduction in your taxes (you paid $4K for a transaction that gave you $0 in pocket). This is constitutional because Court has said the annual approach (Sanford) is all we are going to hold the tax system to.

To deal with this problem, we have IRC § 1341.

IRC § 1341

If 1) you included income yr 1and 2) you must return and deduction in yr 2 and 3) it is >3K

Offers the TP two options to deal with the cases where this an egregious problem:1) Can stick with original formula (see above table), take deduction of what you

repaid and you are done.2) Alternative: don’t deduct the repayment in year 2, recalculate your year 1 tax to

see how much less it would have been without the returned income, and take this amount as a reduction in taxes in year 2 (in example above take $7000 deduction instead of $3000, this is exactly what Lewis wanted to do).

TP use #1 when tax rates is higher in yr.2 than yr. 1, while use #2 when tax rate was higher in yr.1 than yr. 2

Imagine 12/31/2003, bank erroneously credits your account with 100K, in 1/1/2004, auditor sees the error and corrects it

Argue that you never did anything, it was not under claim of right, so Court would say no income in 2003 Factual arguments would be you didn’t know and it was simply a clerical

error What if you withdraw some funds from the ATM?

Maybe a better argument that it was under claim of right, but still difficult to argue

3. Embezzled Funds

a) James v. US (1961) (page 312)

Page 51: Tax - Harvard Law · Web viewAmount Today/Interest Rate/# yrs into the future/total amount in x yrs 2nd way to think of it: ... Alternative: don’t deduct the repayment in year 2,

Issues: Whether a union official who embezzled almost $1 million has to include those

funds in income. Whether the TP can be found guilty of the crime of tax evasion (“willful”) in this

case. Relevant Precedence

Wilcox embezzled not income, no claim of right Rutkin – extortion, yes, income

This Court says it was income, overruling Wilcox Concerned about equal treatment of legit and illegitimate workers In a previous case, extortionist did have to include extortion income as tax

(Rutkin) Court says it is fair to make James report this as income TP says it can be looked at as a loan (offsetting obligation) but Court says it was

not consensual transaction Does he have a claim of right?

No If it is not a loan and not claim of right, then what makes it income? Loan you must repay so we don’t tax you, COR you think its yours so you pay taxes,

court says illegal income is somewhere in the middle of these two Court opinions:

Include inIncome? Reasons Convict TP? Reasons

Warren, Brennan, & Stewart

Yes Lawful income must be included as well as unlawful money.

No There is no “willful” evasion here, past cases conflicting.

Black & Douglas No If we tax the embezzler, the real victim is hurt because he gets less money while the government makes money on the whole thing.

No If it is not income, you can’t convict.

Clark Yes Same reasons as Warren et al.

Yes Law is not that unclear.

Harlan & Frankfurter

Yes Remand for new trial

See whether the TP was relying on the previous cases.

Whittaker No Count the embezzlement as income when the debt is canceled rather than the point of stealing (kind of like a loan).

No If it is not income, you can’t convict.

Totals: 6-3 yes on income

6-4 no on jail

Page 52: Tax - Harvard Law · Web viewAmount Today/Interest Rate/# yrs into the future/total amount in x yrs 2nd way to think of it: ... Alternative: don’t deduct the repayment in year 2,

– Concern that being convicted of tax evasion for embezzlement is like a double sentence, two convictions for the same act (no embezzlers include it on their tax returns, except McKinney!)

– Also concern for evenness, greater motivation to go after those who did not report their income from illicit sources

– Do you have pay tax on a loan you never plan on paying back?o Unclear, what would happen this is right on the line of loan and theft (1st is

it not income, 2nd one is) b) McKinney – embezzled $ & reported the income

– Problem is over the undue taxes he should get credit for when he has to pay the money back

– TP has excessive deduction and wants to get it to count for another year– He argues that he should be able to carry over the Net-Operating Loss back to the

year he embezzled under § 172 because it was embezzlement was his trade (he’s done it before). § 172 the deduction has count as a business loss.

o Court says that embezzlement is not a legitimate trade/business and moreover that even if it were, embezzlement was not his trade, he had a real job.

– TP also argues that he had a claim of right under §1341 and that he should get a back deduction when he had to return the $

o Court says that he reported the income not under a claim of right but because it was stolen

– If it is stolen you have to report it but you don’t get benefit under §1341 if you need to pay it back

– McKinney loses on both accounts and is stuck with his deduction in yr. 5

4. Contributions to Capitala) Cuba Railroad

– Cuba gives the company subsidies to construct railroad, the company agreed to carry mail, troops etc.

– Court says this is not income to the company, because it is not payment for services– Court says this is under the head of contribution to capital/subsidy – Keep in mind, This case takes place in 1925, we don’t get Glenshaw Glass till much

later when the court swings towards the perspective of very expansive approach to income

– The Court mentions the fact that the government was also given land and property, which it says is of course not income, they have to say this because U.S. government was giving out property to construct railroads as well, which were not being taxed

o Otherwise there is no real distinction between cash and property – Today we have §118

o (a) income not include contributions to capital (restatement of Cuba Railroad)

o (b) cannot have a contribution to capital from any customer or potential customer (this contradicts Cuba Railroad somewhat, government of Cuba was a future customer)

– Hypo:

Page 53: Tax - Harvard Law · Web viewAmount Today/Interest Rate/# yrs into the future/total amount in x yrs 2nd way to think of it: ... Alternative: don’t deduct the repayment in year 2,

o Instead of getting cash, the company gets a bridge o Under §118 and Cuba Railroad it is not income o But the other tax consequence relates to what your basis is in the bridge,

which you want to know if you sell it or if you want to depreciate it §362

(1) If the contribution for capital is from a shareholder you get carry over basis

(2) But if it is not from a shareholder, the basis is 0 o This is reminiscent of §109, §1019 when landowner

gets land back with new building on it, not counted as income but basis is 0

o This has the effect of being a tax deferral o Another place this is done §108 (b), cancellation of

income you can put off being taxed in exchange for giving up tax goodies, which also has the element of deferral

o What about if you receive cash? We know it is not income because of §118 (a) Under 362 (b) (2), if you buy stuff with the cash within 12 months

of receiving cash, your basis in that stuff is 0 But if you don’t spend the cash in 12 months, the basis in your

other assets is reduced by that amount – If it is from a customer, you are taxed on the cash but your basis on the stuff you buy

is FMV, this of course the normal realm – In Cuba Railroad, the stuff the company bought with subsidy would have a basis of 0 – One obvious question is who is giving companies money without wanting something

in return (who would be customer)? o Federal/Local Governments – could argue that they represent potential

customers of the country but court does not do thiso Community Groups

– Hypo:o Employer who is downtown Boston, workers in suburbs, you give $20K to

a bus company buy a bus and pick them upo This is a payment for services not a contribution to capital, it will count as

income– Hypo:

o One neighbor gets money from all the neighbors to build a pool on his property for everybody

o Could view it as Neighbor is starting a business of selling pool access Gift (no income) Partnership, joint venture (not clear what would happen here)

F. Tax Expenditures and the Concept of Income1. Tax Exempt Interest

– Under §103, interest income from state/local bonds are not taxed– Helps states borrow money because they can offer lower interest rates

Page 54: Tax - Harvard Law · Web viewAmount Today/Interest Rate/# yrs into the future/total amount in x yrs 2nd way to think of it: ... Alternative: don’t deduct the repayment in year 2,

– Who else is benefiting? o See pg. 10

For the taxpayer in a 30% tax bracket, there is no after tax difference between the 20% taxable bond and the 14% tax-exempt bond.

The federal government gave up $60 of tax; the state gets a $60 benefit from lower interest payments relative to taxable bonds.

For the taxpayer in a 50% tax bracket, the tax-exempt bond provides a better after tax return than the taxable bond.

The federal government gave up $100 of tax; the state gets a $60 benefit from lower interest payments relative to taxable bonds.

The extra $40 is going to the TP This seems inequitable, unintended windfall to richer tax

payers One way to correct this is to lower the interest rate to 10%?

Why doesn’t the state do this? Because it would not be worth it to 30% tax-bracket buyer. Impossible to create an administrable dual system.

30% taxpayer Interest Fed Tax Net to TPTaxable bond (1K) 200 (20%) 60 140Exempt Bond (1K) 140 0 140Difference 60 60 0

50% taxpayer Interest Fed Tax Net to TPTaxable bond (1K) 200 (20%) 100 100Exempt Bond (1K) 140 0 140Difference 60 100 40

Why not just make this a direct subsidy?o There are a lot of politics involved here though. Some state and local

politicians fear that it would be easier for the federal government to pull this direct subsidy later than it would be for them to take the § 103 tax exemption out of the Code.

o If we decided to get rid of §103, would this be unconstitutional? No, under Baker.

What can you do from the money raised with a § 103 bond?o Traditional government actions are permitted.o After 1986, more limits were put on private use / private activity bonds.

See IRC § 141 for provisions about how § 103 proceeds might be used to help a private corporation.

Page 55: Tax - Harvard Law · Web viewAmount Today/Interest Rate/# yrs into the future/total amount in x yrs 2nd way to think of it: ... Alternative: don’t deduct the repayment in year 2,

2. Tax Expenditures– Concept that the government giving up revenue on things that they could/should

have taxed is the equivalent of a government expenditure– This concept is fundamentally based on the idea of a baseline tax system, where

Congress taxes everything that is Constitutionally permissible– Some examples

o Tax exempt bonds o Gifts? Imputed income? (Depends on what you think what is included in

“the baseline”) – Two big questions:

o Why bother with this conceptual exercise at all? A way for policymakers to talk about where $ really needs to go

(i.e. environment is already getting enough $$) For courts, the constitutional analysis for indirect tax expenditures

should be identical to direct expenditures (if it is unconstitutional for gov’t to give direct subsidy to Bob Jones, tax exemption should be unconstitutional as well)

As a strategic matter if you recognize this analysis you can figure out the best way for your client to get what he/she wants

o What is the baseline? Haig-Simon definition of income = what you consume +

accumulation (what you save) The code defines income from the sources These are two different, though not incompatible, ways of thinking

of income Three different options of a baseline

Haig-Simon (most inclusive) Baseline used by Congress and Treasury (more realistic,

“plausible, reasonable and consistent with American practice)

Current IRC– Recurring Question:

o Is better to do it as a direct subsidy or as a tax expenditure? Factors to consider

How much flexibility you want Efficiency (do you have to create a new infrastructure to

distribute the subsidy v. the tax system already being there) Annual $$ feuds (good or bad thing) Is the party you are trying to confer benefits to actually in

the tax system? (low-income individuals who don’t get taxed)

Group benefits that tax system can’t provide – national defense almost impossible to recreate through the tax system

– Baseline H-S Baseline (used by IRC

Page 56: Tax - Harvard Law · Web viewAmount Today/Interest Rate/# yrs into the future/total amount in x yrs 2nd way to think of it: ... Alternative: don’t deduct the repayment in year 2,

Congress/Treasury) Pure Income Tax includes imputed income

No tax on unrealized appreciation or imputed income

– Consumption Tax movement

o Our current IRC is really something in between H-S income tax system and consumption tax

o Another way to define income but called consumption tax base H-G tax you on consumption + accumulation Consumption Tax just tax you on just consumption

If not is at a sales tax, how do we implement a consumption tax, the formula below

o Consumption = “income” (consumption + accumulation) –savings

The difference between them is the base which is being taxed, we can graduate the tax in both systems

o Example Yr 1

$100 Salary $70 spend $30 save CT world the taxable income is $70 IT world the taxable income is $100

Yr 2 $0 salary $30 spend CT tax on $30 IT tax of $0

o How do you decide on which system is preferable? Fairness

Even if you have a graduate consumption tax rate schedule it can’t get to the following situation: someone who makes $10 will be taxed the same as someone who makes $100K if they both spend $9K in a given year, how does this matter? Rich people are able to defer their consumption and taxes when the taxes are lower. What is the right measure? The resources you use up (consumption) or the resources you control (salary)?

Efficiency $100 year in wages, 10% interest return, 50% tax rate

If consume all $100 yr. 1, Leaves nothing for yr. 2

If you don’t consume in yr. 1 till year 2

Comparison of yr 1 to yr 2 consumption

Page 57: Tax - Harvard Law · Web viewAmount Today/Interest Rate/# yrs into the future/total amount in x yrs 2nd way to think of it: ... Alternative: don’t deduct the repayment in year 2,

No tax $100 $110 100/110 50/55

Income tax $50 $52.50 50/52.50Consumption tax

$50 (50% of $100 consumption)

$55 50/55

Wage Tax $50 $55 (no tax on interest income)

50/55

– Less likely to save in IT world v No tax world because you lose more of your consumption by saving

o Aside: The tax base in CT is $ you consume + $ of your taxes paid, we hide the idea that you are taxed on the money you are paying taxes better in IT world but the issue exists in both worlds

o Efficiency arguments that consumption tax system is better because you do not discourage savings

But doesn’t a wage tax achieve the same goal? Then it can’t be the only reason to advocate for a consumption tax

Some intersections between wage tax and consumption taxo A trust-fund baby would never be taxed in a wage

tax world but would be in the CT world o The difference in the I.T. world is that you are taxed on your investment

incomeo But do people fine tune their decisions specifically based on the tax

system? This is an empirical question we need to pursue? o How much of our system is income tax v. consumption tax world.

Income Tax (H-S) CTOwner occupied housing Tax it is because it is

consumption (our system deviates)

Tax because it is consumption (our system deviates)

Tax Exempt Bonds Taxed as accumulation (our system deviates)

No Tax (our system does not deviate)

Retirement Savings Taxed as accumulation or the interest income (our system deviates)

No Tax (our system does not deviate)

– From this chart looks like our system is more like Consumption than Income tax!

II. DeductionsA. Profit-Seeking Expenditures

1. Ordinary and Necessary Business Expensesa) In General

– Why allow business deductions at all? o Consider two TP’s

One sells advice for $100K

Page 58: Tax - Harvard Law · Web viewAmount Today/Interest Rate/# yrs into the future/total amount in x yrs 2nd way to think of it: ... Alternative: don’t deduct the repayment in year 2,

One sells shoes for $100K, costs him $90K to make the shoeso If you don’t allow normal/ordinary deductions you have distorted the

situation between these two taxpayers – Could you deduct dinner tonight?

o §262 says no deductions for personal deductionso Why not?

Well, the dinner represents consumption and in an IT world we are taxing consumption

– What if you buy land for your family farm, can you deduct it that year? o You are consuming the food off of it o Also it is capital expenditure, you don’t use it all up in that year, §268 says

that these types of purchases cannot be immediately deduct

(1) Welch– Welch paid of the debts of his defunct bankrupt company in order to improve the

current reputation of his new company, he wanted to deduct these expenses o Court says that the expense was necessary but not ordinaryo Court also says that is a capital expenditure, so the goodwill investment

will probably be recovered as basis – Out of this emerges a picture of §162:

o Trying to keep out personal expenses (263) and capital expenditures (262)o “Necessary” really means it can’t be personalo “Ordinary” real means it can’t be a capital expenditure

– Why can’t you say that a fancy office is not necessary? o We’re not really using necessary in the ordinary sense of the word

– What are the other reasons you are denied an exemption under 162?o $1000 for an MBA, probably deductibleo $100 for business advice from a wizard? o $800 to a religious adviser? In Amend v. Commissioner, TP tried to argue

for this but he lost. o Vitali v. Commission spend $$ for research on prostitution for a book, 4th

Circuit said it was just too personal, demonstrates that a lot of this is just intuitive aspects

– Courts sometimes say that deductions is an act of “legislative grace” as opposed to an absolute right, or that the 16th amendment requires it

o Since there is no baseline right to it, courts say they can interpret deductions as narrow as they want

o But do they really think they could get rid of §162 in full? Consider the first example.

o In Sanford v. Brooks and Dobson, we saw the idea whether there is a distinction between having basis and having deductions, does the tax system treat them differently?

b) Public Policy Considerationso Under 162 (c) (1) can’t deduct bribes to officials (criminal conviction not

actually required)

Page 59: Tax - Harvard Law · Web viewAmount Today/Interest Rate/# yrs into the future/total amount in x yrs 2nd way to think of it: ... Alternative: don’t deduct the repayment in year 2,

Seems to create an Income Tax problem but we are importing other values into the system

Also, recipient is not going to report it so we have mismatch o Suppose you are in charge of buying for household store and one of the

vendors offer you a kickback and you accept it Under 162 (c) (2) other business kickbacks are also not deductible Have the same problem of mismatch if we allow it (you won’t

report income as o Kickbacks for Medicare are also not allowed as deduction even if they are

legal Here the value we are worried about is spending too much on

Medicare as opposed to something technically being illegal o International Businessman bribes a foreign official

If unlawful under U.S. law (doesn’t matter if it is illegal in that country) you cannot deduct it under 162 (c) (1)

o Fined for polluting Under §162 (f) cannot deduct fines, why not?

Mitigates the deterrence if you can just deduct a fine o But this usually depends on the facts of the case and

the numbers, but we are erring on the side of being conservative

– Hyp: earn $100 per trip by overloading will be caught 1 in 10 times fine is $1100, Tax Rate is %50, 10 trips

o Before Tax receipts $1000 fine $1100 Net loss = -$100

o After Tax and you can deduct finesReceipts $500 Fine is actually $550 Net loss of -$50, still not worth it but less so

– Suppose what are you doing is speeding to get home and you get fined, how do you set deterrence in this case? Here the good is not $$ but extra time/leisure with family, which is not taxed itself. So when we set the penalty, before we ask whether the fine should be deductible we need to figure out what to weigh the fine against.

– Civil Damages, as opposed to fines, are deductible under 162 (f) o Under 162 (g) convicted of anti-trust, can only deduct 1/3 of treble

damages (treble damages include punitive damages), the penalty piece is not excludable, this is consistent with the rest of the code

– Suppose you have an illegal prostitution business and have enormous telephone billo Before 1970, court says unless you are violating a major public policy you

can deduct, just because it is related to an illegal business is not enough Real question whether IRC should be used to police morality, these

early cases say not unless Congress is very clear about it. Tax system is just around to tax income, other regimes can handle morality.

Page 60: Tax - Harvard Law · Web viewAmount Today/Interest Rate/# yrs into the future/total amount in x yrs 2nd way to think of it: ... Alternative: don’t deduct the repayment in year 2,

o In 1970, Congress added 162 (c) (2) saying if the payments are illegal then they are not deductible, prostitute’s telephone bill still seems deductible

o 280 (e) came into effect 1982, which does not allow any deductions involved drug trafficking

Why this special treatment for drug business? Passed during the drug war, Congress taking a tough line against drug dealers, but is this the best way to deter drug dealing? Drug dealers do not file returns till they are already in jail.

One interesting point, when you look at the legislative history and they say they don’t mean the cost of drugs themselves, that it is still deductible (not things like paying the runner, airline tickets, etc.). Why was Congress worried about? They are concerned about constitutional restraints, there is something more protected in basis than other expenses. They are worried about going too far.

– Legal Feeso Hypothetical: you are sued for legal malpractice and hire another lawyer

to represent you. Are these fees deductible?o Yes, even if you lose, it is about your business. If the expenses are in the

course of business, it is deductible. What if you are securities dealer instead of lawyer and face a

lawsuit? Still deductible under Tellier and §162. o It is even deductible if the proceeding is criminal and you are convicted.o Doesn’t this go against what we were talking about in § 162(f) where we

did not allow the deduction of fines? Again, court says unless it clear that Congress wants to disallow it

will not be disallowed under public policy. Also, legal fees may be viewed a little differently because of the

pervasive right to counsel that we have in our system. Fines are used to deter your behavior, legal fees are to protect your rights.

o May not want deductibility of these expenses to depend on the outcome of the trial.

Lobbying expenses: Hypothetical: Bill in congress to raise the prices of milk by $2000, you are a milk

producer and pay a lobbyist to lobby Congress on a milk bill. Post 1993, you cannot deduct the expense of this lobbyist. IRC § 162(e) (1) does not allow deductions of non-local lobbying expenses.

If you are a consumer group, all of your lobbying expenses may not be deductible. This group is probably not a trade or business under § 162(a), the interest here is

personal, not under a business. You don’t even get to the lobby rules because this is not even a business.

Issue: $2,000 increase in milk prices, how to determine whether you should spend the $$ to lobby. In a 50% tax world, the $2,000 issue is worth $2,000 in before tax income and

$1,000 in after tax income for the producer: if producers are successful, will get

Page 61: Tax - Harvard Law · Web viewAmount Today/Interest Rate/# yrs into the future/total amount in x yrs 2nd way to think of it: ... Alternative: don’t deduct the repayment in year 2,

$2000 pre-tax increase in revenues, which is only $1000 post-tax, so will spend only $1000 to lobby

For the consumers, the $2,000 issue is a $2,000 “cost” after tax, tax incentives are irrelevant to them

How much will you spend in lobbying expenses on this issue?

No Tax WorldPre 1993 Tax Rules

(lobby expenses deductible)

Post 1993 Tax Rule(lobby expenses not

deductible)Producers spend up to $2000 spend up to $2000 spend up to $1000Consumers spend up to $2000 spend up to $2000 spend up to $2000

In the Pre 1993 rules, you can spend up to $2,000 as a producer because the after tax cost of lobbying would only be $1,000, which is equal to the after tax income you would receive from the issue.

In Post 1993 when the lobbying costs are not deductible, producer’s total gain is only $1000 so that is the maximum they will spend to lobby against the bill

Some trade associations that might have tax exempt status and engage in some lobbying. Can the milk producers’ payments to the trade association be deductible? IRC § 162(e)(3) states that no deduction is allowed for payments that go to

lobbying. The IRS is trying to keep people from using these tax exempt groups to avoid

the rules. Under 162(e)(5)(a) people in the business of lobbying can deduct their expenses,

so while the milk producers cannot deduct the lobbying costs the trade association can

You can deduct your lobbying expenses at the local level. If § 162(e)(5)(a) comes from the idea that there is excessive lobbying by the

producer side of the universe, there may not be the same disparity at the local level and therefore, you may not need the same incentive changing tax rules.

Think about language in Sullivan language about what Tax System is supposed to do and think about today’s code and what that says about the change in vision of what the tax system is.

c) Salary Salaries are deductible.

One of the things we worry about is whether it is salary, or something else. Who is being taxed?

If salary, the business gets to deduct it and the TP is taxed on it. If dividend, the corporation does not get a deduction and the TP is taxed on it.

Therefore, it is better for the corporation to pay salary than dividend. If gift, the corporation does not get a deduction and the TP is not taxed.

Keep in mind, whose being taxed, at what rate, and are there any deductions.

Page 62: Tax - Harvard Law · Web viewAmount Today/Interest Rate/# yrs into the future/total amount in x yrs 2nd way to think of it: ... Alternative: don’t deduct the repayment in year 2,

For instance, might want to disguise gift as salary in cases of small business (differential tax rate for corporation v. individual)

– Patton:

Salary depended in part on the net profits of the corporation. Because of a big new contract, the employee starts getting a lot of money.

Is this salary? There is no suggestion in this case that they are trying to make a tax-free gift here. The employee does not have an interest in the company, so there is no dividend

here. We are left with the idea that this is salary (not something else).

The court does not allow all of the salary to be deductible. Seems to put a lot of weight on the idea of reasonableness.

They say $13K is reasonable, TP should have the burden to prove that the $46K was reasonable

But there is no requirement of reasonableness in other types of expenses, like a phone bill.

Dissent says that if two independent parties enter into to a contract which produces this salary, who is the court to say it is not reasonable. Also emphasize that the $13K is completely arbitrary.

If you look at just the facts of this case, the company makes this arrangement right after the new contract. You can infer that there was some arrangement for Kirk to get the money as salary at a low tax rate and then turn the money back to the owners (perhaps under the table). Otherwise the result seems puzzling.

Limits on deductible salary: § 162(a)(1), requirement that a “reasonable” salary or allocation for salary is

deductible Historical origins: For small business, allowed a deduction for reasonable

allowances for excess profits, even if you don’t specifically account for it in your business but there is no longer any excess profits tax, so this irrelevant but phrasing remains

Current Use: IRC § 162(m) limits how much a publicly held corporation can deduct as salary to its

top employees. Limit is $1,000,000 for certain top officers Why do we have this provision?

Some concern that these executives are “de facto owners.” Response to public concern over compensation levels of these executives.

This section’s enactment may be similar to the adoption of the lobbying expenses provision and the drug dealing deduction provision.

In the immediate sense, the actual penalty falls on the shareholders There are exceptions for those paid on commission/performance so there are

creative ways to avoid this by writing in easily attainable goals

Page 63: Tax - Harvard Law · Web viewAmount Today/Interest Rate/# yrs into the future/total amount in x yrs 2nd way to think of it: ... Alternative: don’t deduct the repayment in year 2,

Relationship between § 162 and § 132 (fringe benefits): All of the fringe benefits in § 132 were exclusions (not on your income tax return).

Deductions play a similar role. In theory exclusions and deductions should work out the same.

– 61 + 162, if you get cash to buy copy paper you have to report as income and then deductible, under 132 (d) if employer just buys you copy paper no tax effect at all. 132 (d) just replaces the messier two-step process.

– Sometimes the parallel works and sometimes it doesn’to If employer sets up gym on premises and you are allowed to use, no tax under

132 (exclusion picture) o But if employer gives you extra cash to buy a gym membership, you get taxed

on the cash and no deduction, thus this not the same picture as above our picture with copier paper

§212– Deals with people who don’t necessarily have a trade or business but you are profit

seeking, not a truly personal activity– Business expenses deductible under §162, personal expenses no deductible under

§262, profit seeking deductible under §212 – §212 looks very similar to §162, “ordinary and necessary” requirement, but

sometimes 212 expenses are more limited than 162 – What is an example of profit seeking? Investing for yourself from home (day traders),

generally courts always consider it profit seeking rather than trade and business. (though probably not going to be successful in deducting their own time.

B. Capital Expenditures– Typical example: buy land/machine for a factory you cannot deduct the cost of it today

Rationale: you spent money that is going to generate income over multiple years– Contrast with postage for advertising you do get the deduction because it is being matched with income generated today– Buy $1,000,000 for a printing press and produce $80K in year 1, won’t get to deduct all $1,000,000 at once, instead it should be a little bit each year that you generate income from the expenditure

a) Mt. Morris– Settlement reached to build a drainage ditch to prevent run off from the drive-in to

adjacent farmland.o Issue is over the recovery of these expenses. TP argues that costs should

be currently deductible.– Holdings:

o Mt. Morris Drive-In Theatre Co. (T.C. 1955)

Page 64: Tax - Harvard Law · Web viewAmount Today/Interest Rate/# yrs into the future/total amount in x yrs 2nd way to think of it: ... Alternative: don’t deduct the repayment in year 2,

Tax Court holds that the drainage ditch is a capital expenditure because it is a “permanent improvement.”

Also emphasis that purchaser knew that a drainage system would probably be necessary when he bought the land, it was part of the original capital expenditure

o Mt. Morris Drive-In Theatre Co. v. Commissioner (6th Cir. 1956) Appeals Court also holds that it is capital because it adds to the

value and is prolonging the life of an existing asset ( a little different than analysis of tax court which emphasized that it should have been known at the time of purchasing the land).

– Dissent analogies this to a cash settlement, which would have been deductible as an ordinary and necessary business expense. Also, they only built the system as a result of litigation.

– Another hypo comes from Midland Case:o TP had a ham factory, TP had a problem with the floor so re-sealed it,

question whether it should be deductible? One factor is that in this case the cost is unexpected Also this might be a more frequent cost like a repair v.

replace/create Something like painting the walls every year will be

deductible Improving looks more like capital expenditure,

maintaining/prolonging life usually currently deductible o The court did allow a deduction in this case

– A new sprinkler system in a building? Probably capital, don’t need to put one in every year (though don’t take this so literally)

– How about if you needed to put it under a change in fire code? Well seems unexpected but Ring thinks this is still a capital expenditure.

– What about repairing your old sprinkler system? Straight up repair is currently deductible, but can see how this could all of sudden become a replacement.

– What if there is an earthquake? Still a capital expenditure if you are replacing the system, though will probably be able to deduct as a loss.

– What if it is “destroyed” by a change in the building code. This will probably be treated differently; may create some loss.

– Identifying expenses which you cannot deduct immediately but rather over time, that is the crux of capitalization of expenditure.– Next question is when you capitalize.– Regulation 1.263 (a) -1, 2 – scopes out when it is you capitalize and expenditure – Hypo – buy an insurance contract related to your business, 3 year control, and you pay upfront $3K o You want to deduct it immediately, IRS would say you should capitalize over

the 3 years since you will get benefit over 3 years o If you buy machine $100 with lifespan of 5 years, classic case of having to

capitalize, deduct $20/yr (depreciation)

Page 65: Tax - Harvard Law · Web viewAmount Today/Interest Rate/# yrs into the future/total amount in x yrs 2nd way to think of it: ... Alternative: don’t deduct the repayment in year 2,

b) Idaho Power– TP is self-constructing his own factory

o If they had hired someone to build a factory, it would have to capitalized– TP buy a bulldozer with a lifespan for 5 years to build a factory with life of 40 years– TP wants to depreciate over 5 years, TC wants to depreciate over 40 years– Instead of $20/yr of deduction, $20/40 yr because the bulldozer is creating income

over 40 years (it is like brick) – Court agrees with TC, saying that depreciation has to be capitalized over life of the

building – Wants to be consistent between those between those who buy construction versus

those who self-construct (check this)c) Indopco

– National Starch bought investment advice – they were going to be acquired by Unilever. They hired Morgan Stanley to evaluate the merger. They want to deduct the investment banker and legal fees– IRS though these fees should be capitalized because they are adding value over a long term – Tax Court agrees, says that it should be capitalized, 3rd circuit affirms– Sup. Ct. agrees because even National Starch said it would improve long-term values– TP argued that Lincoln Savings case says that a separate asset must be created in order for capital expenditure to be available, Court says that this factor is sufficient but not necessary; no defense to capitalization that there was no separate asset present – Tone of the case is that we should presume capitalization most of the time, this led the IRS to become very aggressive against current exemptions, unclear whether Sup. Ct. really wanted this expansive interpretation of Indopco (PNC case)

d) 263A – Uniform capitalization rules, try to cope with the fact that there was a lot of uneven

treatment of business expenditures– For certain kinds of matters try to standardized treatment, requires TP to capitalize

certain direct and indirect costs for producing real property – Put in place to help some uniformity for when to capitalize

Why does everyone fight over capitalization?

Capitalized Expenditure

Yr. 1 invest Yr. 5 divest Profit Rate of ProfitPre-Tax $100 $200 $100 100%Tax (or refund) 30A.T. 100 170 $70 70%

– In yr. 1 there is no taxable event

Page 66: Tax - Harvard Law · Web viewAmount Today/Interest Rate/# yrs into the future/total amount in x yrs 2nd way to think of it: ... Alternative: don’t deduct the repayment in year 2,

– In yr. 5 you are taxed $30 on $100 gain

Expensed Capital Cost

Yr. 1 invest Yr. 5 divest Profit Rate of ProfitPre-Tax $100 $200 $100 100%Tax (or refund) (30) $60A.T. $70 $140 $70 100%

– The $30 you represent how much your taxes are reduced by, your income reduced by $100 and tax rate is $30

– In Yr. 5 the $60 represents the tax you pay upon sale, your basis is $0 since you took the deduction in yr. 1

Even if you invest $100, after-tax, you get the same result (some people like to see the same after-tax investment and see if it still works)

Yr. 1 invest Yr. 5 divest Profit Rate of ProfitPre-Tax $143 $286 $143 100%Tax (or refund) (43) $96A.T. $100 $200 $100 100%

– A couple of places where TP have the choice to deduct or capitalize, §173, §174, §175, §179 (soil and water conversation, small business), when we are trying to encourage a certain type of businesses/industries

2. Recovery of Capital a) Losses

– When you buy a house for $10,000, nothing happens on tax front, no realizable event to tax and it is capital expenditure so no current deduction, this will be deducted at the time you sell because assets like land are thought to be indefinite so they are not capitalized over specified period of time

– If you sell $9000, you have a loss of $1000 which you are allowed to deduct and this point your investment is fully recovered

(1) SS White Dental– 1918 – German Company invested $130K in property, later that year all property

was seized by government, TP sought all $130K loss deduction – IRS says that there is no realization event, no closed transaction and there is a

possibility that TP will get property back– Court allows Company to treat it as a loss, TP does not have to establish that there is

no possibility that property will be returned , it is enough of an event to trigger loss deduction so TP is allowed to take full $130 deduction in 1918

– If TP gets $6K back in 1921 you will take that gain to remain consistent

Page 67: Tax - Harvard Law · Web viewAmount Today/Interest Rate/# yrs into the future/total amount in x yrs 2nd way to think of it: ... Alternative: don’t deduct the repayment in year 2,

– HYPO:o TP has 100 shares in German Co, and value of declines from $100K to

$10K o If there is no actual sale, there is no loss deduction, we are looking for

something more as an event, mere fluctuation in value is not a trigger point (just like unrealized appreciation from earlier in the course)

(2) Cottage Savings– TP is savings & loan org., their pool of mortgages that declined in value and they

wanted to sell it but fed regulations will not allow them to– There is a change in regulatory approach that you can sell the mortgages if you

exchange your pool for substantially similar pool, it will be allowed by fed regulations

– TP wants this exchange to trigger loss so they can deduct, they say an exchange should trigger a loss

– Is this realization event?– Are the pools of assets materially different from each other? – For a realization event, IRS says there needs to be an exchange of materially

different things in order for tax realization and here the pool of mortgages are not different, they are economic substitutes

– Tax Court rules in favor of TP – Court of Appeals overturns– Sup. Ct. ultimate decision is that exchange of assets must be materially different in

order to trigger realization, but as long as the two assets are two distinct sets of legal entitlements they are different, a very low threshold. SO in this case TP can realize the loss because the pools of mortgages are legally different assets.

– This case basically holds that it is easy to trigger a realization event.– This “easy trigger” rule, in the long run, may give more flexibility to taxpayers to

trigger realizations.

A special set of loss rules

Capital Asset Losses: – Must remember Capital Assets has nothing to do with capitalization, indeed

capitalization need not be for an asset – Capital assets are special subset of purchases define by §1221, there are special rules

for gains/losses of the assets and the losses may be limited

KG KL (§1211 (b)) 1986 Special partial tax Limited1987 Full tax LimitedNow Special partial tax Limited

– Sometimes depending on the kind of asset there will be limits on losses you can deduct, we will be dealing with this later in the course

Page 68: Tax - Harvard Law · Web viewAmount Today/Interest Rate/# yrs into the future/total amount in x yrs 2nd way to think of it: ... Alternative: don’t deduct the repayment in year 2,

Personal Losses:– What happens if you get robbed can you deduct the loss? Yes, under 165 (c) (3)

o Theft losses specified in the code, why? The stolen income can’t be used for consumption. Income is supposed to be accumulation + consumption, if it is stolen it is neither accumulate or consumed.

– What if you buy a house for $100K and then a highway ramp is put next to your driveway, and the value of your house plummets to $10K and you sell it for $10K

o You want to deduct the loss for $90K, under 165 you probably can’t deduct

Could argue it is a “disaster” beyond control Could also argue that it was a profit seeking loss rather than

personal loss o Under 1.165 –a, the answer is no, you can’t deduct loss in house

Why? Because you consumed the house. Not going to ask how much declined because you consumed versus how much it declined because of the highway ramp.

Gambling Losses:– 165 (d), you can deduct gambling losses up to the point only to the extent of

gambling gains – Why?

o They are like consumption but it is potentially profitable, so it is like buying movie ticket or a profit seeking venture

o Because are uncertain we have this intermediate treatment o To the point it is a profit seeking venture we allow you to deduct but as

you have no more profits it is more like going to the movies a lot We don’t want to subsidize entertainment

Losses between related parties:– Ring sells stock to her brother for a loss, why would IRS worry about this?

o How do we trust that this was the market price? o What if price is verified? Any other concern? Well, is it a really sale? Is

ring still controlling her brother’s management of the stock? – In the code, §267, does not allow for loss deduction for transfers within the family,

they define family linearly (aunts, cousins do not count as family in these cases, but watch out in other areas of the code)

Resale for: Gain Scenario Loss ScenarioSeller’s basis $1000 $1000Sale price to brother $900 $900Disallowed loss (100) (100) Resale Price $1500 $500Transferee realizes g/l $600 ($1500-$900) -400 (500 – 900) Transferee recognition g/l $500 ($600-$100) -400Family g/l $500 -500

Page 69: Tax - Harvard Law · Web viewAmount Today/Interest Rate/# yrs into the future/total amount in x yrs 2nd way to think of it: ... Alternative: don’t deduct the repayment in year 2,

– In scenario #1, 267 (d) says that brother only has to recognize a $500 gain, even though his basis is $900 and his actual realized gain is $600, when he sells outside of the family he can take the $100 loss

– In scenario #2, 267 (d) does not say that you can increase a family member’s loss and the IRS has not done allowed for it either, we saw this earlier when we were very nervous about transferring losses

o If someone already losses we get uncomfortable in transferring more losses to them, not as concerned when they have a gain, sort of inconsistent fear but still pervasive

o This suggest transferring a depreciated asset within the family is a bad idea because you will loss some of the loss

– 267 DOES NOT AFFECT ANYONE’S BASIS, it only has to with recognition of gain and losses

– What type of property does 267 cover? All property, not just stocks.– §1091, aiming for people who sell depreciated property and buying it up again, but

scope is only for sale of stock and securities unlike §267

11/10/03 ended with §267 last week Buy car for $100 and sell it for $75 get loss deduction for $25 so that fully recover

investment. What does §1091 cover? Sales of stocks and securities §267 covers any property §1091 – She buys stock and sells it for a loss. Then buys more shares. She cannot

take the loss because not really realized because she is ending up in same position moments later. We have a realization-based system. Need to have an event and cannot create one that is not really real – here really own same shares.

o Under §1091, if buy/sale/buy within 30 days on either end, cannot take loss. Cannot buy 5 years in past, then buy today and then sell tomorrow either. Order is insignificant.

What happens to a loss? Hypo: Buy stock for $100 (basis), sell for $90 and same day repurchase for $90. TP

would like loss of $10, but $10 is disallowed here. Then want to figure out with current shares what basis is. See §1091(d).

o Basis of new shares = basis of stock sold + (price of new – price at which old was sold)

B = 100 + (90-90) B = 100 (this makes sense because loss was not recognized under the

rule)o Alternative view: Basis of stock that was repurchased effectively equals price

you pay for it plus any disallowed loss form prior sale. 90 + 10 = $100 (this is Ring’s approach, but not IRC approach)

Loss Case Hypo: Buy for $100, sell for $90 so that disallowed loss is $10. When repurchased, paid $85. Basis of these shares is basis of stock originally sold ($100) + pirce of new stock ($85) – price at which original sold ($90).

Page 70: Tax - Harvard Law · Web viewAmount Today/Interest Rate/# yrs into the future/total amount in x yrs 2nd way to think of it: ... Alternative: don’t deduct the repayment in year 2,

o B = 100 + (-5) o B = 95o So price at which purchased stock ($85) + disallowed loss ($10) = $95.

Gain Case Hypo: Same scenario with repurchase price at $95. Basis = 100 + (95 – 90) = 105. Or new shares ($95) + disallowed amount ($10) = $105.

Mechanism for dealing with disallowed loss changes the basis. Prior loss is built into basis.

Because it brings disallowed loss into new basis, will always get it back. Why do you think this is always guaranteed when get true realization? Compare this to §267.

o §1091 focuses on timing for one TPo §267 focuses on timing and price as between two TPs. Are talking about

shifting of losses. Concerned about allowing other people to take your losses. Not concerned when only you are involved – will give it back to you.

Can you “wash a gain”?o WAG = wash is that sell stock and immediately re-buy it at same price. Want

to trigger a gain without getting out of economic position. May do this to offset loses during the tax year. Or maybe foresee increase in tax rate and want to pay taxes now.

HYPO: Buy 20 shares at $30 and then 20 at $50. Shortly thereafter sell 20 shares at $40. Is this a §1091 loss? Issue is which shares were sold – need this info to determine whether it was a loss or a gain. Common issue. §1091 allows you to choose. If do not do so, regulations assume first in is first out for purposes of sale. “FIFO”

McWilliams: Husband sells shares to trigger loss and then has wife’s account purchase the shares. Inter-family sales are not allowed under §267. Wants to trigger loss, keep economic position, and avoid §267.

o How do we know that TP wanted to do this? TP told $ advisor.o Tax Issue: Whether covered by §267

Is not direct family sale. Is very real intermediary. o Ruling: Spouses have same tax interests and intermediary does not apply,

therefore, §267 applies and does not allow taking a loss here.o §1091 – loss limit, single family and change ????? TP says if wanted to cover

this, IRC would have included it in §1091. Court disagrees. §267(g): §1041???? Something about father/daughter vs husband/wife transactions Courts not eager to apply McWilliams unless is totally obvious and intentional Limits: Realization, capital losses, losses on personal assets (sale of home), gambling

losses. Idea that should get investment back even if through losses, but is limited by these things.

Illegal Activity: o Mazzi: counterfeiting con-artist with black box where money was stolen and

TP declared a loss Issue: Is loss deductible under circumstances? If was regular robbery,

could take loss. Ruling: Counterfeiting illegal with loss closely-connected to it. No

explicit provision barring this type of loss in the IRC. Congress limited public policy exceptions in §162 – has built-in exceptions.

Page 71: Tax - Harvard Law · Web viewAmount Today/Interest Rate/# yrs into the future/total amount in x yrs 2nd way to think of it: ... Alternative: don’t deduct the repayment in year 2,

Court analogizes this to paying fines which is explicitly covered in IRC. So court adding in PP that was not explicitly stated by Congress.

Concurrence support: was voluntary in terms of putting self in this position. Allowing this loss does not mean are including criminal money in GI – are not taxing counterfeiting business proceeds but rather are just disallowing a loss.

Dissent: Need to follow statute which is explicit with regard to making PP exceptions. Also, what if TP got sick on the way to NY and had medical, could have deducted those expenses. Should that be disallowed as well since is related to criminal activity? So where is majority going to draw line. What if robbed of $10K on street by stranger before entered hotel? Does it matter that only reason had money was for this potential crime? What if burned up in the oven?

b) Defaulted Debts Bank lends money to business that defaults. Can bank take deduction for bad debt?

Yes. See §166(a). Suppose student loans Ring $10K. Can student take deduction? §166(d)(1)(b) says if

non-business debt, is treated as loss from sale or exchange of short-term capital asset (held less than a year). This relates to §1211 limiting ability to get loss. This is less desirable than business treatment of losses.

§165(g) – basic provision about losses – worthless stocks and securities – KG/KL is capital asset (not about bad debts)

§165(e) – basic provision about losses – bad debts not included in §165(g) §166(a) – OL – business bad debt” §166(d) if loss from non-business bad debt and is short term capital loss then base

restriction on §1211. Both contracts and expands ways can get bad debt loss §162, §212, §165 talks about losses §262 – don’t get personal expenses deducted including losses §165(c) is similar In table see dividing line between personal and everything else, except that with third

row (bad debt – is it a loan or a gift?), is line between business and everything else. Is too hard to police line at personal division.

Business Expenses §212 Profit-Seeking Personal162 212 262165 165 165(c) some166(a) 166(d) 166(d)

Recall administrative component in terms of policy issues for deciding the dividing lines here.

Payor PayeeGifts Not Deductible Not TaxedBad Debt Partial Deduction Taxed

Page 72: Tax - Harvard Law · Web viewAmount Today/Interest Rate/# yrs into the future/total amount in x yrs 2nd way to think of it: ... Alternative: don’t deduct the repayment in year 2,

Whipple – helping to manage a corporation does not make your work a trade or business and therefore, any losses from this cannot be deducted as business losses, but rather must be treated as non-business losses which are treated like the sale of short-term capital assets.

o TP owns and loans money to Mission Orange Corp. Wants to deduct under 166(a).

o Issue: Whether can deduct under §166(a)o TP is managing – has to show that he has a business in managing companies

in order to show loss arose out of trade/businesso Court – Cannot deduct loss under §166(a) because did not come from

business/trade even though is shareholder. He is not in the lending business either. Court says may be in lessor (owns building and property that he is leasing) business/trade and so remands.

o So think of Whipple/TP as investor in stocks and securities so that if tries to say bad debt, gets better tax treatment 166 ensures that even if labels it as loan, still cannot get full treatment??

Generes (page 683)– construction company with father, son and son-in-law who all own stock in and are employees of company. Is money that TP has loaned to company a bad debt loss? TP said should be viewed in trade or business of being an employee. TP loaned company money in that context (trying to protect job) and so is business bad debt deserving full deduction with no restrictions. Court said dominate motive must be trade or business and that here, TP’s role as investor was dominant (not role as employee since equity was huge and salary was minimal).

3. Depreciation– Goal is to find net income because illustrative of how much tax you can pay. Method of getting at this is to find it and then allow TP to recover for costs incurred. o §162/212o §1055 loss/sale - dispositiono §263 – capitalize cost – can depreciate/amortize

Important to determine when depreciation/amortization will apply:o Antique Musical Instruments - Richard Simon: Antique bows used by

violinists can be depreciated. Issue: depreciate bows Gov’t position: antiques increase in value and so should not be

allowed to depreciate for tax purposes. Court: Irrelevant that value may have increased over time or that as a

collectible an item has value independent of it use, thus giving it unending useful life. §168 has four requirements (page 693) which are met here:

(1) tangible(2) placed in service after 1980(3) of character subject to allowance for depreciation

(have to have suffered wear and tear)(4) used in trade or business or held for production of

income

Page 73: Tax - Harvard Law · Web viewAmount Today/Interest Rate/# yrs into the future/total amount in x yrs 2nd way to think of it: ... Alternative: don’t deduct the repayment in year 2,

Court: Irrelevant that value may have increased over time or that as a collectible an item has value independent of it use, thus giving it unending us

HYPO: Imagine sell bow in Year 6 for $40K which bought in first year for $20K and depreciated $4K each year. Think about AR ($40) –B ($20) = G/L ($20K). This is what would have happened if TP lost in Simon. Because TP won, let TP recover $4K each year so that at end of year, have no basis left to recover. Because have nothing left to recover, AR ($40K) – B ($0) = $40K. SO now have large gain that have to report on return.

– Issue: What kind of assets are going to depreciable? Are you going to have to wait till you sell it to recover your basis or can you do it over time?

– Simon: Does it make sense to depreciate assets that are actually increasing in value? The bows were wearing out but they also were increasing in value. This is what the court was struggling with. Court says for your purposes, for your business, it is wearing out.

– World Publishing: o TP A leases land to TP B, TP B will pay 10K rent for the next 70 years,

then TP A decides to sell the property to TP C for 700Ko The land itself is only worth 300K, what is the other 400K for?

The rental lease a guaranteed rental stream, locked in at 10K/year for 70 years

Must assume that is above market value if he is willing to pay 400K for it

o TP C cannot depreciate the 300K for the land have to wait till sale, but hoping that you can recover the premium lease (as intangible asset with a definite life) over the set term of the lease

Before 1990, Courts split on how to deal with recovery of intangible and in particular, premium leases

In 1993, congress comes into organize the rules about intangibles; §197 specifically excludes interest in land

Also in 1993, §167 (c) (2) when property is acquired subject to lease, all goes into the basis of the property (i.e. plane (400) + lease (300) basis of property is 700K, depreciated over the life of the plane)

Ignores premium leases and folds them into the underlying property

Is this good for TP? Depends on life of lease v. life of plane, you always want to recover more $$ sooner rather than later. Varies on the situation.

Not clear that 167 (c) will not address our situation because it is not subject to land, only subject to property that is subject to 167 generally

Page 74: Tax - Harvard Law · Web viewAmount Today/Interest Rate/# yrs into the future/total amount in x yrs 2nd way to think of it: ... Alternative: don’t deduct the repayment in year 2,

o This might even be bad in the case because if you incorporated lease into the land, you couldn’t recover any of the 700K until you sell all the land

– Newark Morning Ledger o Herald buys Booth’s assets one of which is paid subscriberso Issue: Can TP depreciate the paid subscribers?o IRS says no, it is not the same as goodwill, where the general rule is no

depreciation o CT says it is separate from goodwill and it will inquire whether paid

subscribers have: It has ascertainable value Limited useful life

o Majority doesn’t wrestle with fact that the difference with precedent where subscriber list was for a defunct business

o Sets forth a new vision for deductibility intangibles/goodwill (this is pre-§197)

o Dissent Failed the court’s own test Critical of changing the goodwill definition without addressing

explicitly o Bottom line; TP wins, but confusion of where the law would go, Congress

steps in with §197 which includes goodwill as a depreciable asset over 15 yrs as straight line

Most TP are happier, but this covers some assets (such as patents) that were already depreciable which now must be recovered over 15 years (since it is swept under §197) instead of some shorter period

– Next question: How much are you going to be able to depreciate?o Hypo: Buy a machine for $200 in yr. 1, you hope to be able to depreciate

the total $200o What if by year 3 it has appreciated to $500? No change, even though it

has gone up in value doesn’t change the $200 you get depreciateo What if its increase in value b/c of inflation? No change, we don’t adjust

for inflation. If we did this we would also have to adjust liabilities by inflation too, so TP might not win out in the end.

– What would be the right amount/pattern of depreciation? o The ideal system would calculate how much of the productive value is

being used up year. o Chalk generates $1000/yr per year for 3 years at the end of year 3 it is

dust.o How much would pay for the chalk if interest rates are at 10%?

Will not pay full $3000o In yr.1 what would pay for that year’s income?

So year 1 you would pay $909 for the $1000 at the end of year Also would pay $826 for $1000 at the end of year 2

Page 75: Tax - Harvard Law · Web viewAmount Today/Interest Rate/# yrs into the future/total amount in x yrs 2nd way to think of it: ... Alternative: don’t deduct the repayment in year 2,

Pay $751 for $1000 at end of year 3 (if I had $751 today it would be $1000 in year 3)

Adding this us, the Total you pay $2486 for the asseto AT the beginning of year 2, the chalk is worth 1735 (909 + 826), the

difference from original price is how much of the chalk I have used of productivity/the depreciation ($751), etc.

o She will never ask us to calculate this o Point is we could define real economic depreciation but it is taking a lot of

work o Baseball Player Machines

Generate less income from year to year Yr: 1000, 500, 250 = $1750 Today this is worth $1510 Beginning of yr .2 worth $661 Beginning yr 3 $228 All depreciation adds up to a total depreciation of $1510,

which is right o For Asset 1, the depreciation is going up over time o For Asset 2, the depreciation is going down over time o Straight line for Asset #1 is a TP benefit, because they can deduct more up

front o But notice for straight line for Asset #2, understates your depreciations

upfront, TP is not happy about this– In the Code, we don’t ask you to figure out the exact life or exact income stream so

you can produce exact income numbers– The system has become more and more standardized over time

o Recovery Period according to the statute and reg for different types of assets we will put you in different assets and we determine different time periods you can take your depreciation

It is usually very generous, shorter than the real expected life of the asset

o Depreciation Method §167/§168 Way the code is going to match depreciation expense with the

income over the defined period of years For intangibles under §197 or §167 it is straight line, which

means equal/pro rata You also can elect straight line for many assets The other option is double Declining Balance, the one TP

usually prefers

Ex. Cost $2000, 5 yr. life, Double Declining Balance Method

5 Year Life Cost / Basis Straight-line (20%) Double Declining1 $2,000 $400 40% * $2,000 =

Page 76: Tax - Harvard Law · Web viewAmount Today/Interest Rate/# yrs into the future/total amount in x yrs 2nd way to think of it: ... Alternative: don’t deduct the repayment in year 2,

$8002 $1,200 $400 40% * $1,200 =

$4803 $720 $400 40% * $720 = $2884 $432 $400 50% * $432 = $2165 $400 50% * $432 = $216

1) Figure out straight line percentage and then double it, in this case you would be recovering 20%/year,

a. For year 40% of $2000 which is 800b. Your new basis in yr. 2 is $1,200, you adjust your basis §1016 tell us we

need to do this, 40% of 1200 is 480c. In yr.3 your basis is $720, depreciation is 288d. Caveat: If you just did 40% forever you wouldn’t get rid of your basis, so

the caveat is switch to straight line actually gives you more recovery , in this case this happens in yr. 4 (50% v. 40%)

e. This gives high depreciation to begin with, this is good for TP versus straight-line, is too generous? Well we don’t know because we are not calculating actual economic depreciation. Whether or not it is really incentive is whether it is better than economic depreciation.

– For an income tax system we would want true economic depreciation to maintain accurate picture of consumption + accumulation

a. An immediate deduction is inaccurate for IT purposes, but it matches consumption tax system (for §179 this is closer to CT system)

– We had something even better than immediate deduction ACRS (Accelerated Depreciation) + Investment tax credit 1981

a. People would actually earn money through the tax systemb. Congress did not intend it and repealed it in 1982

– IT system: Not taxing unrealized appreciation: bada. Not deducting for capital expenditures: good

– CT: Not taxing unrealized appreciation good (it is savings)a. No deduction for capital expend. Bad

Sale of Depreciated Assets o You buy an asset for $100, and you get $25 depreciation in yr. 1 at the end of yr.1

you sell it for $90, AR $90 B $75, Gain =$15 o Seems to all work out, your return shows $10, which seems to make

perfect senseo But the problem is that the deduction is for regular income, but the gain

might be capital gain taxed at a lower rate, the deduction is more valuable than the offsetting gain, you get a great deal here. What is it that we are going to do? We want to equalize the tax rates this is done under §1245 and §1250, covers depreciation recapture

o Again the way that depreciation system interacts with other systems in the code must be kept in mind

Page 77: Tax - Harvard Law · Web viewAmount Today/Interest Rate/# yrs into the future/total amount in x yrs 2nd way to think of it: ... Alternative: don’t deduct the repayment in year 2,

4. Transfer of Tax Incentives– In order to take advantage of Depreciation Deductions (or any deduction) is not worth

anything unless it reduces income, it is worthless if you don’t have the income to offset

– The simplest way to get value of this would be to sell it, but this is not allowed, so you try to structure transactions so the goodies can be shared

o Classic example: Airlines, high depreciation amounts they couldn’t use, they would have a company with a lot of cash, they would buy the plane and lease it back to the airline, the idea is you have taken a deduction that no one was using and now cash co. is

o Run the risk of IRS saying that cash co. doesn’t own the airplanes, in reality it is airline owns the plane

– Congress came out with guidelines, to regulate these type of set-ups, they didn’t want to totally oppose the self-help to airlines

– Safe-Harbor Guidelines, as long as follow certain guidelines can do this, but public outcry so Congress had to pull back, so IRS will still evaluate whether it is real lease or a sale

5. Depletion– Parallel for Depreciation for natural resources – Two methods for depletion:

o Cost Depletion $1000 cost of oil units/100 units of oil = $10/unit §1.611-2

o Percentage Depletion §613 TP gets deduction = specified % of TP’s GI from depleted prop.

During ye. Not exceed 50% of taxable income (TI) $1000 Gross income of uranium mine (22% of Gross Income),

trying to $220 of depletion Gross Income – others costs = $500 TI can take the $220 because

it does not exceed 50% of taxable income If gross income – other costs was $400, you can only depreciate

$200 Remember, unlike cost depletion, you can keep on deducting

every year even after you have recovered your cost! Explicit evidence that this is the result of lobbying power and

they were trying to encourage exploration of mines

6. Preparatory Expenses– Traditionally, prior to §195, all preparatory expenses were capitalized, which would

basically meant you couldn’t get it back until you sold your businesso Even if they look like §162 expenses, this is before you actually have the

business or trade so can’t immediately deduct o People would want to say that it is part of current business versus

creating a new business, characterize newer ventures as merely an expansion

Page 78: Tax - Harvard Law · Web viewAmount Today/Interest Rate/# yrs into the future/total amount in x yrs 2nd way to think of it: ... Alternative: don’t deduct the repayment in year 2,

– §195 lets you recover preparatory expenses over 60 months period with straight-line depreciation, but can only start deducting once your business/trade starts, if your business never works out you are out of luck, not clear you will get a recovery

o Would have to qualify for immediate deduction under §162, if it is an item that would have to be capitalized over a certain lifespan (you buy a printing press as preparatory) that rule applies instead of §195

Only expenses that if you were in the trade or business you could deduct under §162, its not just anything you spend money on setting up your business

– Being an employee can be your trade or businesso Hypo, lawyer looking for a new job and incurs some cost, laywer would

argue already in trade or business, this is simply cost of business of being lawyer, IRS has approved immediately deducting these costs

– What about a law student headhunter’s costs? Seems perfect fit for §195. But look at §195 (c)(1)(a)(ii), “active” trade or business requirement, this means more than being an employee, cannot prepare to be an “employee” so a law students headhunter’s not deductible under §162 or §195

o What if your employer pays for your ticket, they can take a deduction, you might worry about whether it is income, since if you paid for it by yourself it would not be deductible. It turns out the IRS has addressed in revenue ruling, which says if firm pays for it, no tax consequence. There is unevenness here, why would the IRS be comfortable with this?

If employer is forking over money, it is probably really business related. If you paid you might fly anywhere and get someone to meet with you.

– Sharono TP trying to get deductions for law school, bar review, cost of sitting for

the bar Court said can’t deduct law school or bar review but can capital

cost of sitting for the bar

C. Business/Personal Bordeline1. Childcare

a) Smith– Want deduction for childcare and court says no– Court says that it is personal expense, but indicates there is tension between

business and personal borderline o Depends how you frame the question

Is the expense from “working” or for “having children?” Answer relates to whether you think it is business or personal expense.

– Court rejects but-for test, because of slippery slope, but for food you would not be able to go to work – too broad because we know we don’t want food to be deductible

Page 79: Tax - Harvard Law · Web viewAmount Today/Interest Rate/# yrs into the future/total amount in x yrs 2nd way to think of it: ... Alternative: don’t deduct the repayment in year 2,

b) §21– Note that congress did not change §162, did not change our mind whether child care

is personal or business expense but did provide some relief – Remember that you are not taxed for providing your own childcare, imputed income– This put pressure that you earn more outside the home because you are taxed on that

income, this gave a disincentive to go outside the home, §21 alleviates the dichotomy – §21 credit 35% equal to childcare costs (employment related expenses) which

decreases by 1% for every $2,000 you earn over $15,000 but never below a 20% credit (why not eliminate totally as you go higher)

o What is the difference between credit/deduction? Credit reduce your tax bill $ for $, deductions are reduction of your

income Deductions are dependent entirely on your tax rate for their

value Who might prefer one over the other? Wealthy taxpayers

get more out of deductions. The higher your tax rate the more valuable the deduction is.

o What is it that qualifies as employment-related childcare expenses? Must be under certain age, also some care for older disabled

children Overnight camp does not count Look at the regulations having a maid does qualify, but

chauffer/bartender are not – Hypo: TP earns 80K, spends $15K on day care (1 child)

o Only get 20% credito Can only $3K for one child

This is important, there is a cap, cannot get a credit of 20% of $15K

o So credit is worth $600o Doesn’t 3000K seem low? o What if you have 5 kids?

Only get 6K, so now can only get a credit of $1200 20% stays the same no matter how many kids you have

c) §129– Up to $5000 of salary excludable from income for dependent care

o Same requirements of care in §21– Has to be through employer’s structure or plan and has to meet various requirements

(nondiscrimination), usually part of cafeteria plan o Does not mean employer has to provide it

– What is it worth?o GI 100Ko And 40% bracket o 40% * 5K = 2K

Instead if you went with credit you would have value of $1200, you want the 2K, but you really want both, but you can’t, there is a specific interaction between §129 and §21

Page 80: Tax - Harvard Law · Web viewAmount Today/Interest Rate/# yrs into the future/total amount in x yrs 2nd way to think of it: ... Alternative: don’t deduct the repayment in year 2,

Look at §21 (c), which says that your qualified expenses must be reduced by whatever you exclude from income under §129 (if you took $5000 deduction under §129 you have nothing left to credit if you have 1 child ($3000K))

So note that you can use both, if you don’t exclude more (i.e. an employer cap) than your qualified expenses

Important it is the deduction that you subtract from the qualified expenses not what it is actually worth to TP

2. Clothinga) Pevsner

– In order to be deductible as business expense, clothing musto Be requiredo Not adaptable to general useo Not so worn outside of work

– TP argues that for her it is business expense, subjectively it is not adaptable for general use

– Court says that an objective is more appropriate for adaptable for general use o Administrability issues o Equal treatment of workers

– Same sort of arguments made in Reginald Turner – What if you are tennis player are costs of tennis sneakers deductible?

o A problem because they are objectively adaptable to general useo A mechanic’s overalls probably more likely to be deductible (especially if

it has logo) – What about an allowance by law firm to buy suits? Not deductible. – Better examples, gorilla and astronaut suits. – Still need to meet the requirement that you don’t wear it outside of work

3. Traveling and moving– If you work in Boston and travel to New York, your transportation is absolutely

deductible. How about your lunch? Yes, as long as you are “away” from Homea) Correll

– In order to deduct meals, sleep or rest must be required (remember travel costs are still deductible)

– Court agrees that this is a good interpretation of “away from” language in 162 (a) (2) which allows deductibility of meals and lodging

– Generally an overnight rule to deduct lodging and meals– Commuting costs have never been deductible even if you live far away

b) Flowers– TP kept his house in Miss. when he took a job in Al., he wants to deduct his

travel/commuting costs– T. Ct. says no deduction because you get his “home” is in Al., home=business– Ct. of Appeals says home = house, allowed to deduct

Page 81: Tax - Harvard Law · Web viewAmount Today/Interest Rate/# yrs into the future/total amount in x yrs 2nd way to think of it: ... Alternative: don’t deduct the repayment in year 2,

– Sup. Ct. no deduction but does not focus on definition of “home” in §162, their view is if there is a business need for this expenditure, they say in this case it is more personal choice. Which decisions are personal and which ones are really for personal

– Congress presumes that you will live near where you work – Revenue Ruling

o Construction worker you go from job to job, you might not have a home for tax purposes, so never get a deduction

– What about employment that requires carrying tools? Court says no. Police officer needs to take a train outside of N.J. because doesn’t have a license to carry in NJ

– How much will we allow you deduct for meals? We are concerned because you will always eat no matter what. You are always getting some personal benefit out of it. – Hypos:– Con Law prof. Goes on leave to teach semester at another school– TP would like to deduct cost of living at other school (lodging/food)– Argue that it is temporary and you are away from home– Generally Court has agreed as long it is under 1 year, you can be temporarily away from home up to a year– What if you have a job in Boston 5 days and moonlight in another city 2 nights/week– Assumption is that main workplace/home is Boston, Providence is your minor post– Transportation is probably covered– Also want to argue for lodging and meals argue that it is away from home – Rules generally try to figure out your main workplace and which one is your secondary and then decide which costs are deductible (look at pg. 28 of handout) – You are a married couple, you work in Boston and your wife works in D.C., TP would like to deduct cost of D.C. travel, house and meals, probably won’t be deductible. The analysis from §162 is done for a person by person basis not as a marital unit, wife lives and works in D.C. no deduction, travel back and forth is totally personal. – Hantzis:– 2L tried to get a summer job in Boston so she took a job in NY, her husband stayed in Boston she wanted to deduct her lodging meals in NY and traveling back to Boston– IRS says no, home is where your business is, she lives and works in NY and she goes to Boston for personal reasons to see her spouse – Taxpayer says her home is Boston, her NY job is temporary – Though not a temporary assignment – Sup. Ct. says that won’t ask definition of “home” but whether she is required to have two homes for business or personal reasons: in this case it is clearly personal so no deduction – Concurrence says home is Boston, but still must have business reason for two locations

Page 82: Tax - Harvard Law · Web viewAmount Today/Interest Rate/# yrs into the future/total amount in x yrs 2nd way to think of it: ... Alternative: don’t deduct the repayment in year 2,

– But why does it matter how home is defined? It is important in the case that she can deduct, still need to figure out which one is “away from home.” You get to deduct those expenses away from home. – What if they gave you a part-time job when you got back to Boston? Still tough to say that while you were in NY-Boston was your home, but see the new argument that emerges. – Commuting is getting you from work and back at home but if you additional stops (dr. that goes to multiple hospitals) you can deduct as transportation costs – You are a fire chief. You go from your house to a fire site in a special fire chief’s car.– Is this gas you use deductible?– You could argue that once you get into the special car, you are essentially at work and this not commuting. – IRS counter is that you are just driving from home to work

4. Moving Expenses– Employer pays for your moving expenses from Boston to Pitt. Under §132 (a) (6) and (g) – What kind of expenses? “Qualified” expenses which are described under §217. – Under §217 outlines what can be deductible if your employer does not pay them– Moving you, your goods, and your family from 1st location to 2nd location– Restrictions:– Got to be at least 50 miles away from where you were working before – Concerned about abuse, moving for personal reasons – Continued employment at the new location for certain period– Can’t just get some business to hire you for 2 weeks and then your moving expenses are deductible– §82– Covers reimbursement and the fact that you can’t exclude and deduct – §217 used to be much more expansive for what it covers

5. Travel & Entertainmenta) Sanitary Farms

– TP, dairy farmer, went to Africa with his wife for 6 months wants to deduct as advertising cost of the business– Court allows it, says that the primary purpose was indeed advertising, they got a tremendous amount of exposure for relatively little cost– Shows the court is really willing to look into the facts – Court is not going to let the fact that they had fun/liked what they were doing will not control whether is personal/business

b) Cohen– Play producer and entertained everybody in the theater business, problem was he kept no records – Court says court should just estimate– This gives some bad incentives to TP’s – We get §274 in response which places lots of limits on what you are able to deduct, one of the big ones you can only deduct 50% of food and entertainment

Page 83: Tax - Harvard Law · Web viewAmount Today/Interest Rate/# yrs into the future/total amount in x yrs 2nd way to think of it: ... Alternative: don’t deduct the repayment in year 2,

– First you have to pass the §162 threshold (ordinary/necessary expense) and now you have to pass more hurdles under §274 in addition to the 50% limit –– Hypo:–– TP in 70% bracket, buys hockey ticket ($50/each)– Before §274, to bring client total cost is $100BT, $30AT, it is cheaper to go with a client than buy yourself – After §273 (a) (1) (A) – If it does only $50 deductible, saves you $50 x 70% = $35, so it costs $65 which is not cheaper than going by yourself – §274 is a response– 274 (n) caps at 50% deductions of food/entertainment expenses – §274 (a) (1) (a) no deduction for entertainment/food unless connected with/clients prior or after a real business discussion (qualifies §162) – Hypo– Take client to a bar, have a drink, and talk about business– Yes, deductible. The meeting is actually happening during the entertainment, so don’t even have to worry how close in time. – Even now, you still only get 50% of your deduction.– Suppose you take client to a bar and discuss sports: No. – What if you take client to rock concert and discuss business? Seems fine, no difference than bar hypo. – What is the problem from IRS’s perspective? How do we really know whether you discussed business or how long you talked about it? – §274 (h)(2) – Physician goes on cruise with a talk on different medical topic every day– (h) deals directly with cruise ships, presumably there are a lot conventions on cruise ships, restrictions on:– Where it launches – looks like a protectionist measure – Substantive- directly related to your business– Cap the amount of deduction at $2K – Also are reporting requirements like attendance of seminars, etc. – §274 (h)(7) – Regards §212, under which you cannot claim seminar, conventions for profit-seeking ventures (as opposed to trade/business), why is this?– Concerned about investing, lots of people could claim they are doing this, so there would be a lot of “how to invest” conferences taking place – What if you work for law firm in Boston and fly to Chicago and you spend money on food, does §274 (n) cap apply? Yes! It does not have to be in entertainment context, food is always 50% cap. But there are a couple of exceptions (food that qualifies for de minimus)– Facilities – Hunting lodges, etc. IRS will not allow deductions for the facilities themselves. Under IRC § 274(a)(2)– Moss

Page 84: Tax - Harvard Law · Web viewAmount Today/Interest Rate/# yrs into the future/total amount in x yrs 2nd way to think of it: ... Alternative: don’t deduct the repayment in year 2,

o Concerns deductibility of law firm meals when the firm ate together everyday at the same local restaurant.

o This is a debate about §162 and §262, have not even gotten to §274, have not even made it past the first barrier

– Rules:o Should only able to deduct if it costs more than you would have paid (but

this is just a piece of thinking) o Should have clients there o Maybe one or two meetings is ok, but Congress did not intend for you to

get a free lunch every single day – Hypo:– Lawyer takes client to lunch to discuss case – yes– Client takes attorney to lunch to discuss case – yes – Atty takes client to lunch to retain goodwill – NO, need to look at the Regs. As a practical matter, as a lawyer, will insert some business conversation. – Partners take associate to lunch to discuss future w/firm? Yes, if you did once. But if someone is getting a free lunch every day will be disallowed. – Partners take associate to lunch to discuss ongoing case? Yes, same as above example. – Two partners – lunch once a week to discuss ongoing case? Close call, hinges on how much is too much. Don’t have a clear sense from Moss.– What are other ways we could curb/cap expenditures?

o Flat $$ limito Why choose the 50% cap?

6. Residential Property– A long time IRS would lose in these cases, so Congress added §280(A) – No deduction of individual taxpayer for payments re use of dwelling which TP uses as resident (exception interest, casualty, taxes)– Special Rule: Can deduct if part used exclusively on regular basis as a principal place of business– Place of bus used by patients, clients etc. meeting w/TP in normal course of trade/business – Or – In case of separate structure not attached to dwelling unit in connection w/TP’s T/B– If TP is ee above must be exclusive use for convenience of employer– Hypo:– Ring does some grading at her home, can she deduct? No, it is not principal place of business.– What if she does some tax consulting? Yes if clients come to meet at her house. (issue of what it means to “meet,” videoconferncing, etc.) – Drucker

o Musician who practiceso IRS and Tax Court say Lincoln Center is principle place of businesso 2nd Circuit says this is really for convenience of employer, you have to

practice 30 hours a week and they give you no place to do it, so can deduct

Page 85: Tax - Harvard Law · Web viewAmount Today/Interest Rate/# yrs into the future/total amount in x yrs 2nd way to think of it: ... Alternative: don’t deduct the repayment in year 2,

under (a), The room in his house was determined by the court to be his principal place of business.

– Solomono Dr. works at multiple hospitals and also had an office at home for

administrative work, like Drucker hospital did not place to do administrative work

o Sup Ct. is figuring out principle place of business on 2 prong testo Time spent in various locationso Relative importance/type of work being done in each placeo Concurrence thinks it should just be where the core of business takes

place. o Congress is upset with this decision and responds in 1997

– §280 (A)(c) – Expands the meaning of principle place of business to include space if it is used as administrative functions when there is no other place to do it (covers Solomon)

7. Hobbies– What are we worried about giving losses to corporative executive losing money on his farm? Might be concerned that it is just for fun and seems to prioritize entertainment of wealthy who are likely to have these type of hobbies.

a) Bessenyey– Raised horses, has big expenses and after 5 yrs later tries to expense them, saying they were profit-seeking activities – Tax Court denies the deduction– Court of Appeals says that must look at whether TP has the intention to seek profit, but affirms Tax Court decision – TP seemed bored by financials, books, just interested in her horses –– §183 comes in after Besseney – §183 (a) general rule – no deduction for losses if you are not engaged in profit-seeking activity (overstated, though nothing new)– §183 (b) will allow deductions– (1) Those that are allowed elsewhere in the Code (casualty loss, interest, etc.) – (2) Deductions up to amount of profit/income [not new, applied in Bessney] (after covering expenses in (b) (1)) (this is stacking rule)– Similar to gambling losses, intent should not matter to the extent you actually generate income, but beyond that intent comes into effect – Hypo for stacking part of the rule– $100 hobby income– $120 hobby expenses– $10 of them related to your hobby but also a casualty loss– the most you can deduct is $90– TP would like the reverse, so they can deduct $110, the statute says no, you have to stack your other deductions first – §183 © defines for profit

Page 86: Tax - Harvard Law · Web viewAmount Today/Interest Rate/# yrs into the future/total amount in x yrs 2nd way to think of it: ... Alternative: don’t deduct the repayment in year 2,

– §183 (d) assume that if profit if you profits in 3 of 5 years than it is assumed to be proft-seeking, this seems to get around the intent we’ll kick you out of the hobby rules to §162 or §212 – Special rule for horses, only need profit in 2 out of 7 years – Encourages you to recognize profits in certain years to create the requisite number years of profit (defer expenses etc.) – Today Bessney would probably come out the same (IRS allowed deduction up to income) did not meet the 2 of 7 requirement

8. Legal Expenses– Suits can have both personal/business components

a) Gilmore– In the context of a divorce, TP is arguing over the protection and/or division of his business investment and assets. TP wins and wants to deduct his legal expenses under IRC § 212.– The Court did not allow the deduction.– Applies the origin of the claim / litigation test, the origin was personal (divorce)– This is a very different test than the “result/effect of the litigation” test. The Court viewed this test as (1) too expansive, and (2) unfair to TPs because it would treat TPs differently depending on their assets.

b) Patrick– Different because there was no litigation it was other legal fees associated with allocating assets and property– Court says it is not just actual litigation fees, it still came from a divorce, which is personal– Dividing up assets is always part of divorce – A and B, both sued for Car accident, A stands to lose his home, B stands to lose his businesso If we went with effects test, A and B would be treated differently o Under claims test, A and B are treated the same

– What if you go to France and meet a future business partner can you deduct cost of trip? No origin of trip was personal.

– What if wife/husband get divorce, settle that wife will get 25% of business profits. 5 years later wife sues company over accounting procedures, can husband deduct these legal fees? Yes, we will not go way far back to find personal origin.

– Two co-workers at law firm and they get married and then divorces. Can you deduct these fees?

– What if wife started harassing your clients (no divorce) and you try to get a restraining order? Probably deductible.

Page 87: Tax - Harvard Law · Web viewAmount Today/Interest Rate/# yrs into the future/total amount in x yrs 2nd way to think of it: ... Alternative: don’t deduct the repayment in year 2,

– Point is, origin of claim may sound clear cut but you can see where gray areas emerge.

– Remember §212(a)(3) – can always deduct preparatory tax work etc. This creates incentive to bill more hours for taxes than for divorce, for the extent that it is tax-related issues you can rely on 212 even if it is associated with a divorce.

9. Education Expenses– Covered under §162 (trade and business) and plus some additional rules added in the 90’s – Look at §1.162-5, when as a trade or business matter education expenses can be deducted

o can deduct education costs if it is to: maintain/improve skills required by TP in employee

Trade/Business Meets express requirements of TP’s employer or is imposed as a

condition to keeping job And NOT:

Minimum education requiremento Requiring teachers to complete their masters at

nighto College will never be deductible

Lead to qualifying for new Trade/Businesso Regardless of whether you pursuit it, accountant

goes to law school at night, real problem with law degree can always lead to new trade/business

How about business school? How flexible we are on this is key.

Look at examples in reg. – Special provisions added in the last year §25A, §221, §222

o §25A Hope Credit

Per student credit – if you are paying for 3 kids applies to each student

How do we calculate?o Qualified Tuition Expenses (not room and board,

qualified institution and paid during that year, if student pays plus dependents )

o Credit equals 100% up to first $1000, plus 50% above $1000 up to $2000

o Maximum credit is $1500 Limits

o Can only use it for two yearso Has to be post-secondaryo Can’t be part-timeo Can’t be a drug felon

Page 88: Tax - Harvard Law · Web viewAmount Today/Interest Rate/# yrs into the future/total amount in x yrs 2nd way to think of it: ... Alternative: don’t deduct the repayment in year 2,

Lifetime Learning Credit (LLC) Per Taxer, only one LLC per tax return 20% x qualified tutition + related expenses paid by TP

during year, 10K Max 2K max credit

o LLC/Hope Coordination If Hope take for a student then the expenses for the individual can’t

be included in LLC, can’t take Hope and LLC for the same student, LLC covers broader range of educational experiences but only covers one student

Seems language sounds like if you qualify for Hope you have to take

Final §25A Credit = Hope + LLC But reduced based on Adjusted Gross Income §25A (d) TP mod. AGI – 83K joint (41K single)/20K joint (10K

single) = reduction in credit/total possible credit Reduces credit as your income increases, make more than

51K you get no Hope/LLC credit whatsoever– Ex. C + K have AGI of 93K, have 3 daughters 1) Annie freshman tutition 14K 2) Marie Sophomore 14K 3) Lilian – junior 14K

o Annie Hope of $1500

o Marie Hope $1500

o Lilian Not eligible for Hope because she is junior C + K can take $2000 (20% of 14K maxed out at 2K)

o They are looking to get 5K credit, but first they have to apply the formula (93K – 83K/)20K = ½ ½ = reduction in credit/total possible credit = $2500

– §221 o Comes in 1998 and basically trying to give deduction for interest you pay

on student loans ($2500 limit) o It is above the line, don’t face various restrictionso Same limitation formula: Modified AG – 100K (50K)/30L (15K) =

reduction/max deductiono Has to be debt of the taxpayero Can cover room & board

– §222o Comes in 2002o Above the lineo Deduction for qualified tuition expenses paid for during the year

(maximum of $3000)

Page 89: Tax - Harvard Law · Web viewAmount Today/Interest Rate/# yrs into the future/total amount in x yrs 2nd way to think of it: ... Alternative: don’t deduct the repayment in year 2,

o Limit based on income but more simple: If AGI is greater than 130K (joint) or 65K (single) then you get no deduction (not a phase out like above)

– What is interaction between §222 and §25A, says you can not get a deduction for an individual if you got a §25 credit. If you have used the student under §25A, can’t use it. This other expenses and people who were not covered under §25A. – Can’t rack all of these up at the same time, so not quite as generous as it appears to be without thinking about the interaction

D. Personal Deductions1. Interest (look at other notes)

– Three different types of interest expenses:o Business interest (§162, deductible)o Investment interest (§163 (d) interest can be deducted up to amount of

investment income) another way having an investment v. business is disadvantageous

Typical investment activity: buying stocks Borrow money to buy stocks Two types of income:

o Dividendso Appreciation

But this does not count as income until you sell, so this is mismatch we are concerned about, so we only deduction up to the amount you actually realize as income

o Different ways to approach Trace (this is the primary approach) Stack Pro-rata

o Personal interest can deduct: Interest up to $1,000,000 of home mortgage debt for up to 2 homes 100K on home equity loan

Monday – Thru C5Tuesday – Thru 4B Skip III C & D

– Think about the different ways you see Congress/Court drawing the line between business/personal

o Motivationo Categorizationo Bright line tests

How does §183 fit into the pictureo Origin

– Why do we use different answers in different cases? Historical doesn’t explain it all. – Business Interest covered under §162, investment interest can only deduct up to the amount of investment portfolio gains

Page 90: Tax - Harvard Law · Web viewAmount Today/Interest Rate/# yrs into the future/total amount in x yrs 2nd way to think of it: ... Alternative: don’t deduct the repayment in year 2,

– Prior to 1986, could deduct all personal interest but now it is limited to mortgage, home equity and education– How do we characterize the money? Usually, through tracing but difficult because money is fungible.

2. Tax Arbitrage– Arbitrage: Tax payer trying to combine deduction for interest and excluding the interest income (borrowing to buy tax-exempt municipal bonds)– Hypo: Borrow at 10%, to buy municipal bonds paying 10%; Tax Rate 30%

Pre-Tax Tax effect After taxReceipts 10 0 10 Costs 10 (10) 7

0 (10) 3– Once you add in the tax effects, this is netting you $3 after taxes, is this bad?

o One Concern: Who is doing this? Mostly going to be sophisticated, rich tax payers.

o Is this the purpose of the rules of tax-exempt bonds? Maybe this does not further the goals.

o Is this a realistic picture? What really goes on in tax-exempt bond market? One thought: this is a great deal for a moment in time, but in the

end tax-exempt bonds will adjust downward to 7% interest. The Market will adjust.

Pre-Tax Tax effect After tax

Receipts 7 0 7Costs 10 (10) 7

0 (10) 3

– Where does Congress wind up on this debate? §265 (A) (2) says there is no deduction for interest on debt to buy/carry tax-exempt bonds.– Still begs the question how to you judge whether you should be able to combine two provisions of the code to your advantage. – Hypo: Have $500 cash and want to $600 oven, $300 stock, $300 car, willing to borrow $700, how do you want allocate your loan?

o Business loans are treated the best and personal loans the worst. Put the maximum cash towards car and maximum loan to oven.

Business Investment Personal Alternativeoven ($600) stock ($300) car ($300) $700 total debt

all loan $200 cash, $100 loan all cash $350/$175/$175o Work with the idea that money is fungible to get around the rules limiting

interest deductions 3. Shams

a) Knetsch– Trying to get a tax deduction at now cost– Δ is buying $4MM annuities at 2% at the same time borrows $4M at 3.5% to pay for annuity (from the same person)

Page 91: Tax - Harvard Law · Web viewAmount Today/Interest Rate/# yrs into the future/total amount in x yrs 2nd way to think of it: ... Alternative: don’t deduct the repayment in year 2,

o Annuities simply go up in value without paying out cash, so not being taxed currently

o He borrows against the annuities (increase in value) to get cash to pay the interest on the loan and he wants to deduct

o He closes the transaction by exchanging the annuity for the debt and walks away owing only about $1,000

o Getting current deductions and delayed inclusion by 3 or 4 years, is this enough (see below)?

– Court is questioning whether there is actually any debt– What makes it really valuable TP anticipated taking the deductions at full ordinary rates but getting the income at C.G. rates, this is what makes the transaction valuable. – Ordinary tax rate = 90%

Before TaxAfter Tax:

25% C.G. rate (on inflow)

After Tax:90% C.G. rate (on

inflow)Cash Inflow 308,000 [2.5%] 231,000 30,800Cash Outflow: interest (441,315) [3.5%] (44,131) (44,131)Cash Outflow: payment (4,000) (4,000) (4,000)Net Profit (Loss) (137,315) 182,868 (17,331)

– The strategy to get your income taxed at low rate and deduction at high rate is a very common technique– Court says it is complete sham and there is no debt and therefore deduction– Not just because the parties and amounts line up (can still buy a couch from the same person who sells you a couch) – Not a real annuity, no real investment because from the same party you are borrowing more than you will receive in return, you keep on borrowing against the annuity to pay off your debt

b) Goldstein v. Commissioner TP wins money in the “Irish Sweepstakes.” TP borrows money from banks at 12% and then buys bonds that pay 10% (borrowing

at a higher rate than the bonds are paying). Bonds are being used as collateral to the debt to pay for it. She is prepaying the interest in order to get a deduction to offset the sweepstakes

winnings. Different from Knetsch have different parties (borrowing from the bank and buying

bonds from someone else)

Before Taxes After Taxes (KG)Gain 10 +8Interest -12 -6Profit -2 +2 The Court found that, although this transaction may not be a “sham” like in Knetsch,

in this case, the taxpayer was solely focused on the tax benefits so deduction will not be granted.

Page 92: Tax - Harvard Law · Web viewAmount Today/Interest Rate/# yrs into the future/total amount in x yrs 2nd way to think of it: ... Alternative: don’t deduct the repayment in year 2,

Non-purposive activity test: her only motivation was tax avoidance which is outside the purposes of §163

What is the line in the Court’s test? What is enough to be “purposive”? 1 penny of profit? What if the deductions are worth way more than 1 penny?

These kind of cases combine rules about income not be taxed (deferral, etc.) and interest deductions.

4. Taxes– May not deduct your federal income taxes, obviously – What about state income taxes? Yeso Interestingly, because you can deduct them, if you are high-bracket tax payer, you don’t mind state income taxes too much because you get a very valuable federal tax deduction, this shifts a significant burden of state taxes to the federal government – Other big one is that you can deduct state property taxes– Whose tax bill? Who pays it. o If you pay someone’s tax bill just because you paid it, questionable whether they can get it too. Thinking of aging parents you offer to pay their property tax bill you can’t deduct it and neither can they. Could try to make into a gift and then they get the deduction, but might not be valuable since they don’t have income. Also, could join them as titleholders to the property.

5. Casualty Losses– §165 (c) 3, allowing casualty loss deductions for items that are entirely personal – Why have this provision?

o Government insurance? (Obviously you don’t get full coverage because you have to multiply the deduction by the tax rate to get its value)

o Fairness? o Income evaporated (consumption + accumulation), the asset does not

belong in your income (negative income) – What does it cover?

o Code says fire, storm, shipwreck, casualty, theft a) Carpenter

– Husband accidentally sent his wife’s diamond ring down the disposal; claimed a deduction on the decreased value of the ring (it was virtually destroyed).– Court allowed the deduction.

o Court takes a broad reading, analogizing to other unintended acts of nature b) Kennan

– Flushed the diamond ring down the toilet.– Court does not allow the deduction.

o May reflect court’s hesitation over lack of proof (in this case, unlike Carpenter, there was no destroyed ring).

c) White– Car door slammed on TP’s hand, knocking diamond out. Diamond was lost.– Court allows the deduction, have a medical bill for the hand and the setting for the diamond

Home Hypotheticals:

Page 93: Tax - Harvard Law · Web viewAmount Today/Interest Rate/# yrs into the future/total amount in x yrs 2nd way to think of it: ... Alternative: don’t deduct the repayment in year 2,

If you buy a house for 100K and it increases in value to 250K. When one sells it for 250K, one is taxed for 150K.

If you buy a house for 100K and it crumbles over time, and you sell it for $50. No deduction is given because it’s just consumption, and income = consumption plus accumulation. Whatever one consumes, one should not get a deduction for that. Plus no casualty loss here, there is no suddenness that the definition seems to require.

If you buy a house for 100K and the next day a highway is built next to the house, destroying the home’s value. You sell it at $50. We would not allow a deduction, even though it is unexpected and sudden,

because it doesn’t really fit our idea of “casualty” or a disaster even though it is sudden. We assume you consumed the value.

d) Computation1) House Case #1

Bought house for $50,000 - personal use causes the value to decline to $40,000. It is destroyed by fire. Value equal $0. What is your loss amount? $40,000 or $50,000?

FMV (before the event) - FMV (after the event). Then take that difference and compare it to the basis of the property. The lesser amount is the amount you get as your loss (aside from limits).

In this case, the right number would be $40,000. This seems fair because that’s how much the house was worth and we assume

consumption of the house took place. 2) House Case #2

Bought house for $50,000 - untaxed appreciation makes house worth $60,000 (and not because you added a 10K pool, which would increase your actual basis) .

It is destroyed by fire. Value equal $0. What is your loss amount?

Decline in value ($60,000) v. Basis ($50,000). Therefore, the lower of these numbers ($50,000) is your loss.

You are basically capped at basis in terms of deductions. The appreciation was not taxed, so you should not get a deduction for that

appreciation, it was never really yours since you never paid taxes on it.

Car House RingLoss $7100 $50,100 $60Floor ($100) ($100) ($100)

$7,000 $50,000 $0

– TP hoping to get casualty loss for each of these, can he (assume no appreciation problem)?

o Look at §165 (h) (1) cannot claim losses below $100 ($100 deductible)o Now look at §165 (h)(2) only get deductions for casualty losses exceeding

10% of AGI, let’s assume that he has AGI of $100,000 o So only get a $47,000 deduction ($50,000 + $7,000 - $10,000)

– Why the 10% cap?

Page 94: Tax - Harvard Law · Web viewAmount Today/Interest Rate/# yrs into the future/total amount in x yrs 2nd way to think of it: ... Alternative: don’t deduct the repayment in year 2,

o Save revenue o Seek private insurance o Why not have flat amount?

This would be regressive 6. Medical Expenses

– §213, Saw this when discussing §104, get this deduction when you are not reimbursed for anyone for these expenses

o Hit by a bus: yes, deductible to the extent that they are hospital bill and not otherwise covered by insurance. Expenses must also exceed 7.5% of your AGI (same concept when we discussed to casualty).

o Plastic surgery: under IRC § 213(d)(9), not deductible to the extent it is not cosmetic surgery (i.e., for a real medical purpose).

o Acupuncture: not in the statute, but it has been ruled that acupuncture and Christian Science are deductible.

o Swimming pool: worry about the fact that increases the value of the home, if you can really document as medical expense to the extent it did not increase the value of your house you can take a deduction (Pool costs $15K and increases value of house by $5K, you can take a $10K deduction)

o Shopping therapy: not covered, according to the courts - it may have the ascribed medical effects but it seems too personal (Rabb case); but we are in the personal area, so must be something else as well.

a) Ochs v. Commissioner TP had to send his kids to boarding school because wife was sick Court says no deduction for sending kids to boarding school when they needed to be

away from the mother due to throat cancer. The majority views this as a family expense, from which the healthy children

derived a benefit. Even though doctor said it was necessary

o If the wife had been sent away somewhere the costs would have been deductible

The dissent wants a different test: Origin of the decision to spend this money; Whether it was done on doctor’s advice; and Whether it had a direct connection to some medical illness. Would deduct only from the time the kids would have been with the mother (e.g., not

from hours older child would have been in public school) What might have been deductible (sending the mom to a care facility), would have

been more expensive than sending the kids to boarding school. Worried about abuses, sunny vacations are necessary, too slippery to turn on whether

the doctor said it was good for you Another factor is social class, to argue that this person would not normally purchase

private schooling for kids, though this argument is ultimately rejected, it is important that court is considering this again

Page 95: Tax - Harvard Law · Web viewAmount Today/Interest Rate/# yrs into the future/total amount in x yrs 2nd way to think of it: ... Alternative: don’t deduct the repayment in year 2,

Rationales for these deductions:1) Unexpected, could be quite substantial.2) “Negative income.” We discussed this in §104 as well

If medical care just restores you to the place where everyone else is (i.e. healthy), then perhaps we should allow a deduction.

3) Government as insurer, subsidy.

– Think about the fact that you must have income to take advantage of this– Deduction is also closely tied to tax bracket, the more taxes you pay the more valuable it is – AGI Cap, on the other hand, is progressive

7. Charitable Contributions (p. 36-37, 38-40) – Covered under §170, Two questions:

o One question is the deduction attributable to you?o What is the tax status of the entity you give to?

– Why do we allow charitable contributions to be deductible?1) Encourage giving.2) Subsidy to qualifying organizations.

o But harder to justify for religious organization3) Shifting consumption.

o You are giving away your consumption 4) Reward organization with a public benefit.

– Impact of the federal tax rate on donations to HLS:

Federal Tax Rate “Cost” to Contribute $1,00070% $30030% $70028% $720

HLS and other charitable organizations therefore have an incentive to argue for high tax rates. Some studies suggest that this impacts giving.

IRC § 170(b): places limits on charitable deductions Basic limit is that you can deduct up to 50% of AGI. Justifications for these limits:

1) Timing concerns.2) TPs choose where to give. If we allowed you 100% deductions, the TPs have all

of the choice over spending of tax dollars and funding of various social activities.3) If we allowed unlimited deductions, the wealthy could give away all of their

current income and live off their assets, and this just looks bad.

Page 96: Tax - Harvard Law · Web viewAmount Today/Interest Rate/# yrs into the future/total amount in x yrs 2nd way to think of it: ... Alternative: don’t deduct the repayment in year 2,

o Everybody should be paying taxes to some degree 4) Eroding the tax base, the government is ceding power to taxpayers what public

services are desirable, we want to limit this shift in power to TP 5) Why not make some hard $$ amount? Every time you create a flat number, you

basically eliminate the incentive for wealthy people to give away their income also linking it to AGI you are ensuring that everyone is paying some tax

Eligible Donees:– IRC § 170(c) defines charitable contributions.– Can’t take a deduction if you give $5K to your favorite professor, §170 (c) not an individual in their individual capacity

Hypothetical:– Endowing a chaired professorship at a University.– You can have control over who gets it (such as, must go to a professor in Economics), but if you have too much control (i.e., naming the professor who gets it), the IRS will not allow this it starts to look like quid pro quo, though obviously not a bright line.

Dowell v. United States (10th Cir. 1977) (page 502)– TP wants to move to retirement home with religious affiliation. After she is admitted, she sent a “contribution,” which was requested and even close to expected. – Government argues that there was a quid pro quo (she got in retirement home in return for contribution).– Court rejects this argument on the facts of this case.

o The timing suggested no quid pro quo (she got in before she made the contribution).

o Nothing on paper that says she was expected to make a contribution.– This was probably not a good case for the Government to bring. The retirement home had good advice.

o The monthly fees paid by residents cover operating costs, but do not cover long-term mortgage costs, etc. This may lead you to believe that everyone is expected to make a contribution as well as pay the monthly fees.

– §170 (f)(7) need to have documentation for all charitable deductions over $250, documentation changes behavior in substantial way

– Hilla Rebay– TP makes paintings and donates them to charity. The TP wants to deduct the FMV. The fight is over what the FMV of these paintings are.– Court held that she could deduct the FMV (there is really no debate about the rule), but in this case, the TP did not carry her burden of proving FMV.

Page 97: Tax - Harvard Law · Web viewAmount Today/Interest Rate/# yrs into the future/total amount in x yrs 2nd way to think of it: ... Alternative: don’t deduct the repayment in year 2,

o The one sell she cited was possibly suspect (she sold the painting to someone who never before bought art and was a friend of her lawyer’s), and the likely value of the paintings was probably much lower than that one sell.

o Her basis in these paintings is her costs of materials, etc. which is probably pretty low (human capital, of course, does not add to basis).

– Why is allowing FMV deduction seem odd? The appreciation in value is never taxed unless we treat giving event as a sale which triggers a gain or loss. So it should not be yours to give away.

– Can it be better to donate then sell? Basis = $10, FMV = $110, Tax Rate 70%(1) She gets $40 benefit from selling (what’s left after paying $70 of taxes (70% on

$100 gain))(2) She gets $77 benefit from donating (70% of $110)(3) She gets $7 benefit from selling and then donating the cash (gain = $100;

donation = $110 to offset other income, get $7 for $10 “lost”)– The key here is that she is benefiting from the untaxed appreciation in case (2) but not in (1) and (3).

o Why allow this? Same reasons we allow deduction for charitable contributions in

the first place (e.g., encourage donations)– Concern

o That there will be inaccurate valuations – no evidence that FMV is otherwise because it hasn’t been sold

o Keeping significant amount of revenue from the government

Two basic kinds of property in our universe: ordinary income property and capital assets.– Ordinary income property (e.g. inventory, paintings you make) is always taxed at regular rates, but capital assets can vary:– Capital assets (e.g., stock, paintings you buy)

o Some are taxed at regular rateso Some are taxed at capital gains (KG) rateso Depends on how long you’ve held the property

– Ex. Stock (and not dealer); painting you bought.

IRC § 170(e)(1)(A)– To the extent you are donating appreciated property and to the extent it would be Ordinary Income there is no deduction for appreciation

IRC § 170(e)(1)(B) applies.– To the extent appreciation would be taxed at preferential rates, your ability to get a FMV deduction depends on what you give and who you give it to if it:

(i) Is tangible personal property: limited to basis unless the donee uses the property in its tax-exempt function, in which case you get a deduction for appreciation.

Page 98: Tax - Harvard Law · Web viewAmount Today/Interest Rate/# yrs into the future/total amount in x yrs 2nd way to think of it: ... Alternative: don’t deduct the repayment in year 2,

(ii) “Anything” to a private foundation: limited to basis (exception in IRC § 170(e)(5) for some kinds of appreciated stocks (because a few individuals have a lot of control, so the IRS is wary of giving them too much leeway)

– Giving up more revenue when you miss taxing the appreciation on ordinary income gains

Hypotheticals:1) Ford gives cars as gift to the Harvard Law School.

– Covered under §170 (e) (1)(A) no deduction for the appreciation, the gain would have been ordinary income so only get to deduct the basis (not that you don’t get anything)

2) Appreciated stock gift to the Harvard Law School.– This is a capital asset, so you look to 170(e)(1)(B). You want to deduct FMV. Are you limited? Because stock is not tangible personal property and HLS is not a private foundation, you are not limited.– Therefore, FMV deduction is appropriate.

3) Antique medical instruments that you want to donate to the law school or the medical school

– These are probably capital assets, so you can bypass 170(e)(1)(A) and go straight to 170(e)(1)(B). But these instruments would qualify as tangible personal property and therefore be subject to that limitation.– If you give it to HLS? Your deduction is basis, unless you can use the tools (e.g., products liability, law and medicine, patent law)– If you give it to the medical school? Your deduction may be FMV if you can argue that the instruments are relevant to the medical school’s teaching of medicine mission.– HLS and HMS are not private foundations, so that issue can be ignored.

4) Donating your memoirs, papers, documents, etc. to HLS.– These are probably ordinary income property and therefore would fall under 170(e)(1)(A). This is property you created (similar to a painting that you made).– Therefore, you can only deduct basis.

– What if you donate your time/services? Can’t deduct except to the extent you concur costs in performing services, why?

o Valuation problems (both value of services and monitoring how much time you spent)

o Imputed income – are you just doing this for leisure/entertainment.

– Why do organizations care whether you are in §170 (c)? Because people will be more likely to give to you if they can get an exemption by donating $ to you.

Hypothetical:

Page 99: Tax - Harvard Law · Web viewAmount Today/Interest Rate/# yrs into the future/total amount in x yrs 2nd way to think of it: ... Alternative: don’t deduct the repayment in year 2,

– Suppose you give $1,000 to a private school that discriminates on the basis of race.

o Do you get a deduction? No. Service has a view that these organizations are not qualified tax-exempts.

o This is what we saw in the Bob Jones case.

Bob Jones University v. United States (1983) (page 478)– School has various rules based on race.– The Code does not explicitly deal with any of this. The IRS is interpreting the code and issuing rules and regulations that they use to determine that this is not a charitable organization.

o Is this a good thing for the IRS to do?o IRS interprets into the statute important public policies.

What public policies? If the government could act this way? – You can look at this as a fight over who is a charitable organization, or as who should the government be subsidizing.– Remember that there may be a distinction between being a tax-exempt organization and being § 170 qualified (and therefore able to receive tax-deductible donations).

o In this case, the school is probably fighting because it needs § 170 status for donations.

What about donations to single-sex schools?– The public policy in this area is much more muddled.– It probably does not rise to the clear level of important public policy that surrounded the issue in Bob Jones.

III. Computation of the Tax– See IRC § 1.– Our rates are also progressive. What underlies this system?

1) The rate of taxes you pay increases with your income 2) Raise more revenues.

– Our rates are marginal and not average rates.o 0-36K you make is taxed at 10% no matter what your total income is,

36,001 – 89K taxed at 20%, etc. o If you had average rates, you would lose money when you went into

higher tax brackets. Example: your rate would change from 10% on your income to 15% when perhaps you earned just $1 more (owe $1,000 if you make 1 more $). A lot of people are not clear on how our system works in this regard.

o Basically, the first bracket is always taxed at a certain rate, no matter how much more you’ve earned – that’s how the constant amount of the tax is calculated.

– What are the arguments for flat tax?1980 1981 1987 2002

Max 70% 50% 28% 38.6%

Page 100: Tax - Harvard Law · Web viewAmount Today/Interest Rate/# yrs into the future/total amount in x yrs 2nd way to think of it: ... Alternative: don’t deduct the repayment in year 2,

Min 11% 11% 15% 10%Zero 0% 0% 0% 0%

– What is the tax on a single person earning total income of 50K, need to look at P. IX of code book

o $3,910 + 25% ($50,000-28,400)o Total tax = $9,310

– See Pg. 41 of the handout – Step 1: Gross Income - IRC § 61 – adjustments (deductions above the line deductions which has two important advantages, everybody can take them and they’re not subject to special restrictions usually trade or business expenses, (except business of being employee) alimony, moving expenses, education, really want to make sure these are accessible, limiting to itemizers would really limit who could use them) = AGI– Step Two Adjusted Gross Income - IRC § 62 and go to

Either: IRC § 63(b) - Personal exemptions - IRC § 151; and

o $3,050 one for yourself, one for your spouse, and each of your qualified dependents, set minimum amount people can earn before they have to pay taxes

o It phases out under §151 (d) start to lose your personal exemptions as your income gets higher (but only when you hit $209,250 for a married couple)

Standard deduction - IRC § 63(c) ($5,750 for standard), if you are married ($9,500)

o If everyone lists things individualized it would be, 70% of TP’s take the standardized deductions

Or IRC § 63(a) - Personal exemptions - IRC § 151; and Itemized deductions (everything below the line created

elsewhere in the code) o Mortgage interest is the biggest deduction for most

people, most people who have a mortgage find it better to itemize

IRC § 67 - Subject to 2% floor; and IRC § 68 - Subject to 3% phaseout

– Got to pg. IX, Table 3

How to think about §67 and §68 applicability1) All of your itemized deductions, first worried about §67, some of your itemized

deductions you can only take them to the extent that they exceed 2% of your AGI (similar to medical, charity, casualty, etc.), called miscellaneous itemized deductions, only get them to the extent as a group they exceed 2% of your income

o If your AGI is $100,000 you need to have more than $2,000 of expenses

Page 101: Tax - Harvard Law · Web viewAmount Today/Interest Rate/# yrs into the future/total amount in x yrs 2nd way to think of it: ... Alternative: don’t deduct the repayment in year 2,

o Charitable deductions, medical casualty losses are not included in this group

All of these have there own built in floors, no need to double the floors here

2) §68 covers a much broader swatho Once your income crosses a certain threshold you start to lose you a small

portion of itemized deductions (more than miscellaneous but NOT everything)

o Why are we doing this? To maintain a certain amount of $$. Keep your rate the same, but another way to increase the tax revenue.

o What doesn’t it cover? Obviously nothing above the line Also casualty, investment and interest, which are below the line

Remember: Always have to pay Social Security Taxes no matter if you owe no income taxes EITC tries to address this because tends to be regressive.

Earned Income Tax Credit (“EITC”) See IRC § 32.

If you have earned income and it is pretty low, you can, under certain circumstances, get a credit.

You can also get a refund of this credit (one of the few places in the Code where you can do this).

Why do we have this provision? Welfare v. Tax Refund: this provision can be viewed as an alternative to the

traditional welfare system. People who get this credit are already paying the Social Security (employment)

taxes, which tend to be regressive.

Hypo: Why we don’t take all deductions at once. TP’s have 70K of medical bills.

TP A500,000 GI400,000 Business Expenses--------------------------------100,000 AGI

TP B100,000 salary100,000 AGI

Both get to start to deduct medical expenses after $7500 of income If §213 applied to gross income, A would be treated worse than B (he could only deduct

costs over $37,500)

Page 102: Tax - Harvard Law · Web viewAmount Today/Interest Rate/# yrs into the future/total amount in x yrs 2nd way to think of it: ... Alternative: don’t deduct the repayment in year 2,

IV. Choice of Taxpayer A. Taxable Unit

– How should you group people: Spouses as individual taxpayers or as units? It matters because of the progressive system.

1. Marriage– Poe

o Community Property State, one spouse earning 40, they each reported 20 on their own tax returns, this at a time where each person reported their own return

o Court says this is allowable because it is community property under state law

– Handouto How do different household units get treated as we change the rules

Pre-1948 - income taxed on an individual basis, regardless of marital status. But note that D, in a community property state, is able to split (see Poe v.

Seaborn, above). E (noncommunity property state) is paying more taxes than C and D.

This looks like a problem, so Congress changes the rules in 1948. For tax treatment, we now treat all couples as if they live in a community property state.

1948-1969 - all couples allowed to split. The problem is now that A (single) is paying more taxes than B (married). To alleviate this problem, in 1970 we created different rate schedules for single

people. Under the new rate schedules, a single person may pay a little more than a

married individual, but is not as bad off as before. 1970-1982 - new rate schedules.

But this can create a “marriage penalty” for a dual income married couple. The married couple C is now worse off than if they had been 2 single As. To partially fix this, in 1982 we gave a deduction to the second earner.

1982-1986 - deduction for the second earner. Problem with this approach is that D and E now pay more taxes than C, even

though each couple has the same total income. 1987-Present - we get rid of the credit for the second earner because it didn’t work

too well. We are back to the 1970s approach. Back to the “marriage penalty” problem becomes bad for you to be married

You can file separately but not singly, can’t avoid the penalty

If you want to progressive system and you want married people to have community property and you want single and married people to be treated the same as individuals, can’t accomplish all 3 goals. This is what happened as the government tried to meet all 3 goals – they kept focusing on one or more to the exclusion of the others. It comes down to a policy problem. How to decide how to make changes:

Empirical look: how many families are in each position and the size of the penalties

Page 103: Tax - Harvard Law · Web viewAmount Today/Interest Rate/# yrs into the future/total amount in x yrs 2nd way to think of it: ... Alternative: don’t deduct the repayment in year 2,

Rank the goals Whether to give incentives to work (bad for B, D, and E) – empirical and policy

question Think about imputed income for spouse who stays at home (couple C has

expenses that the one-income families don’t have – childcare, commuting, less free time, etc.), so maybe changes should be made.

Hypothetical: Husband and wife earn $60,000 each in 2000 in taxable income. How would you figure out the marriage penalty, if any?

Compare the tax burden filing joint v. tax burden filing single. 2 Singles: IRC § 1(c) (see front for 2000 numbers):

Taxes = $3,910.00 + 25%($60,000-$28,400) = $11,810. (per person) * 2 = $23,620 (total).

Married filing jointly on $120K: Taxes = $22,280.50 + 28% ($120,000-$114,650) = $23,780

Therefore, marriage penalty = $160.50– Congress has reduced the marriage penalty by changing the marriage tax rates– Remember that D&E actually benefit from being married

a) Mapes v. United States (Ct. Cl. 1978) (page 798) Couple is unhappy with the marriage penalty. Claims that this is unconstitutional. Court goes through the analysis, but holds that the tax is not unconstitutional.

A person can choose whether or not to get married. A lot of married couples benefit from the tax structure. Rejects equal protection and due process arguments Doesn’t burden fundamental right to marry because it’s helpful to be married for

some people for tax purposes. Doesn’t burden women as a class by burdening their constitutional right to choose

a career by providing a disincentive to have a second income (and in a historical context, women didn’t work in a couple) because it’s helpful for some women, so it doesn’t burden all women as a class.

This is an interesting opinion that goes through a lot of the different effects of the tax.

b) Boyter v. Commissioner (4th Cir. 1981) (page 811) This couple tries to avoid the marriage penalty by divorcing in December for the end

of the year and then remarrying in January. The court holds that the sham doctrine will apply to this. If the divorce was a sham, it

would not be valid for tax purposes and they are married. This divorce is basically a sham (although the court does not say this). The court sends it back to the Tax Court to see if the divorce was really a sham.

Reminiscent of Knetsch. The letter of the law is followed, but is it a sham?

2. Divorce– Need to look at transfers of property incident to divorce

o Under divorce decree, the husband is transferring property to the wife.

Page 104: Tax - Harvard Law · Web viewAmount Today/Interest Rate/# yrs into the future/total amount in x yrs 2nd way to think of it: ... Alternative: don’t deduct the repayment in year 2,

o The question is whether this is a taxable distribution for the husband.o Court says “yes.” He exchanges this property for the wife’s marital rights

and triggers taxation.o In a footnote, the Court notes that they are not going to treat this as a

taxable event for the wife.– Current state of the law:

IRC § 1041: a transfer of property incident to divorce is not taxable (overrules Davis).

Transferee (e.g., wife) gets carry-over basis, non-taxable event (basically treated like a gift).

Alimony Payor gets a deduction (IRC § 215); recipient must include it in income (IRC § 71). Before 1984, it was a little unclear what constituted “alimony.”

This caused payor to label it as “alimony” (to get the deduction) and recipient to label it as property settlements (which were not taxed).

This caused the government a lot of headaches trying to figure out which it really was.

Congress cleaned up the definition of “alimony” in 1984 [see IRC § 71(b)]: Has to be in cash. Has to be pursuant to the divorce decree or separation agreement. IRC § 71(b)(1)(B) is essentially an election: if you in the divorce decree say it is

alimony (i.e., deductible and included in gross income), it will be treated as such but if you say it is not alimony fine too as long as you don’t take the deduction. If the payor is in a high tax bracket, he would want to characterize as alimony. The payor and recipient will choose whichever option is better tax wise and

then presumably share this tax benefit. This is an example of “private ordering.” – Ring

Can’t live in the same household Ends at death

This is an above-the-line item in § 62, which is beneficial if one gets to deduct it.

Children (child support) IRC § 71(c): child support is not deductible and not includable in gross income.

Payor: if they were supporting a child normally, they would not get a deduction for these expenses (would be paying them with after tax earnings). This is why there is no deduction.

IRC § 152(e)(2): the baseline rule is that the custodial parent gets the child tax exemption. But, this parent can give this tax goodie up to the other parent if they choose

(worth more to the parent in the higher tax rate, who is usually the payor of child support).

– This is another opportunity for “private ordering.” This is something we don’t see very often in the Code

Page 105: Tax - Harvard Law · Web viewAmount Today/Interest Rate/# yrs into the future/total amount in x yrs 2nd way to think of it: ... Alternative: don’t deduct the repayment in year 2,

B. Assignment of Income– What happens when you give people income– You own an asset that produces income. Give that asset to your daughter. Transfer is a gift of income, gifts are not taxable events but any subsequent income is taxable to daughter. – You might want to do this to benefit from your daughter’s lower tax bracket.– IRC § 1(g): the “kiddie tax” - adopted to prevent asset shifting to children

o Basic rule is that any unearned income over $1,500 (adjusted each year) of a child under 14 is taxed at the parents’ marginal tax rate.

– Does not include earned income (e.g., paper route); but would include earnings from your savings account regardless of where the money came from (i.e., earned or unearned).– Child actors – their salary is earned income, but investments have a problem if they earn a lot of money, but if they earn so much, they may be in a high tax bracket themselves.– But the Code limits your ability to do this now. See “kiddie tax” below

Hypothetical #2:– Want to give your daughter half of your salary. What will happen?– You will be taxed on your salary even though you gave half to your daughter. See Lucas v. Earl, below.– The earner is taxed and then we will treat it is as cash gift.

a) Lucas v. Earl (1930) (page 789)– Court says that you cannot contract away your salary to someone else.– This case is about earned income.– When the state imposes splitting by statute (Poe), that is different than contract.

o This is okay, but splitting by contract between private parties is not.

Disparity between one’s ability to give away earned income (human capital) (hypothetical #2) and ability to give away investment/property that produces income (hypothetical #1).

– You are stuck paying taxes on labor income.– You can shift property with income.

Hypo:– Give my daughter a farm

o A gift of property, no tax, and any income that is her’s taxed her rates– Give me daughter income from farm for 5yrs (I keep farm)

o Horst says that it is taxed at my (father’s rates), father is taxed and then give income to daughter and she gets it as a gift

– Gift of property (donee is taxed on income) v. Gift of income from property (donor is taxed on income)

o How do you tell the difference This is the issue in Blair, where Father assigns some of his rights

as beneficiary in a trust to his son and daughter. Has he given them property or has he given them a gift of income

from property? Court says here it is gift of property.

Page 106: Tax - Harvard Law · Web viewAmount Today/Interest Rate/# yrs into the future/total amount in x yrs 2nd way to think of it: ... Alternative: don’t deduct the repayment in year 2,

The father gave a little of everything he had, he gave equal slivers, different picture then when father holds on to the property and gives out income (Horst)

Also he gave a way more than income (the kids gots rights too, could demand accounting, etc.)

Hypo:– Daughter can have my farm and remainder to goes to my niece, how is this treated?

o Tax Daughter in full for whatever he gets out of the property, whatever C gets is his gift

We are treated it as if Daughter is not getting a gift Helvering v. Eubank (1940) (page 848)

– Companion case to Horst.– TP is assigning his future commissions, which he does not have to work for (guaranteed) – Court says this is indistinguishable from Horst, TP/donor is taxed before the gift takes place.– Looks a little like Earl (giving your labor income to someone else) but here also in this case no more work needs to be done.

V. When is an Item Taken into Account?

A. Gains and Losses on the Disposition of Property1. More on Basis

You buy an apartment building worth $200,000. You pay either:1) $100,000 cash: what is your basis? $100,000 even though it is worth 200K2) Securities you own worth $100,000; What is your basis? FMV of property

received or $200,000.– How does this happen?

Philadelphia Park Amusement Co. v. United States (Ct. Cl. 1954) (page 977)– When you exchange property for property, your basis is the FMV of the property you received.– The assumption is that if it is an arms-length transaction, the values of the property exchanged are equivalent.

Hypothetical (based loosely on Philadelphia Park):– TP exchanges franchise for bridge.– A has Bridge: basis is $100, FMV is $150, B has Franchise: basis is $50, FMV is $75.– This hypothetical may seem a little unusual because the values are so different. We would expect the values to be fairly similar in this type of exchange.– This is a taxable exchange:– Amount realized is $75 (FMV of franchise).– Basis is the basis in the property they gave up, $100.– TPs tax position in this current transaction is a loss of $25 for this year.– TPs new basis in the franchise is $75 (FMV).

Page 107: Tax - Harvard Law · Web viewAmount Today/Interest Rate/# yrs into the future/total amount in x yrs 2nd way to think of it: ... Alternative: don’t deduct the repayment in year 2,

– Basis is what is yours ($100 tax free). The basis should not be $150 because he has not paid tax on that $150. He has no right to this as his basis.– The $75 makes sense because he has invested $100 and is recovering $25 in loss from the transaction, so he is left with having $75 that hasn’t been recovered.– Other side of the transaction: tax treatment for franchise owner:– Amount realized = $150; basis = $50; therefore gain of $100.– New basis in the bridge is $150.– This is the rule of Philadelphia Park (FMV of property received).– Has an original basis of $50 and is taxed on an additional $100 in the transaction (you have “invested” a total of $150). He should be able to recover all of this tax free down the road, so his basis is now $150.– Basis is all about keeping track of how much you have already paid taxes on.– So logically this simple rule makes sense, you are never overtaxed or under taxed.

2. Deferred PaymentsBurnet v. Logan (1931) (page 982) Income tax arrives in 1913, take as your basis FMV in 1913, for TP this was $173K TP in 1916 sells her stock to Youngstown for cash (137K) plus royalty contracts

(certain price per ton of future ore removed.) The TP views this sale in 1916 as an “open transaction.” In 1916, the TP does not

know what the amount realized is because she does not know what the royalty income in the future will be. (wait tot tax 137 + royalties > basis)

Because the cash paid at closing did not cover TP’s basis, the TP wants to take in royalty payments until they reach her basis; then start paying taxes on the payments after that. This is basically deferring taxes into the future.

IRS claims that this a closed transaction - thinks that you can calculate the value paid in 1916. (AR = 137K + FMV of the contract) She sold her stock for cash + property. The Service thinks they can value this

property (contract). If this is a closed transaction, the contract income that is coming in the years after

closing should be treated as regular income but would get to amortize the cost of the contract. You could think about this as a transaction where the value of the cash and

property at time of closing would be reflected in the basis of her new stock.– The Court accepts the TPs view.

The TP is getting the value of deferring taxes. Since this case, it is hard to get the courts to accept that you really can’t value a

contract. We do not like to find “open” transactions. Most cases like this will now fall under the installment sale doctrine (under certain circumstances you can spread the gain out over multiple years).

3. Non-Recognition– Nonrecognition v. realization; sometimes you may realize but not recognize (don’t have to put it on your return), usually assume that if it realized it is recognized

o Policies to encourage certain kind of exchanges/investments o Not enough of a shift of investment to merit taxation

Page 108: Tax - Harvard Law · Web viewAmount Today/Interest Rate/# yrs into the future/total amount in x yrs 2nd way to think of it: ... Alternative: don’t deduct the repayment in year 2,

o §1033, not your choice, not in your control, it seems fair to force you to recognize

Hypotheticals:1) Ring exchanges a Boeing 747 for a farm. Is this a taxable event? Yes.

There is nothing in the §1031/Code that would override the general rule governing exchanges.

2) Ring exchanges one farm for another farm. This may be a moment where we could tax this, but choose not to recognize it. IRC § 1031 might override the general rule that when you have an exchange, you

have to realize gain or loss. If the exchange is for “like kind” there is an exception.

3) Ring exchanged GM stock for GE stock. You might think this would be a like kind exchange (stock for stock) but there is

specific exception for stocks, so that stocks do not fall under “like kind.”

Revenue Ruling 79-143 (1979) (page 273) This ruling determined whether the US $20 gold coin and South African Krugerrand

were considered “like kind” for purposes of IRC § 1031. The Service held that the coins were not like kind because they were different

investments (one was important for its gold content (Krugerrand), the other for its historical value (American gold coins).

The ruling gives us some perspective on what it means to be “like kind.”

IRC § 1031(e) Explicitly states that exchange of livestock of different sexes is not “like kind.”

Handout - “Basis Aspects of Section 1031” Exchanges of Greenacre for Blueacre.

1) “Like kind” transaction: no gain is recognized. IRC § 1031(d) provides the rule for determining the basis in the new property. In the simple case, it is basically carry-over basis (e.g., carry over the basis

from Greenacre to Blueacre).2) “Like kind” transaction with “boot”

IRC § 1031(b): if you fall under the “like kind” provisions, but you also receive some “boot,” you have to recognize your gain to the extent of the “boot.”

In this case, Blueacre has to recognize gain up to $25 (the “boot”). Aside note: if instead, Blueacre had a loss (he would want to recognize

this loss), he cannot use this “boot” strategy to recognize loss. IRC § 1031(c) prohibits recognizing loss in these cases.

B’s basis in Greenacre does not include the “boot,” because that portion is recognized as gain. IRC § 1031(d): basis is decreased by amount of cash ($25) and increased

by the amount of gain recognized ($25) = $75.

Page 109: Tax - Harvard Law · Web viewAmount Today/Interest Rate/# yrs into the future/total amount in x yrs 2nd way to think of it: ... Alternative: don’t deduct the repayment in year 2,

3) “Like kind” transaction with a “boot” being a painting worth $25 with a basis of $10 The painting does not constitute a like kind transaction. Its tax treatment is

the equivalent of G selling the painting and using the proceeds as “boot” for the Blueacre transaction.

AR = $125 Basis = $60 = $15 for painting $50 on property IRC § 1031(d) provides formula new basis for G: original basis ($60) +

amount realized ($15) = $75. IRC § 1031(d) provides basis for B (at the end of the provision). $25 is

recognized for B, you get taxed on your gain to the extent you received boot, something other than the like-kind. $25 basis (FMV) in the painting and $75 for the real estate. Essentially same as cash example but the painting triggers some additional gain. Ring will leave us to look at this.

Ring encourages us to go over this on our own to figure it out.

“Like Kind” - IRC § 1031(a) To qualify for this treatment:

Has to be held for a productive trade of business or for investment; and Can’t fall into the list of exceptions (e.g. stocks, bonds, etc.).

– IRC § 1033(a): involuntary conversions If the property is by virtue of being destroyed, stolen, condemned, etc. converted into

similar property, we don’t require you to recognize gain or loss.

IRC § 121: Adopted in 1997. If you own and live in the house for 2 of the last 5 years, you can get an exclusion of

up to $250,000 (single) or $500,000 (married) from gain. You can do this every two years.

This is an example of a place where the Code does not require you to recognize gain even though the transaction is a taxable event. Possible rationales for this rule:

1) Liquidity in market.2) Retirement type vision - help people who are shifting home asset to savings in

anticipation of retirement.3) Capital gain “lockin” - people are hesitant to sell because of large capital gain

liability.

B. Leverage and Deferral 1. Borrowing against appreciation

– Corporation has the property and want a high basis by saying there was a realization event when Mrs. Wood borrowed against the appreciation – Mrs. Wood borrowed heavily against this property (with low basis and high FMV) using nonrecourse borrowings.

Page 110: Tax - Harvard Law · Web viewAmount Today/Interest Rate/# yrs into the future/total amount in x yrs 2nd way to think of it: ... Alternative: don’t deduct the repayment in year 2,

o Nonrecourse: lender only has recourse against the land (i.e., not against the borrower).

o Simple hypothetical to understand this:– TP has property with basis of $5 and FMV of $100. TP borrows $100 nonrecourse with the property as collateral. Should these borrowings be considered income for the TP?

o No - it is just a “loan” and we normally say that borrowing is not taxable.o Yes - this is not a regular “loan”; there is no obligation to repay the loan in

a non-recourse situation because you can walk away from the deal, this more like a sale with an option to buy back.

o The original reason we said borrowings were not taxable is because of the obligation to repay.

– Two ways to view nonrecourse mortgages:1) It is just like a loan plus an option to put (i.e., sell) the property for $100.

o Not a sale nor a taxable event.2) Non-recourse mortgages are different. It is more realistic to view it as if the

TP sold the property to the bank for $100 and holds an option to buy it back from the bank for $100 at a later date.

Yes it is a sale; yes it is a taxable event.– In Woodsam, they are arguing over the appropriate basis.

o If you view a non-recourse loan as a taxable event, you have to give the TP credit for this by increasing the basis as you borrow.

o The corporation had stepped into Mrs. Wood’s shoes (i.e., assumed her basis) and was arguing that the basis was high because it should have been increased alongside the non-recourse borrowings (and Mrs. Wood should have been paying taxes on these borrowings).

o The corporation is arguing a position that seems contrary to most TP’s positions (most TPs do not want to be taxed at time of borrowing) because many of the years where they would have had to pay taxes under their view are now “closed years” where the IRS is not going to go back and assess deficiencies.

– The court says that it does not view nonrecourse loans differently.o Borrowing is not a taxable event; it does not create a taxable event and

does not increase your basis.– After Woodsam, if you have lots of appreciated property, you should borrow against it to live. When you die, your appreciated property basis is stepped up and can be used to pay off the debt without the appreciation every being taxed.

2. Effect of borrowing on basisCrane v. Commissioner (1947) (page 344) (continued)

– Facts of the hypothetical we will use:o TP inherits or buys property worth $260,000 with a non-recourse

mortgage of $260,000 attached.o TP takes depreciation deductions of $28,000.o TP sells the property with mortgage attached for $2,500.

Page 111: Tax - Harvard Law · Web viewAmount Today/Interest Rate/# yrs into the future/total amount in x yrs 2nd way to think of it: ... Alternative: don’t deduct the repayment in year 2,

– TP never put any cash into this property at the end of the she walks away with $2,500 in real cash (hopefully whatever tax scheme we use will reflect this) – The main tax question is what is the basis (although the Crane case was really more focused on amount realized). The tax question was whether the basis includes non-recourse debts that were used to buy the property.– TP argues that she has no equity. None of her own money is invested in this property and therefore she should have zero basis in the property. Because she received $2,500 and had a basis of zero, her taxable income should be $2,500.

o The problem with this view is that the TP took $28,000 in deductions on this property. This is inconsistent with her assertion that her basis is zero.

o If you accept the TP’s view, she got real world cash flows of $25,500 (deductions less gain she is willing to recognize). The TP would reflect a net loser of $25,500 clearly this is not what’s going on.

– The Service says that amount realized should include the cash received ($2,500) as well as the mortgage taken off your hands ($260,000). The TP’s basis should include the money she borrowed to buy the property ($260,000) and should be less any depreciation deductions ($28,000). AR = $262,500 basis is $232,000 this gives a gain $30,500 minus her deductions leaves a tax picture of $2,500 which is intuitively right. – If you pretend borrowing non-recourse is regular loan we will also treat as just a loan when it is transferred. – Judge Magruder’s concurrence accepts the idea of negative basis.

o Judge Magruder wrote the opinion in Parker v. Delaney (1st Cir. 1950) (page 351).

o TP’s basis of zero less deductions of $28,00 equals a basis of ($28,000).o His solution is not to bring the mortgage into the picture, but to account

for the deductions by using a negative basis.o This is not the law, but Magruder’s view gets you to the same place as the

Service.o The implication of this view is that it should only be your own and not

borrowed money in the basis.o This view also implies that we should not be allowing depreciation

deductions when the basis is zero.–

TP IRS Judge MagruderAR 2,500 262,500 2,500Basis 0 232,000 (28,000)G/L 2,500 30,500 30,500Previous Depreciation -28,000 -28,000 -28,000Overall Tax Impact -25,500 2,500 2,500

Hypothetical:– Ring buys a $10 million piece of property with $1,000 of her money and the rest non-recourse mortgage.– Ring’s basis is $10 million.– She would get depreciation deductions based on this $10 million basis. This would create enormous deductions for only having invested $1,000.

Page 112: Tax - Harvard Law · Web viewAmount Today/Interest Rate/# yrs into the future/total amount in x yrs 2nd way to think of it: ... Alternative: don’t deduct the repayment in year 2,

– This idea that non-recourse loans goes into basis came from Crane and really gave rise to a boom in tax shelters and led to lots of abuses in the 80s.– So far:

o Borrow against appreciation no realizationo When you borrow against property that goes into your basiso When you sell the property the mortgage goes into AR

Estate of Franklin v. Commissioner (9th Cir. 1976) (page 741)– Involves transaction in which the Romneys sold Thunderbird Inn to TP who leased it back.

o TP claims sale and then lease (two separate transactions and wants to take depreciation deduction).

o Service claims that it is an option.– Doesn’t seem like TP really owns the property because they are being paid $9,000 a month paid for rent and paid back as interest.

o We are left with the $75,000 prepaid interest paid at the beginning and the balloon payment at the end.

o It looks like the TPs (Associates) bought an option. Paid $75,000 today for the right to buy the property in the future for the balloon payment.

– This transaction benefits the TPs because for very little money of their own ($75,000), they have a basis of $1.224 million on which they can take depreciation deductions.– The court doesn’t quite focus on the option image, but focuses on the fact that the purchase price for this Inn was likely less than its FMV.

o We have to be careful with this approach though because we don’t want to ensnare a seller who merely got a very good deal.

o The court seems to know what is going on here and doesn’t like it and says you don’t have real property to depreciate until value of property exceeds the debt, sort of like a sham but not quite

3. Disposition of mortgaged property

Commissioner v. Tufts (1983) (page 356)– Mortgage debt more than FMV – Modified hypothetical to make the numbers a little easier:

Non-recourse debt 100,000Basis (see Crane) 100,000Depreciation, IRC § 167 60,000Repay Principal 10,000FMV of Property Now 70,000Now Debt 90,000Basis 40,000

($100,00 - $60,000)

Page 113: Tax - Harvard Law · Web viewAmount Today/Interest Rate/# yrs into the future/total amount in x yrs 2nd way to think of it: ... Alternative: don’t deduct the repayment in year 2,

– In the real world, the TP in this hypothetical is out $10,000 cash, in real world he got back 0, overall it was -$10K economic experience.– The FMV of the property ($70,000) is less than the amount of the non-recourse loan ($90,000).– TP gives the property to someone for no $ they just take over the mortgage and property. – How should this disposition be taxed?

o TP view: amount realized should not include the face of the note, because the note is now worth less than its face value, it should be FMV ($70,000), the maximum the lender can get from me in the form of property is $70,000 for my $90,000 debt.

o TP relies on a footnote in Crane for this view. The Crane Court used the idea of taxpayer’s benefit to the extent of the mortgage as a justification for including the mortgage in the amount realized. They set aside this kind of case in a footnote and said that it may be different (this case doesn’t fit very well with the “benefit” language of Crane).

TP and Crane footnote 37 TuftsAR 70,000 90,000Basis 40,000 40,000G/L 30,000 50,000Depreciation -60,000 -60,000Net Cash -30,000 -10,000

– Getting too good of deal in the real world under the TP’s view.– Court says that amount realized is the amount that you still owe on the mortgage, regardless of FMV. Whatever you put in basis has to be put in AR.

o This matches our real world understanding of the TP’s situation.o Justifies this under parallel treatment (as opposed to the more “benefits”

language of Crane).– Ring - “Be comfortable with this. It underlies so many of the ideas and strategies in tax.” The idea that you can borrow money and put it into basis is not the only way to go (we could have adopted Judge Magruder’s view). The decision to do this in Crane, which led to Tufts, is an important decision.

Annual reporting– Can constitutionally do it on an annual basis

o But, §172 (c) can carry NOL, but it is mostly business deduction, not quite that narrow but primarily in that direction

– Accounting methodso Cash – report income when you get the cash (this is how we’ve done in

class)o Accrual – it is income when you earn it (if you build in Dec but don’t get

paid in January, it is income in December)VI. Capital Assets

Introduction– We have saved this for last because it is a special regime.

Page 114: Tax - Harvard Law · Web viewAmount Today/Interest Rate/# yrs into the future/total amount in x yrs 2nd way to think of it: ... Alternative: don’t deduct the repayment in year 2,

– Why does it matter if something is a capital asset?o Gains on sale or disposition of these assets may be taxed at different rates.o May be limits to the amount of otherwise deductible losses.o Other places in the Code have tapped into this distinction (such as gifts of

appreciated property).– The differences that result from this distinction between capital and other assets sets up all sorts of games and strategies.– The rules that determine a capital asset are complex (involve asset classification, how long you hold the asset, etc.).– Note that capital gains have nothing to do with capital expenditures.

– Must have the sale of capital assets and might have preferential capital gains rates o Instead of having one special capital gains rates we have many

– Losses are subject to restrictions

2003 – 15% CG rate for anyone in 25%-35% OI brackets– 5% CG rate for 10% and 15% OI brackets– Other exceptions for certain types of assets

1) Take all G/L from sales of Cap Assets and divide into Long Term (more than one year) & Short-term.

2) Net the LTKG – LTKLa. Losses or gains?

3) Net the STKG – STKLa. Losses or gains?

– When you combine these there are 4 possible outcomes: If both produce gains, short-term gains are taxed at a regular rate;

long-term gains are taxed at their own special rate (don’t worry about the mechanics of this).

§ 1211 – if there are capital losses in both short-term and long-term categories, you can use them to reduce up to 3K of ordinary income; the rest of your capital losses can be carried over into the next year. When you carry over into the next year, you aggregate.

Long-term gains but short-term losses – then, net the two If losses are greater than gains, go to § 1211 and do the

same thing as in last category If the gains are greater than losses, you have net long-term

gains and you calculate net long-term gains at special long-term rates

Long-term loss but short-term gain – net the two. If the gain is greater than the loss, ordinary income. If loss is greater than the gain, go to § 1211.

2 Negatives to capital losses:

Page 115: Tax - Harvard Law · Web viewAmount Today/Interest Rate/# yrs into the future/total amount in x yrs 2nd way to think of it: ... Alternative: don’t deduct the repayment in year 2,

1) Capital losses go against capital gains; but capital gains are the lowest tax rate. You would rather have an ordinary loss than a capital loss in order to offset ordinary income (taxed at a higher rate than capital gains).

2) They are limited.

RationaleWhy do we have a special rate structure for capital gains (and attacks on these rationales)?1) Incentives to invest. Want people to save instead of spend.

Attack: perverse incentives (the idea of “target savings” – people will not save beyond a given target).

Attack: this is not enough if you really want to encourage savings and investment (the gains are still taxed at least some).

Attack: distorts behavior to comply with the special benefits (e.g. companies not paying dividends to get stock appreciation).

At the end of the day, if you buy this rationale, you have to buy the proposition that the loss to the public fisc would be less than the gain resulting from the increased investment of this provision.

2) Break the “lock-in” effect - in a realization based system, high taxes on realization may deter you from moving to another investment that would be efficient. Attack: not really enough to eliminate the lock-in effect. We still have some lock-

in because of the taxes and holding period limitations. Attack: capital gain benefits do not require you to reinvest. If we really want to

break the lock-in, we need to ensure reinvestment.3) Reduce the harmful effects of “bunching” - taxing all of your accumulated gain in one

year. Attack: TP has a lot of control over when he realizes the gain. Attack: TPs that get the benefits are typically higher bracket tax payers who so

not get “bunched” into a higher tax bracket. Attack: bunching is the price you pay for the benefit of deferral. Attack: you can start getting good rates in a short period of time. In these cases,

bunching does not seem like much of a concern. Attack: the current rate differentials (which are much flatter than in the 1980s)

minimize the impact of bunching.4) Account roughly for inflation (which is essentially taxed by capital gains).

Attack: there is no relationship between the inflation and your tax rate. Attack: if we were really going to adjust for inflation, we might want to focus on

your basis and the present value of your investment (instead of focusing on the gain in the capital gains system).

Think about how the arguments pro and con may make sense for some assets in some markets and not in others.

– Spring 2003, certain dividends can be taxed at ltkg, but do not get put into the formula, the formula is for sales of capital assets KG and KL

Why do we have detrimental treatment for losses?

Page 116: Tax - Harvard Law · Web viewAmount Today/Interest Rate/# yrs into the future/total amount in x yrs 2nd way to think of it: ... Alternative: don’t deduct the repayment in year 2,

1) Limit “culling/cherry picking” - in the absence of IRC § 1211, a TP could sell a big loser to offset ordinary income. The government is worried about the package of effects - “gaming” by the

taxpayer. TPs will use realization to try and screw the government; always get rid of the

losers to offset your ordinary income. This is what IRC § 1211 prevents.2) To balance out the good treatment we give KG3) Historic – Depression, if we let unlimited KL in 1934, there would be no tax base

Statutory Requirements1) Have a sale or exchange

o We now have IRC § 1231 to handle things like involuntary conversions. Handles dispositions that were not technically sales or exchanges.

2) Need to have a capital asset3) Holding period requirement

o More than year, has been as little as 6 months in the past

Capital Asset IRC § 1221 - broadly defines capital asset with five exceptions (inventory, accounts

receivable). Almost a negative definition (tells you what it is by telling you what it isn’t). For example, it’s not inventory.

Just because some items are not capital assets does not mean it will never get the good rates

Inventory Exception: Language of “to customers” adopted in 1934 because of the depression.

Government did not want “traders” who had large losses on stocks to take a deduction and get ordinary loss treatment.

Need to be a dealer and not a trader to get this treatment now. Example:

Lawnmowers you sell as inventory would be taxed as ordinary income.

Corn Products Refining Co. v. Commissioner (1955) (page 1052) Involved the tax treatment of a corn futures asset.

These corn future contracts are exchange traded. Give you the right to buy a certain amount at a given price some date in the future.

Why would TP buy these contracts?1) Speculate on the market.2) Hedge against future increases in corn prices. It appears that the TP in this case was primarily concerned with the hedging

function of these contracts. Guarantees the TP access to corn. TP has a huge gain on sale of these contacts and claims it is a taxable gain.

Claims that the definition of capital gains is broad with five specific exceptions. When you march through the exceptions, none seem to fit these contracts. Therefore, they are capital assets.

Page 117: Tax - Harvard Law · Web viewAmount Today/Interest Rate/# yrs into the future/total amount in x yrs 2nd way to think of it: ... Alternative: don’t deduct the repayment in year 2,

IRS agrees that these contracts are like “inventory” Capital gains are a benefit. We should therefore construe the definition of capital

assets to be narrow and the definitions of the exceptions to be broad. This is something that you bought for a business purpose and you should

therefore get ordinary income treatment. The Court construes definition narrowly and exceptions broadly.

Views the contracts as a “substitute” for corn. It allows them not to have to build the storage and inventory the corn now.

IRS wins the case; this is not held to be a capital asset.

What is a “whipsaw”? Heads I win; tails you lose.

TP always win.

What whipsaw is the government worried about in the corn future? If TP wins and can say that the corn future is a capital asset, if the corn future has

income (profit), they can sell the corn future and only have a capital gain. If the future declines in value, they will take delivery of corn (which is not a taxable event); and sell the corn in the market at a loss (for an ordinary loss). All losses will be ordinary and all gains will be capital. TP will always win.

Arkansas Best Corporation v. Commissioner (1988) (page 1057) TP wants an ordinary loss on some bank stock it sold. Believes that at least to their

later purchases of stock, they deserve an ordinary loss because this was an asset they held for a business purpose (relies on Corn Products for this idea).

Court revisits Corn Products. Does not allow TP an ordinary loss. Everyone has overstated and misunderstood Corn Products. That case was really

a focus on inventory and inventory substitutes (not held for business purpose).

IRC § 1221(2) It looks like depreciable property falls out of capital asset treatment. You actually fall out of IRC § 1221 and into IRC § 1231, which is in some ways even

better than capital asset treatment. This helps out taxpayers, but we will not talk about it.

I make a painting and sell it:– OI, 1221 (a) (3)

What if I give it to you and you want to sell it?– If you made it for me that is a problem, also looks at (c) which says OI if your basis is the original creator

Patents– You can always get capital gains treatment.– Why can these always get the treatment, but copyrights cannot? You can think about this

Page 118: Tax - Harvard Law · Web viewAmount Today/Interest Rate/# yrs into the future/total amount in x yrs 2nd way to think of it: ... Alternative: don’t deduct the repayment in year 2,

Stern v. United States (5th Cir. 1959) (page 1101)– The question was basically what was the character “Francis the Talking Mule” when he was sold.– Wants to take sale of the talking mule as a capital gain rather than ordinary income

o Not inventory, so OK thereo § 1201(a)(3) – certain literary works that one makes and sells aren’t

considered capital gains– TP’s argument:

o Sold him for a movie and not a novel– The court said that TP sold the idea of Francis and he has to pay. He may be carved out of a story but he’s still a literary creation.– TP loses.

o Gives you a feel for how the court will try and keep you in these exceptions.

Exam:– Key is to argue both sides

Wed: 1:30-300Friday 2:00-4:00Monday 12:00-2:00